Esercitazioni di FIsica Generale II - Alessandro Strumia

Università di Pisa
Corso di laurea in Fisica
Esercitazioni di Elettromagnetismo
www.df.unipi.it/˜astrumia/fisica2.html
Ultimo aggiornamento: 4 maggio 2005 (capitolo 14)
2 Conduttori
Es 31 Piano conduttore . . . . . . . . . .
Es 32 Lastra conduttrice . . . . . . . . . .
Es 33 Metodo delle immagini . . . . . . .
Es 34 Piano carico fra 2 piani conduttori .
Es 35 Carica fra 2 piani conduttori . . . .
Es 36 1 lastre conduttrice carica . . . . .
Es 37 2 lastre conduttrici cariche . . . . .
Es 38 Capacitatore cilindrico . . . . . . .
Es 39 Capacitatore di dimensioni variabili
Es 40 Conduttore in capacitatore . . . . .
Es 41 Sfera conduttrice a terra . . . . . .
Es 42 Sfera conduttrice isolata . . . . . .
Es 43 Sfera conduttrice in E costante . .
Es 44 Tetraedro conduttore . . . . . . . .
Es 45 Condensatore sferico . . . . . . . .
Es 46 Condensatori in serie . . . . . . . .
Es 47 Effetto delle punte . . . . . . . . . .
Es 48 Sfera conduttrice bucata . . . . . .
Es 49 Carica dentro sfera . . . . . . . . .
Indice
I
Elettrostatica
1 Campi e potenziali elettrici
Es 1
Gravità vs elettromagnetismo . . . .
Es 2
Rompere una bacchetta . . . . . . . .
Es 3
Sistemi stabili? . . . . . . . . . . . .
Es 4
Campo elettrico di un filo . . . . . .
Es 5
Campo elettrico di un piano . . . . .
Es 6
Campo elettrico di una sfera . . . . .
Es 7
Energia di un guscio sferico . . . . . .
Es 8
Modelli dell’atomo . . . . . . . . . . .
Es 9
Energia di una sfera . . . . . . . . . .
Es 10 Raggio classico dell’elettrone . . . . .
Es 11 Masse dei nuclei . . . . . . . . . . . .
Es 12 Differenza di massa protone-neutrone
Es 13 Nucleo che si spezza . . . . . . . . . .
Es 14 Energia di due cariche . . . . . . . .
Es 15 Forza su cariche superficiali . . . . .
Es 16 Scattering debole . . . . . . . . . . .
Es 17 Scattering Rutherford . . . . . . . . .
Es 18 Esplosione Coulombiana . . . . . . .
Es 19 Sfera polarizzata . . . . . . . . . . . .
Es 20 Cilindro polarizzato . . . . . . . . . .
Es 21 Formule di base sui dipoli . . . . . .
Es 22 Paradosso sui dipoli I . . . . . . . . .
Es 23 Paradosso sui dipoli II . . . . . . . .
Es 24 Paradosso sui dipoli III . . . . . . . .
Es 25 Paradosso sui dipoli IV . . . . . . . .
Es 26 Allineamento di dipoli elettrici . . . .
Es 27 Coordinate polari . . . . . . . . . . .
Es 28 Laplaciano . . . . . . . . . . . . . . .
Es 29 Potenziale di Yukawa . . . . . . . . .
Es 30 Atomo di idrogeno quantistico . . . .
4
.
.
.
.
.
.
.
.
.
.
.
.
.
.
.
.
.
.
.
21
21
21
22
22
23
23
23
23
24
25
25
26
26
27
27
27
27
28
28
5
5
5
5
7
7
8
8
9
9
3 Dielettrici
29
10
Es 50 2 dielettrici in condensatore piano . . 29
11
Es 51 N dielettrici in condensatore piano . 29
11
Es 52 Condensatore in acqua . . . . . . . . 30
12
Es 53 Carica davanti a semipiano dielettrico 30
12
Es 54 Dielettrico in condensatore . . . . . . 31
12
Es 55 Forza di conduttore su dielettrico . . 31
13
Es 56 Dielettrico in campo esterno . . . . . 32
13
Es 57 Buco in dielettrico . . . . . . . . . . . 32
14
Es 58 Sfera dielettrica in dielettrico . . . . . 33
15
Es 59 Attrazione fra dielettrici . . . . . . . 33
16
16
35
16 4 Correnti
Es 60 Capacitatore piano imperfetto . . . . 35
16
Es 61 Scarica di sfera carica . . . . . . . . . 35
17
Es 62 Resistenza fra sfere concentriche . . . 35
17
Es 63 Sonda marina . . . . . . . . . . . . . 36
17
Es 64 Semipiano dielettrico imperfetto . . . 36
18
Es 65 Diodo termoionico . . . . . . . . . . . 36
19
Es 66 Piatto dielettrico . . . . . . . . . . . 37
19
20
Es 67 Sfera dielettrica . . . . . . . . . . . . 37
1
2
Indice
5 Circuiti
Es 68 Resistenze in parallelo . . . . .
Es 69 Resistenze su cubo . . . . . . .
Es 70 Ponte di Wheatstone . . . . . .
Es 71 Impedenze . . . . . . . . . . . .
Es 72 Potenza dissipata . . . . . . . .
Es 73 Filtro che taglia frequenze alte .
Es 74 Filtro che taglia frequenze basse
Es 75 Pendolo accoppiato . . . . . . .
Es 76 Attenuatore . . . . . . . . . . .
Es 77 Catena LC . . . . . . . . . . . .
II
.
.
.
.
.
.
.
.
.
.
.
.
.
.
.
.
.
.
.
.
.
.
.
.
.
.
.
.
.
.
.
.
.
.
.
.
.
.
.
.
.
.
.
.
.
.
.
.
.
.
.
.
.
.
.
.
.
.
.
.
.
.
.
.
.
.
.
.
.
Magnetostatica
6 Campi magnetici
Es 78 Forza fra 2 cariche . . . . .
Es 79 Disco di Rowland . . . . .
Es 80 Attrazione o repulsione? .
Es 81 Filo rettilineo . . . . . . .
Es 82 Cavo coassiale . . . . . . .
Es 83 Spira circolare . . . . . . .
Es 84 Due spire circolari . . . . .
Es 85 Filo a U . . . . . . . . . .
Es 86 Piano a U . . . . . . . . .
Es 87 Solenoide rettilineo infinito
Es 88 Solenoide rettilineo finito .
Es 89 Solenoide toroidale . . . .
Es 90 Sfera ruotante . . . . . . .
7 Moto
Es 91
Es 92
Es 93
Es 94
Es 95
Es 96
Es 97
Es 98
Es 99
Es 100
Es 101
Es 102
Es 103
Es 104
Es 105
Es 106
Es 107
.
.
.
.
.
.
.
.
.
.
.
.
.
.
.
.
.
.
.
.
.
.
.
.
.
.
.
.
.
.
.
.
.
.
.
.
.
.
.
in campo magnetico esterno
Trottola magnetica . . . . . . . . . .
Cilindro su piano inclinato . . . . . .
Ago magnetico . . . . . . . . . . . . .
Carica in B costante . . . . . . . . .
Campo magnetico galattico . . . . . .
Spettrometro . . . . . . . . . . . . . .
Carica in B ed E costanti . . . . . .
Fotomoltiplicatore in B, E . . . . . .
Ciclotrone . . . . . . . . . . . . . . .
Carica in B con direzione non uniforme
Carica in B con modulo non uniforme
Carica in B(t) uniforme . . . . . . . .
Atomo in B(t) uniforme . . . . . . .
Carica in B non uniforme . . . . . . .
Carica in quadrupolo magnetico . . .
Carica in quadrupoli magnetici . . . .
Ottica geometrica matriciale . . . . .
8 Induzione magnetica
Es 108 Circuito allungato . . . . . .
Es 109 Cilindro ruotante . . . . . .
Es 110 Generatore . . . . . . . . . .
Es 111 Generatore in orbita . . . .
Es 112 Trasformatore . . . . . . . .
Es 113 Trasformatore con due spire
Es 114 Induzione . . . . . . . . . . .
Es 115 Trapano . . . . . . . . . . .
.
.
.
.
.
.
.
.
.
.
.
.
.
.
.
.
.
.
.
.
.
.
.
.
.
.
.
.
.
.
.
.
.
.
.
.
.
.
.
.
39 9 Forze magnetiche fra circuiti
39
Es 116 Due circuiti lunghi . . . . . . .
39
Es 117 Rotazione di due spire circolari
39
Es 118 Una spira ed un dipolo . . . . .
40
Es 119 Monopolo magnetico . . . . . .
40
Es 120 Traslazione di due spire circolari
40
41 10 Campi magnetici nella materia
Es 121 Sbarra magnetizzata . . . . . .
41
41
Es 122 Materiali ferromagnetici . . . .
42
Es 123 Due bacchette . . . . . . . . . .
Es 124 Correnti parassite . . . . . . . .
Es 125 Correnti parassite . . . . . . . .
44
Es 126 In pratica . . . . . . . . . . . .
45
45 III Elettrodinamica
45
46 11 Corrente di spostamento
46
Es 127 Scarica di un filo . . . . . .
46
Es 128 Piano con carica ondulata
46
Es 129 Sfera radioattiva . . . . . .
47
Es 130 Scarica di un condensatore
47
Es 131 Condensatore in alternata
47
Es 132 Cavità risuonante . . . . .
47
Es 133 Effetto pelle . . . . . . . .
48
Es 134 Filo conduttore interrotto
48
Es 135 Due cilindri cavi . . . . . .
48
Es 136 Carica in moto . . . . . . .
.
.
.
.
.
.
.
.
.
.
.
.
.
.
.
63
63
63
64
65
66
.
.
.
.
.
.
.
.
.
.
.
.
.
.
.
.
.
.
69
69
69
70
70
70
71
72
.
.
.
.
.
.
.
.
.
.
.
.
.
.
.
.
.
.
.
.
.
.
.
.
.
.
.
.
.
.
50
12 Onde e oscillazioni
50
Es 137 Sorgenti di onde . . . . . . . . .
50
Es 138 Ricevitore di onde . . . . . . . .
51
Es 139 Luce solare . . . . . . . . . . . .
51
Es 140 Vettore di Poynting . . . . . . .
51
Es 141 Rilfessione di onde in una corda
52
Es 142 Rifrazione . . . . . . . . . . . .
52
Es 143 Rifrazione . . . . . . . . . . . .
52
Es 144 Riflessione da un metallo . . . .
53
Es 145 Onde adiabatiche . . . . . . . .
54
Es 146 Guida d’onda . . . . . . . . . .
54
Es 147 Cavità risuonante . . . . . . . .
55
Es 148 Pressione di radiazione . . . . .
56
Es 149 Velocità di gruppo . . . . . . . .
56
Es 150 Pulsar . . . . . . . . . . . . . .
56
Es 151 Interferenza . . . . . . . . . . .
57
58
13 Irraggiamento
Es 152 Atomo di idrogeno . . . . . . . .
59
Es 153 Scattering elettrone/nucleo . . .
59
Es 154 Scattering ee . . . . . . . . . . .
59
Es 155 Onde gravitazionali . . . . . . .
59
Es 156 Scattering elettrone/fotone . . .
60
Es 157 Polarizzazione della CMB . . . .
60
Es 158 Un’antenna . . . . . . . . . . . .
60
Es 159 Due antenne . . . . . . . . . . .
61
Es 160 Dipolo magnetico . . . . . . . .
61
.
.
.
.
.
.
.
.
.
.
.
.
.
.
.
.
.
.
.
.
.
.
.
.
.
.
.
.
.
.
73
73
73
74
74
75
76
76
77
78
79
.
.
.
.
.
.
.
.
.
.
.
.
.
.
.
.
.
.
.
.
.
.
.
.
.
.
.
.
.
.
.
.
.
.
.
.
.
.
.
.
.
.
.
.
.
80
80
80
81
81
82
83
83
84
86
86
87
87
88
88
88
.
.
.
.
.
.
.
.
.
.
.
.
.
.
.
.
.
.
.
.
.
.
.
.
.
.
.
90
90
90
91
91
92
92
92
92
93
Indice
14 Relatività
Es 161 Contrazione di Lorentz . . . . .
Es 162 Che cosa è l’elettromagnetismo
Es 163 Forza fra 2 cariche bis . . . . . .
Es 164 Scattering debole bis . . . . . .
Es 165 Carica in E e B ortogonali bis .
Es 166 Filo in moto . . . . . . . . . . .
Es 167 Forza prodotta da filo in moto .
Es 168 Onda vista da sistema in moto .
3
.
.
.
.
.
.
.
.
.
.
.
.
.
.
.
.
.
.
.
.
.
.
.
.
94
94
94
95
95
96
96
96
97
Es 169
Es 170
Es 171
Es 172
Es 173
Es 174
Es 175
Es 176
Riflessione da specchio in
Aberrazione relativistica
π 0 → 2γ . . . . . . . . .
GZK . . . . . . . . . . .
Effetto Compton . . . . .
Esperienza d Fizeau . . .
Iraggiamento da elettroni
Miraggi . . . . . . . . . .
moto . . . 97
. . . . . . . 97
. . . . . . . 97
. . . . . . . 98
. . . . . . . 99
. . . . . . . 99
relativistici 99
. . . . . . . 100
Parte I
Elettrostatica
Capitolo 1
Campi e potenziali elettrici
Forza di Coulomb: F = kr̂q1 q2 /r2 dove k = 8.9875 109 N m2 /C 2 , riscritto in termini della ‘costante dielettrica
del vuoto’ 0 come k = 1/4π0 con 0 = 8.8542 10−12 C 2 / N m2 . Altre unità usate altrove sono k = 1, k = 1/4π.
È utile introdurre il campo elettrico E, ed il potenziale elettrico ϕ. La forza di Coulomb F ∝ rp ha importanti
proprietà speciali vere solo per p = −2, che rendono possibile reinterpretarla come ‘teorema di Gauss’ Φ(E) =
Qin /0 e poi come
2
∇ · E = ρ/0
∇ ϕ = −ρ/0
↔
∇×E = 0
E = −∇ϕ
(Analogamente a come ẍ = a è equivalente ma più utile di x = 12 at2 ). Energia elettromagnetica e sua densità
Z
Z
Z
X qi qj
1
0
1 X qi qj
1
ρ(x1 )ρ(x2 )
U=
dV1 dV2
=
dV ρϕ =
dV E 2
=
=
4π
r
2
4π
r
2
4π
|x
−
x
|
2
2
0 ij
0 ij
0 1
2
i>j
i6=j
Un Coulomb sono 6.24 1018 elettroni. Una unità di misura molto usata è l’elettron-volt eV = J(qe /C) =
J/6.24 1018 , che è l’energia che un singolo elettrone acquista passando per una differenza di potenziale di un
Volt.
Esercizio 1: Gravità vs elettromagnetismo
Un atomo di idrogeno è composto da un elettrone e da un protone (con massa me = 0.911 10−30 kg e mp ≈
1836me ) a distanza circa Å = 10−8 cm. Calcolare la forza elettrica e gravitazionale.
2
bSoluzione: La forza elettrica ha un valore quasi macroscopico FC ≈ ke2 /Å ≈ 10−8 N. La forza gravitazionale
è invece trascurabile:
FN
GN me mp
=
= 4.4 10−40 N
FC
ke2
Sebbene esistano motivi plasusibili non si sa da dove un rapporto cosı̀ grosso esca fuori.
Le particelle si combinano formando materia neutra in modo da cancellare, in media, l’enorme forza elettrica.
L’energia di legame vale
me 2
e2
k e2
E=
ve − k − = −
= −13.6 eV
2
r
2 r
per r = 0.53 Å. L’elettrone ha velocità v = c/137.036 e quindi è non relativistico.
Esercizio 2: Rompere una bacchetta
Una bacchetta ha sezione di 1 cm2 . Che forza bisogna avere per romperla?
bSoluzione: Se la materia è costituita da atomi di dimensione Å legati dalla forza di Coulomb, uno deve
rompere n = ( cm/A)2 = 1016 legami e quindi serve una forza nFC ∼ 108 N. Tenendo conto che non distinguiamo
idrogendo da gesso da acciaio, la stima non è male; avrebbe potuto venire una cosa sbagliata di 100 ordini di
grandezza. Il legame vero è più debole; non occorre ionizzare gli atomi.
5
6
Capitolo 1. Campi e potenziali elettrici
Esercizio 3: Sistemi stabili?
Assemblare un sistema di cariche elettriche in equilibrio stabile.
bSoluzione: Presentiamo tre tentativi fallimentari: la cosa interessante è capire perchè non funzionano.
2) Usando solo due cariche non è possibile assemblare un sistema stabile.
3) Proviamo con tre cariche: due cariche q lungo l’asse x
P1 = `(−1, 0, 0),
P2 = `(1, 0, 0)
ed una q 0 in mezzo a P3 = (0, 0, 0) dove E = 0. Scegliendo q 0 = −q/4 si ha E = 0 anche sulle cariche q.
Quindi abbiamo realizzato un sistema in equilibrio, e rimane da vedere se si tratta di equilibrio stabile o
instabile. È facile vedere che q 0 è in equilibrio instabile: il campo elettrico per X = (x, y, z) ≈ 0 è
~
E(x,
y, z) =
2
~ − P~i
q X X
q
(−4x, 2y, 2z) + O(x, y, z)2 .
'
3/2
~ − P~i |
4π0 i=1 |X
4π0 L3
Come intuitivamente atteso l’equilibrio è instabile lungo x e stabile lungo y e z. È facile verificare che il
~ calcolato su di un cubetto attorno a 0 vale zero: −4 + 2 + 2 = 0.
flusso di E
Attorno alla carica 2 si ha
~
E(x,
y, z) '
x−L y z
q
(
,− ,− )
4π0 L3
4
8 8
~ calcolato su di un
chè è stabile lungo z ed y ma instabile lungo x. È facile verificare che il flusso di E
cubetto attorno a P2 vale zero: 41 − 18 − 18 = 0.
4) Con quattro cariche: tre q ai vertici di un triangolo equilatero di lato `:
1
P1 = `(0, √ , 0),
3
1 1
P2 = `( , √ , 0),
2 2 3
1 1
P3 = `( , √ , 0),
2 2 3
√
2 2
Ciascuna
(a distanza
√ risente una forza F = 2·(kq /` )( 3/2) diretta verso l’esterno. In mezzo al triangolo
√
d = `/ 3 dalle altre) il campo elettrico vale zero, quindi provo ad aggiungere una carica q 0 = −q/ 3 in
modo che le altre cariche risentano forza zero. L’equilibrio e’instabile. Ad esempio il campo elettrico per
X = (x, y, z) ≈ 0 è
~ =
E
√
~ − P~i
q X X
q 9 3 x y
'
(− , − , z) + O(x, y, z)2
~ − P~i |3/2
4π0 i |X
4π0 L3
2
2
che è instabile nel piano (x, y) e stabile lungo la direzione z. In pratica uno può calcolare le componenti
meno laboriose Ez ed Ey , ed ottenere Ex sapendo che il flusso vale zero: infatti si ha −1/2 − 1/2 + 1 = 0.
Il campo elettrico attorno alla carica 1 è
~ '
E
q
5 7
(− x, (y − P1y ), −z)
3
4π0 L
2 2
che è stabile lungo z e lungo x ma non lungo y. Di nuovo il flusso su di un cubetto attorno alla carica 1
vale zero.
Capitolo 1. Campi e potenziali elettrici
7
È importante notare che per altri potenziali V ∝ 1/rp con p 6= 1 sarebbe possibile costruire sistemi stabili.
Nel caso speciale di V ∝ 1/r il problema non ha soluzione, in quanto il fenomeno che abbiamo verificato è del
tutto generale: il flusso del campo elettrico generato dalle cariche esterne ad una superficie è zero (teorema di
Gauss). Lo si può verificare in generale espandendo il campo generato da una singola carica q — il campo di
tante cariche è la sovrapposizione dei campi delle singole cariche, ciascuna delle quali dà flusso zero. Mettendo
la carica q in (0, 0, 0), nella regione attorno a X = (r, 0, 0) + (x, y, z) si ha
~ =
E
~
q
x y z
q
X
'
(1 + (1 − p) , , ) + O(x2 , y 2 , z 2 )
p−1
p/2
~
4π0 |X|
4π0 r
r r r
che ha flusso zero solo per p = 2. Quindi il campo elettrico non può essere solo entrante o solo uscente, come
sarebbe necessario per avere una forza attrattiva in qualunque direzione.
Questo fallimento ha una conseguenze fisica importante: l’elettromagnetismo da solo non può spiegare la
stabilità della materia.
Esercizio 4: Campo elettrico di un filo
Si calcoli il campo e potenziale di un filo rettilineo infinito con densità lineare di carica λ.
bSoluzione: A distanza d dal filo
Z
+∞
Er =
dz
−∞
2kλ
kλr
=
,
r
(r2 + z 2 )3/2
Eθ = 0
Il potenziale elettrico ϕ consente di calcolare facilmente E = −∇ϕ di un filo di lunghezza finita 2`
Z
+`−z
p
+`−z
kλ
kQ
`2 r2 − 2z 2
`
4
0
02
2
dz √
'√
+ O(` )
= kλ ln z + z + r 1+0 −
d
6 (r2 + z 2 )2
r2 + z02
r2 + z 2
−`−z
0
ϕ(z, r) =
−`−z
dove Q = λ2`.
kλ
Ez = −∂z ϕ = p
r2
+ (z −
`)2
−p
kλ
r2
+ (z + `)2
,
Er = −∂r ϕ = ...
Espandendo in serie di Taylor per ` r, z, il primo termine corrisponde alla carica totale. Il termine successivo
è ‘di quadrupolo’ (il dipolo vale zero).
Per ` = ∞ viene ϕ = ∞. Avendo cariche all’infinito non è possibile mettere ϕ(∞) = 0. L’infinito è tutto
nella costante addittiva arbitraria in ϕ: fissando ϕ(r0 ) = 0 viene
Z
+∞
dz √
ϕ(r) = kλ
−∞
1
r
1
p
−
= −2kλ ln
= −2kλ ln r + costante
2
2
2
2
r
r +z
0
r0 + z
Se il mondo avesse 2 dimensioni invece di 3, questi sarebbero il campo ed il potenziale di Coulomb. Verificherebbero ancora ∇2 ϕ = 0. Siccome ϕ cresce per r → ∞, in 2 dimensioni non esisterebbero cariche
libere.
Esercizio 5: Campo elettrico di un piano
Calcolare il campo elettrico di un piano infinito con densità superficiale di carica σ
bSoluzione:
1) Calcolo diretto. Per motivi di simmetria E ha solo la componente Er ortogonale al piano. Passiamo
attraverso il campo generato da un disco circolare di raggio R.
Er (r) =
1
4π0
Z
R
2πρ dρ
0
σ
σr
r
R→∞ σ
=
(1 − √
) =
sgn r
20
20
(r2 + ρ2 )3/2
r 2 + R2
8
Capitolo 1. Campi e potenziali elettrici
Per r R in termini della carica totale Q = πR2 σ viene il risultato ovvio (espansione di multipolo)
Q
3R2
4
1
−
+
O(R
)
Er '
4π0 r2
4r2
Per R r si ottiene il piano cairico: Er (r) = σ/20 . Per motivi dimensionali E ∼ σr.
2) Teorema di Gauss: flusso = carica interna/0 . Prendendo un cilindretto schiacciato che attraversa una
qualunque superficie con densità di carica variabile σ
1
2
Φ = S(E⊥
− E⊥
) = σ/0
in qualunque punto
Nel caso del piano, aggiungendo considerazioni di simmetria, si riottiene il risultato precedente.
3) Il potenziale lungo l’asse di un disco uniformemente carico di raggio R vale
ϕ(r, 0) =
Per R r
ϕ(r) =
√
σ p 2
Q
R2
( R + r2 − r2 ) '
(1 − 2 + O(R4 ))
20
4π0 r
4r
σ
20
Z
0
∞
ρ
p
ρ2 + r 2
ρ
σ
−p
=
σ(r0 − r)
20
ρ2 + r02
Esercizio 6: Campo elettrico di una sfera
Calcolare il campo elettrico generato da una densità superficiale di carica σ = Q/4πR2 distribuita su di un
guscio sferico di raggio R.
bSoluzione: Il teorema di Gauss darebbe immediatamente il risultato, ma qui lo vogliamo ottenere tramite
un calcolo a testa bassa. Mettiamo la sfera nell’origine e calcoliamo E in (r, 0, 0). Per motivi di simmetria E
ha solo la componente radiale:
Z π
σ
r − R cos θ
0
r<R
Ex =
R
dθ
2πR
sin
θ
=
{z
} [(r − R cos θ)2 + R2 sin2 θ]3/2
Q/4π0 r2 r > R
4π0 0 |
dS
Esercizio 7: Energia di un guscio sferico
Si calcoli il lavoro necessario per comprimere un palloncino sferico contenente una carica Q uniformemente
distribuita da un raggio r1 ad r2 .
bSoluzione: A raggio r generico il campo elettrico è radiale
Q/4π0 r2 fuori
Er (r) =
0
dentro
Le cariche vorrebbero espandersi: in generale una densità di superficie risente una forza σE medio dove E medio =
(E 1 + E 2 )/2. In questo caso il campo interno è zero e σ = Q/4πr2 . Per comprimere da r1 ad r2 occorre
esercitare un lavoro
Z r2
Z r2
Q2
1
Er (r)
dr
Q2 1
2
dr =
( − )
L =
πr · σ(r)
=
2
2
8π
r
8π
r
r
0 r1
0
1
2
r1
2
R
R
E
Sostituendo σ == 0 Er e S dr = dV ottengo anche L = Sσ E2r dr = 0 2r dV . In generale il campo elettrico
contiene una densità di energia u = 0 E 2 /2. Le dimensioni sono giuste. In questo caso la verifica è semplice
perchè mano a mano che si contrae varia solo il campo nella zona di contrazione
Z
Z r2
E2
Q2 1
1
∆U = u dV = 0
4πr2
=
( − )
2
8π
r
r
0
1
2
r1
• • • • • • • • • • • • • • • • • •
Capitolo 1. Campi e potenziali elettrici
9
Come ulteriore verifica calcoliamo anche l’energia totale per ri = R
Z
Z
1
? 0
U=
ρϕ dV =
E 2 dV
2
2
Inserendo nella prima espressione ϕ = Q/(4π0 R) e Sσ = Q, si ottiene subito
U=
1
Q2
Sσϕ =
2
8π0 R
Inserendo nella seconda E = Q/(4πr2 0 ) viene lo stesso risultato:
Z
Z ∞
0 ∞
Q2
Q2
dr
2 2
U=
4πr E dr =
=
.
2 R
8π0 R r2
8π0 R
Esercizio 8: Modelli dell’atomo
Nel modello di Thomson per l’atomo di idrogeno, la carica positiva e è distribuita uniformemente in una sfera
di raggio a0 . L’elettrone di carica −e è considerato puntiforme e si muove all’interno della sfera.
a) Calcolare il campo elettrico ed il potenziale generati dalla carica positiva e la posizione d’equilibrio per
l’elettrone (assunto in uno stato di momento angolare nullo).
b) Determinare l’energia di ionizzazione UI (ovvero l’energia necessaria ad estrarre l’elettrone dall’atomo).
Trovare il valore di a0 consistente col valore sperimentale UI = 2.18 × 10−18 Joule.
c) Determinare il periodo di oscillazione dell’elettrone intorno alla posizione d’equilibrio e confrontarlo col
valore sperimentale T = 3.04 × 10−16 sec−1 .
d) Si calcoli il momento di dipolo elettrico p indotto nell’atomo da un campo esterno E 0 , la polarizzabilità
α dell’atomo e la costante dielettrica dell’idrogeno allo stato solido (cioè nello stato in cui tutti gli atomi
sono adiacenti fra loro a formare un reticolo).
bSoluzione: L’energia di ionizzazione si può anche riscrivere come 13.6 eV dove eV = e × mN/C = 1.60 10−19
Joule.
a) Il campo è radiale e si ha equilibrio stabile in r = 0:
2
(
(
e
r
3
r
r < a0
− 4πe0 a0 2a
2 − 2
4π0 a30
0
Er (r) =
ϕ(r) =
e 1
e 1
r > a0
+ 4π
4π0 r 2
0 r
r < a0
r > a0
b) UI = −eϕ(0) = 3e2 /8π0 a0
c) Dall’equazione del moto
r̈ = −
e e r
= −ω 2 r
me 4π0 a30
si ha ω 2 = e2 /4π0 me a30 e quindi T = 2π/ω = 7.9 10−16 s.
d) La nuova posizione d’equilibrio req è data da E(req ) = E 0 da cui r eq = 4π0 a30 E 0 /e; p = −er eq = αE 0
dove α = 4π0 a30 ; la densità di atomi è n = 1/(2a0 )3 ; quindi = 1 + nα/0 = 1 + π/2.
Nel modello di Rutheford l’energia di ionizzazione vale
UI = −
me v 2
1
e2
− ϕ(a0 ) = (− + 1)
2
2
4π0 a0
e la frequenza di rotazione
ω2 =
v2
F/me
e2
=
=
2
a0
a0
4π0 me a30
Modello
ω2
2
Rutherford e /4π0 me a30
Thomson e2 /4π0 me a30
Energia di ionizzazione
e2 /8π0 a0 = 8.5 eV
3e2 /8π0 a0 = 26 eV
Per fissare ω 2 serve a = 8.4 10−11 m in entrambi i casi; dopodichè entrambi i modelli non azzeccano UI (la
frequenza è data dalla differenza di energia tra i livelli quantistici).
10
Capitolo 1. Campi e potenziali elettrici
Esercizio 9: Energia di una sfera
Calcolare l’energia potenziale di una sfera di raggio R contenente una carica Q distribuita uniformemente.
bSoluzione:
1) Integrando la densità di energia
Z R
2
Z
Z ∞
0
0
Qr 2
Q
1
Q
Q2
2
2
2
2
U=
E dV =
4πr (
) +
) =
4πr (
+
2
2
4π0 R3
4π0 r2
4π0 10R 2R
0
R
2) Calcolo doppiamente sbagliato. Densità di carica: ρ = Q/V = 3Q/4πR3 . Carica dentro una sferetta di
raggio r < R: qin (r) = Q(r/R)3 , quindi ϕ = qin (r)/4π0 r (soddisfa ϕ(∞) = 0 e continutità ad r = R)
Z
Z R
qin (r)
1 3 Q2
=
U = ρϕ dV =
4πr2 ρ dr
R
4π0 5 R
0 | {z } |4π
{z0 }
dq
ϕ
R
20 ) Calcolo giusto. Il potenziale dentro la sfera vale ϕ(r) = Er dr = cte − 21 qin (r)/(4π0 r). Imponendo
continuità ad r = R
Q(3R2 − r2 )/8π0 R2 per r < R
ϕ(r) =
Q/4π0 r
per r > R
Integrando
U=
1
2
Z
ρϕ dV =
1
4π0
R
Z
0
3 Q2
3Q2 r2 (3R2 − r2 )
=
6
4R
5 R
E
ϕ
I plot delle funzioni Ee e ϕ giuste sono:
r
r
3) Il calcolo sbagliato al punto 2) non dà il risultato giusto per caso. Partendo dai principi primi, costruisco
la sfera aggiungendo mano a mano carica dq facendo crescere il suo raggio r da 0 a R a densità costante
ρ. La ϕ usata in 2) non è il potenziale della sfera finale, ma l’energia necessaria per portare cariche da ∞
a r nel modo appena descritto.
dU =
1 q(r)dq
40 r
q(r) = ρ
4πr3
r
Q( )3
3
R
dq = ρ 4πr2 dr
quindi
dU =
4πρ2 4
r dr
30
Z
U=
dU =
4πρ2 R5
1 3Q2
=
150
4π0 5R
• • • • • • • • • • • • • • • • • •
Le stesse cose valgono per la gravità. I fisici credevano che la terra fosse più giovane di 30 Myr, siccome
l’energia del sole E ∼ GM 2 /R emessa con potenza d2 K K = 8.53 · 1011 MeV cm−2 s−1 dura solo per
(GM 2 /R)/(d2 Ksun ) ∼ 30 Myr. I biologi (Darwin) sostenevano che almeno 300 Myr erano necessari per l’erosione
e l’evoluzione delle specie. Teologi (like Lightfoot) sostenevano che la terra era stata creata il 23 ottobre −4004,
alle nove del mattino. I fisici successivamente scoprirono l’energia nucleare.
Capitolo 1. Campi e potenziali elettrici
11
Esercizio 10: Raggio classico dell’elettrone
Approssimando l’elettrone come una sferetta di raggio R, calcolare il valore di R tale che U = me c2 .
bSoluzione: Approssimare le particelle come puntiformi è un limite singolare. Ad esempio: (1) Quando si
calcola la forza su di un elettrone non si include nel campo elettrico quello infinito generato dall’elettrone stesso.
(2) L’energia elettromagnetica U diverge per R → 0.
Nel secolo scorso ci sono stati tentativi di migliorare questa situazione proponendo teorie dell’elettrone in
cui l’elettrone veniva approssimato con una palletta di raggio R finito. L’energia elettromagnetica U allora è
finita: U = cqe2 /4π0 R dove c dipende da quale distribuzione di carica viene assunta. c = 3/5 per una densità
ρ uniforme; c = 1/2 per una densità superficiale σ uniforme. Assumiamo c = 1.
L’energia elettromagnetica contribuisce alla massa dell’elettrone, secondo m = U/c2 . Assumendo che tutta
la massa sia di origine elettromagnetica si determina
R = re =
qe2
= 2.82 10−15 m
me 0 c2
chiamato ‘raggio classico dell’elettrone’, sebbene non abbia niente a che vedere con le dimensioni di un elettrone
(solo una piccola parte dell’energia dell’elettrone è di origine elettromagnetica). Numericamente, re è simile alla
dimensione di un protone: si tratta di un accidente fuorviante.
Una aspettazione qualitativa più corretta è U < me c2 (altrimenti U eccederebbe la massa dell’elettrone):
questa implica R > re . Oggi si sa quello che succede: a scale R ∼ 1000re iniziano a farsi sentire gli effetti del
positrone, una particella identica all’elettrone ma con carica positiva. Ripetendo il calcolo di U in teorie di
campo quantistiche relativistiche si trova che il contributo elettromagnetico alla massa dell’elettrone esiste ma
è piccolo, circa 1/100 della massa totale.
Esercizio 11: Masse dei nuclei
Si può approssimare un nucleo come una sfera a densità costante, contenente Z = A/2 protoni e circa A/2
neutroni con raggio R = A1/3 rN con rN = 1.2 10−15 m. Calcolare l’energia elettromagnetica e discutere la
stabilità dei nuclei.
bSoluzione: Aggiungiamo all’energia eletromagnetica un termine che descrive l’energia di legame media fra
nuclei contigui e la differenza di massa mn − mp 1
mnucleo = Zmp + (A − Z)mn +
U
,
c2
U=
3 (Ze)2
+ Elegame A.
5 4π0 R
Esistono nuclei stabili fino a Z ∼ 100: imponendo dU/dZ|Z∼100 = 0 si trova l’energia di legame per nucleone:
2/3
100
e2
Elegame = −
≈ −10 MeV
4
4π0 rN
A grandi Z la repulsione Coulombiana diventa l’effetto principale ed impedisce di formare nuclei grossi. Minimizzando U/Z si trova che il nucleo che ha la maggior energia di legame per nucleone ha Z ∼ 20, ed infatti è il
ferro (Z = 26).
Esercizio 12: Differenza di massa protone-neutrone
Stimare la differenza di massa protone-neutrone approssimandoli come 3 quarks fermi ai vertici di un triangolo
equilatero come p = uud e n = udd (qu = 2e/3, qd = −e/3).
bSoluzione: L’energia elettrostatica vale
Ep = (qu2 + 2qu qd )
1I
e2
= 0,
4π0 r
En = (qd2 + 2qu qd )
e2
e2 1
=−
4π0 r
4π0 3r
nuclei sul bordo sono meno legati. Trascuriamo tale effetto, legato alla ‘tensione superficiale’, che è importante solo per nuclei
piccoli.
12
Capitolo 1. Campi e potenziali elettrici
Convertendo energia in massa tramite E = mc2 l’elettromagnetismo tende a rendere il protone (carico) più
pesante del neutrone (neutro): mp − mn ∼ e3 /12π0 r = MeV · 0.5 10−15 m/r. L’ordine di grandezza è giusto
(mn − mp = 1.3 MeV, rN = 1.2 10−15 m) ma il segno è sbagliato. Se fosse mp > mn non esisterebbero atomi.
Tenere conto che i quark ruotano con v ∼ c (e quindi c’e’anche una energia magnetica) non cambia il segno. Il
neutrone n ' ddu pesa più del protone p ' uud perchè i quark d hanno massa maggiore dei quark u:
mn − mp = md − mu + O(e2 /4π0 r)
{z
}
| {z } | {z } |
1.3 MeV
3 MeV
−1.7 MeV
Il fatto che il neutrone sia poco più pesante del protone è essenziale per avere una chimica complessa. Il neutrone
libero decade: se fosse mp < mn il protone (e quindi l’atomo di idrogeno) decadrebbe. Se mn − mp fosse un
poco più grande supererebbe l’energia di legame e non esisterebbero nuclei.2
Esercizio 13: Nucleo che si spezza
Si può approssimare un nucleo come una sfera a densità costante. Un nucleo si spezza in due nuclei di carica
Q0 = Q/2 e raggio R0 = R/21/3 . Di quanto cambia l’energia elettromagentica?
bSoluzione: Ricordando che U = 3Q2 /5R/4π0 , l’energia elettromagnetica liberata da un nucleo che si spezza
è
3 Q2 Q2
0
∆Eem = Eem − 2Eem
=
1 − 2−2/3 = 0.22
5 4π0 R
4π0 R
Per Q = 100e e R = rN N 1/3 ∼ 10−14 m viene ∆U ∼ 250 MeV: l’ordine di grandezza è giusto. L’effetto
elettromagnetico è proporzionale a Z 2 , ed a grande Z diventa più importante dell’effetto dovuto alla differenza
di massa protone/neutrone ed alla loro energia di legame, proprozionale a Z.
Un kg di uranio contiene circa 4 moli: quindi fissionandolo si libera un’energia 4NA · 250 MeV = NA GeV ≈
1014 J ≈ 20kton ≈ (kg/1000)c2 (dove kton = 4.2 1012 J è un unità di energia usata per bombe e corrisponde
all’energia rilasciata nell’esplosione di 1000 tonnellate di TNT).
Esercizio 14: Energia di due cariche
Per due cariche q1 e q2 ad x = ±1.
bSoluzione: Usando il potenziale si ottiene subito U = q1 q2 /4π0 d. Integrando E 2 i calcoli sono troppo difficili,
ma interessanti perchè occorre rinormalizzare
Z
Z
Z
1
1
q1 q2
r2 − 1
2
2
2
(E 1 + E 2 ) − E 1 − E 2 =
E1 · E2 =
U=
8πk
4πk
4πk
(r2 − 2x + 1)3/2 (r2 + 2x + 1)3/2
L’integrale in r⊥ dà 4π/x2 se x2 > 1 e 0 altrimenti. L’integrale in x è banale
U =k
q1 q2
d
La cosa qualitativa importante è che due cariche opposte hanno E = 0 nel mezzo: quindi si attraggono in quanto
avvicinandosi minimizzano U . La stessa cosa accade per due fili: siccome il campo B è rotazionale invece che
radiale si ha B = 0 nel mezzo con correnti uguali, che quindi si attraggono.
Esercizio 15: Forza su cariche superficiali
Dimostrare che una generica densità di carica superficiale σ induce una variazione E⊥1 − E⊥2 = σ/0 e subisce
una forza F = σ(E 1 + E 2 )/2
2 L’energia
di legame nucleare è indirettamente dovuta alle masse dei quark che controllano la massa dei π (che sono le forze di
van der Waals nucleari) ed è di ordine mu + md .
Capitolo 1. Campi e potenziali elettrici
13
bSoluzione: Applicando il teorema di Gauss ad un cilindretto schiacciato infinitesimo che attraversa perpendicolarmente la superficie si ottiene la variazione di E⊥ . E= è continuo.
Per calcolare la forza occorre vedere spessore zero come limite di uno spessore finito. Chiamando z l’asse ⊥
alla superficie si ha dEz /dz = ρ/0 (relazione che segue dalla dimostrazione Rprecedente, ed è un caso particolare
di ∇ · E = ρ/0 ). Intergrando in dz si riottiene Ez2 − Ez1 = σ/0 con σ = ρdz. La densità di forza vale
Z
Z
2
dFz
E 2 − Ez1
Ez2 + Ez1
dEz
pz =
= Ez ρ dz = 0 Ez
dz = 0 z2
=σ
= σEzext
dS
dz
2
2
La media geometrica viene per un motivo fisico semplice: la forza deve essere generata solo dal campo elettrico
‘esterno’, non da quello generato dalla σ stessa.
Esercizio 16: Scattering debole
Una carica q urta su di un sistema di cariche totali Q. Calcolare il piccolo angolo di deflessione θ assumendo
simmetria cilindrica (o che sia possibile osservare soltanto uno scattering medio, come capita in esperimenti che
utilizzano un fascio di molte particelle) e che q e ciascuna delle cariche in Q vengano perturbate poco dall’urto.
bSoluzione:
∆p⊥
θ(b) =
=
p=
R
F⊥ dv/x
1 q
=
mv
mv 2 2πb
Z
E⊥ dS =
qQ(r < b)
4π0 bK
K≡
m 2
v
2
Quindi misurando θ(b) si fa una tomografia della distribuzione di cariche.
Esercizio 17: Scattering Rutherford
Una particella α (di massa me m mN , carica +2e, ed energia E = 4 MeV) viene fatta collidere su atomi
contenti nuclei di carica Ze e massa mN . Calcolare l’angolo di deflessione in funzione del parametro d’impatto
b A e la sezione d’urto.
bSoluzione: Siccome m me ed E (energia di ionizzazione) gli Z elettroni hanno effetto trascurabile, se
la particella α entra nella zona b A dove gli elettroni non schermano il nucleo. Conta solo il nucleo, che
approssimativamente rimane fermo. Facciamo il conto in 3 modi.
1. Adattando le note formule per le orbite dei pianeti. Tenendo conto che in questo caso la forza F = α/r2
(α = 2kZe2 ) è repulsiva
r=−
`
,
1 + e cos θ
e2 = 1 +
2EL2
m2 v 4 b2
=1+
2
α m
α2
Si ha r > 0 per cos θ < −1/e e cioè in un range ∆θ dato da cos ∆θ/2 = 1/e. L’angolo di deflessione è
definito come θd = π − ∆θ e vale quindi
sin
θd
∆θ
1
= cos
=
2
2
e
o anche
tan
θd
1
α
=√
.
=
2
bmv 2
e2 − 1
Il parametro d’impatto che produce una data deflessione è
b=
kZe2
θd
tan−1
E
2
dove
kZe2
Z MeV
= 10−13 m
.
E
70 E
14
Capitolo 1. Campi e potenziali elettrici
2. Procedendo in modo diretto, senza usare tecniche sofisticate. Usando coordinate polari (r, θ) e mettendo
l’asse x lungo la linea di simmetria, l’equazione del moto è
mv̇x =
α
α
cos θ = m θ̇ cos θ
r2
L
dove L = mr2 θ̇ è il momento angolare rispetto al nucleo, che è una costante del moto, uguale a L = mv0 b.
Siccome F ∝ 1/r2 , è sparita la dipendenza da r. Diventa banale integrare ottenendo vx = (α/L) sin θ e
quindi v = vx / cos θ = (α/L) tan θ. Ad r = ∞ tan θ0 = mv02 b/α. L’angolo di deflessione è θd = π − 2θ0 .
3. Approssimazione perturbativa.
dp⊥
dx dp⊥
1
1 α b
α/v0
=
' F⊥ =
= 2
dt
dt dx
v0
v0 r2 r
(x + b2 )3/2
Quindi
∆p⊥
αb
θd =
=
p=
mv02
Z
+∞
−∞
(x2
dx
2α
=
2
3/2
bmv02
+b )
p
che Rè corretta per θd 1. La primitiva è ∝ 1/ 1 + b2 /x2 . Usando il teorema di Gauss verrebbe
2πb E⊥ = 4π dove E⊥ = b/(x2 + b2 )3/2 .
Il parametro d’impatto in un singolo urto non è misurabile sperimentalmente. È invece nota la distribuzione
di probabilità dei parametri d’impatto in un numero n 1 di urti. Secondo la meccanica quantistica questa
cosa è vera non solo in pratica, ma anche in linea di principio. Il ‘punto d’incontro’ convenzionale fra teoria ed
esperimento è la sezione d’urto per collisioni su di un singolo nucleo.
σ=
numero di particelle deflesse
flusso di particelle incidenti
σ caratterizza gli effetti misurabili prodotti da un nucleo; spetta allo sperimentale tenere conto che i nuclei sono
tanti (e che ci possono essere scattering multipli). σ la dimensione di un’area, e dice quanto è grosso un nucleo,
quando viene visto tramite interazioni elettromagnetiche. Ad esempio, la sezione d’urto totale per eventi con
angolo di deflessione maggiore di un qualunque valore θ è
2
kZe2
1
σ(θd > θ) = πb2 (θ) = π
E
tan2 θ/2
E.g. σ(θd > π/2) dice quante particelle rimbalzano all’indietro. La probabilità che una particella rimbalzi
indietro, quando viene inviata perpendicolarmente su di un piano di atomi a distanza d ∼ Å fra di loro, è
σ/d2 ∼ 10−6 . Se viene mandata su di una targhetta lunga ` composta da atomi con densità n, è σn` < 1.
La sezione d’urto totale è σ(θd > 0) = ∞: a differenza di altre forze, l’elettromagnetismo è una interazione
a lungo raggio3 , che deflette tutte le particelle (anche quelle con b → ∞). In realtà quando b >
∼ Å gli elettroni
schermano il campo elettrico del nucleo, e poi ci sono altri nuclei.
Di solito si preferisce descrivere lo scattering usando la sezione d’urto differenziale
b db 1 kZe2 2
numero di particelle deflesse in dΩ
dσ dϕ
1
=
dσ =
=
d cos θ = flusso di particelle incidenti
d cos θ 2π
sin θ dθ 4
E
sin4 θ/2
Avendo scritto l’angolo solido in coordinate polari dΩ = dϕ d cos θ e tenendo conto che lo scattering non dipende
dall’angolo polare ϕ
Esercizio 18: Esplosione Coulombiana
Una nuvola sferica di raggio R e carica totale Q è costituita da N particelle di carica q = Q/N e massa m,
inizialmente (t = 0) distribuite con densità uniforme.
a) Calcolare l’energia potenziale di una carica posta a distanza r dal centro della nuvola.
particella massiva genererebbe una forza ‘di Yukawa’ F ∝ αe−r/r0 /r 2 (dove m ∝ 1/r0 ). Essa darebbe, in approssimazione
perturbativa θd ∼ θdCoulomb e−b/r0 e quindi σ(θ > 0) ∝ r02 .
3 Una
Capitolo 1. Campi e potenziali elettrici
15
Figura 1.1: Linee di campo (linee continue) e superfici equipotenziali (linee tratteggiate) generate da una ‘sfera
polarizzata’.
Per effetto della repulsione coulombiana la nuvola inizia ad espandersi radialmente, mantenendo la simmetria
sferica. Nel corso del moto radiale le particelle non si scavalcano (cioè se inizialmente due strati di particelle si
trovano alle distanze r1 (0) e r2 (0) > r1 (0) dal centro, ad ogni istante successivo r2 (t) > r1 (t).)
b) Sia r = r(t) la posizione al tempo t delle particelle che a t = 0 sono a distanza r0 = r(0) < R dal centro.
Mostrare che l’equazione del moto per r = r(t) è
m
d2 r
qQ r0 3
=
dt2
4π0 r2 R
(∗)
c) Si dica a che distanza dal centro si trovano inizialmente le particelle che acquistano la massima energia
cinetica durante l’espansione, e si dia il valore di tale energia massima.
d) Si mostri che per ogni strato di particelle si muove secondo la legge oraria r(t) = r0 λ(t) dove λ(t) non
dipende da r0 e che di conseguenza la densità di carica rimane uniforme durante l’espansione della nuvola.
bSoluzione:
a)
(
V (r) =
Q
r2
4π0 (− 2R3
Q 1
4π0 r
+
3
2R )
per r < R
per r > R
b) Poichè le particelle non si scavalcano, la carica contenuta entro una sfera di raggio r(t) rimane costante.
c) L’energia potenziale corrispondente all’equazione del moto (*) è Ur0 = (Q/4π0 )(r0 /R)3 /r. L’energia
cinetica massima viene acquistata a distanza infinita ed è uguale a Ur0 , che è massima per r0 = R.
d) Inserendo l’ansatz nell’equazione del moto (*) si trova d2 λ/dt2 = qQ/4π0 λ2 R2 nella quale r0 non compare
più. Quindi la nuvola si dilata in modo omogeneo.
Esercizio 19: Sfera polarizzata
Calcolare il campo elettrico generato da una sfera di raggio R con carica superficiale σ(θ) = σ cos θ.
16
Capitolo 1. Campi e potenziali elettrici
bSoluzione: Conviene usare il principio di sovrapposizione e vederla come la la distribuzione di carica generata
da tanti piccoli dipoli allineati. Separando le cariche positive da quelle negative, lo si può anche vedere come
sovrapposizione di due sfere con densità uniformi ρ e −ρ con i centri a distanza d tale che dρ = σ. La
corrispondenza diventa esatta per d → 0 (e quindi ρ → ∞).
Come calcolato precedentemente una singola sfera genera al suo interno un campo elettrico E = rρ/30 .
Quindi due sfere di carica ±ρ sovrapposte a distanza d generano al loro interno E = −ρd/30 = −P /30 dove
P ≡ ρd viene chiamata ‘densità di polarizzazione’.
All’esterno della sfera si ha il campo di un dipolo p = Qd = V P , dove V è il volume della sfera.
Il potenziale in tutto lo spazio, in coordinate sferiche è
3 2
σ cos θ
R /r per r > R
p cos θ/4π0 r2 per r > R
ϕ(r, θ) =
=
r
per r < R
Er cos θ
per r < R
30
La seconda espressione permette di verificare che ϕ è continuo a r = R.
Esercizio 20: Cilindro polarizzato
Calcolare il campo elettrico generato da una cilindro polarizzato trasversalmente con carica superficiale σ(θ) =
σ cos θ.
bSoluzione: Si procede in modo analogo alla sfera, sovrapponendo due cilindri con densità uniformi ρ e −ρ.
Esercizio 21: Formule di base sui dipoli
Due cariche ±q a distanza d formano un dipolo p = qd.
bSoluzione: Ricavo le formule di base sui dipoli.
• Campo elettrico generato da un dipolo. Espandendo r i = r ± d/2 (d punta verso la carica positiva)
ϕ
E
q
p·r
1
1
1
( − ) ' −kqd · ∇ = k 3
4π0 r1
r2
r
r
1 3(p · r)r
p
1 2(p · r)r r × (r × p)
= −∇ϕ =
−
+
=
4π0
r5
r3
4π0
r5
r5
=
In coordinate polari ϕ = p cos θ/4π0 r2 e quindi
∂ϕ
2kp cos θ
1 ∂ϕ
kp sin θ
=
,
Eθ = −
=
,
3
∂r
r
r ∂θ
r3
diverso da zero per ogni θ. A grande distanza E ∝ 1/r3 .
Er = −
Eϕ = 0
E=
kp p
1 + 3 cos2 θ
r3
• Forza sentita da un dipolo in un campo elettrico esterno:
F = q(d · ∇)E = (p · ∇) = −∇U
dove
U = −p · E = −pE cos θ
L’energia è minima quando p si allinea ad E.
Il momento delle forze vale M = p × E, M = −∂θ U = qdE sin θ.
Negli esercizi successivi discutiamo alcune sottigliezze nell’uso di queste formule.
Esercizio 22: Paradosso sui dipoli I
Un dipolo p, obbligato ad orientarsi lungo l’asse z è libero di muoversi lungo l’asse x in un campo elettrico
esterno Ez = αx. Calcolare la forza sul dipolo.
bSoluzione: Secondo le formule precedenti U = −pαx, quindi F = −∇U = pαx.
Tuttavia è ovvio che la forza totale su due cariche ±q poste una sopra l’altra lungo l’asse z è zero.
La formula non si applica perchè il campo elettrico proposto non è irrotazionale, come si vede da un circuitino
o da ∇ × E = −αŷ.4
4 Volendo
eccetto 0
complicare le cose si può rifare lo stesso esercizio con un campo elettrico Eθ = 1/r, che ha rotore zero in tutti i punti
Capitolo 1. Campi e potenziali elettrici
17
Esercizio 23: Paradosso sui dipoli II
Calcolare l’energia di un dipolo prodotto da un campo elettrico esterno
bSoluzione: Se p = αE il lavoro necessario per portare il dipolo da zero a p vale
Z
Z
p·E ?
L = F · ds = E · dp =
= −∆Utot
2
L’energia di interazione fra il dipolo ed il campo elettrico esterno vale U = −p · E. Affinchè Utot = U + U 0 sia
giusta ci deve essere un’altra energia potenziale U 0 = + 12 p · E, dovuta al fatto che l’esistenza stessa del dipolo
è dovuta alla forza esterna.
Come verifica del risultato generale consideriamo il sistema particolare più semplice possibile: due cariche
±q a distanza x tenute assieme da una forza elastica. Il valore della costante k dovrebbe essere irrilevante. Il
dipolo vale p = qx = qE/k. L’energia di legame vale
U0 =
k 2
pE
x =
2
2
Questo sistema è realizzato fisicamentente dall’atomo di Thomson, nel quale avevamo visto che α = 4π0 a30 .
L’energia U 0 di interazione fra elettrone e protone è di tipo ‘elastico’ e vale
U 0 (r) − U 0 (0) = −e[ϕ(r) − ϕ(0)] =
pE
e2 2
r =
2α
2
(verifico che ϕ è giusto usando il laplaciano in coordinate polari: ∇2 ϕ = −3e/α = −ρ/0 ).
Esercizio 24: Paradosso sui dipoli III
Calcolare la forza fra due dipoli p e p0 a distanza x, orientati parallalelamente alla loro separazione. Come
cambia la risposta se p0 è indotto da p come p0 = αx?
bSoluzione: Abbiamo due formule generali che danno la forza fra dipoli. Vediamo come applicarle.
1. La formula generale è F = (p · ∇)E = p∂x E dove
1 3(p0 · r)r
p0
1 p0
E=
−
x̂
=
5
3
4π0
r
r
2π0 x3
r = (x, y, z)
è il campo elettrico generato da p0 . Quindi la forza è ‘radiale’ e vale Fx = −3pp0 /2π0 x4 .
È ovvio che la risposta non cambia se p0 è indotto.
2. Una formula meno generale ma più semplice è F = −∇U con
U = −p · E = −
1 pp0
2π0 x3
Nel caso di p0 costante si riottiene il risultato precedente. Nel caso di dipolo indotto, p0 = αx, viene
un risultato che differisce di un fattore 2/3: per utilizzare correttamente questa formula meno generale
occorre prima calcolare il ∇ come se p0 fosse costante e poi inserire il valore del dipolo indotto p0 (x).
Esercizio 25: Paradosso sui dipoli IV
Due dipoli sono orientati rispettivamente lungo gli assi z ed x. Verificare che le forze sono uguali ed opposte
(ma non radiali). Verificare che i momenti non lo sono.
bSoluzione: Il momento totale è zero se calcolato rispetto ad un polo fisso. Una situazione analoga più banale:
un dipolo nel campo elettrico di una carica. Il dipolo sente un momento delle forze (che tende ad allinearlo con
il campo elettrico), ma anche una forza.
18
Capitolo 1. Campi e potenziali elettrici
Esercizio 26: Allineamento di dipoli elettrici
Come si dispongono due dipoli a distanza r fissata, liberi di ruotare su loro stessi?
bSoluzione: L’energia fra due dipoli pi = qi di a distanza r fissata (senza vincoli non esistono configurazioni
di equilibrio stabile)
U=
k
[p · p − 3(p1 · r̂)(p2 · r̂)] ∝ cos(θ1 − θ2 ) − 3 cos θ1 cos θ2
R3 1 2
che è minima a θ1 = θ2 = 0 (più configurazioni simmetriche), cioè per dipoli stesi nella stessa direzione. Tenerli
verticali e contrapposti richiede energia maggiore.
Un dipolo può essere la molecola H2 O (l’O attira gli elettroni più di H), oppure l’atomo di idrogeno in un
campo esterno.
Esercizio 27: Coordinate polari
Calcolare gradiente, divergenza, rotore e ∇2 in coordinate cilindriche e polari ed in un generico sistema di
coordinate ortogonali.
bSoluzione: Il ∇ è sia un operatore differenziale che un vettore. Questo significa e.g. che ∇ · (f E) = ... e che
∇ × ∇f = 0, (∇f ) × (∇g) 6= 0, ∇ · (∇ × E) = 0. In linea di principio per passare a coordinate polari si procede
come per altri vettori, rispettando le proprietà dell’operatore derivata. In pratica si fa molto prima usando i
teoremi in cui compaiono gradienti, rotori e divergenze: i teoremi del gradiente (linee), Stokes (superifci), Gauss
(volumi) sono casi particolari di integrali di forme asimmetriche a n indici
Z
I
∂ ∧ A(n) ∧ dx(n+1) =
A(n) ∧ dx(n)
X
e cioè (in 3 dimensioni):
Z
I
∇f · dx =
f = ∆f
L
Gradiente
∂L=P
∂X
Z
I
Z
(∇ × E) · n dS =
E · dx
S
∂S=L
I
(∇ · E)dV =
V
E · n dS
∂V =S
In un generico sistema di coordinate ortogonali xi si ha
df = f (x + dx) − f (x) =
X
dxi
i
∂f
≡ dx · ∇f
∂xi
−1
x̂i ∂i . E.g. in coordinate polari grr = 1, gθθ = r e gϕϕ = r sin θ in quanto
Siccome dx = gii dxi x̂i allora ∇ = gii
2
dx = dr r̂ + r dθ θ̂ + r sin θdϕ ϕ̂ e ds = dr2 + r2 dθ2 + r2 sin2 θdϕ2
∇ = x̂
∂
∂
∂
∂
1 ∂
1
∂
+ ŷ
+ ẑ
= r̂
+ θ̂
+ ϕ̂
∂x
∂y
∂z
∂r
r ∂θ
r sin θ ∂ϕ
Divergenza Per calcolare la divergenza di un generico vettore E usiamo il teorema di Gauss
Z
Z
(∇ · E)dV = E · dS
applicato ad un volumetto elementare di lati dxi . La differenza dei flussi sui lati lungo x1 vale
(E1 g22 g33 )+ dx2 dx3 − (E1 g22 g33 )− dx2 dx3 = dx1 dx2 dx3
∂
dV
∂
(E1 g22 g33 ) =
(E1 g22 g33 )
∂x1
g11 g22 g33 ∂x1
Quindi
∇·E =
1
∂
∂
∂
(E1 g22 g33 ) +
(E2 g11 g33 ) +
(E3 g11 g22 )
g11 g22 g33 ∂x1
∂x2
∂x3
Capitolo 1. Campi e potenziali elettrici
19
In coordinate polari la divergenza di un vettore con solo componente radiale vale
∇·E =
1
∂
1 ∂
Er r2 sin θ = 2 r2 Er
r2 sin θ ∂r
r ∂r
Riotteniamola procedendo in un altro modo, senza usare il teorema di Gauss: Applicato ad un vettore radiale
E = r̂Er vale
∇·E =
∂Er
θ̂ ∂r̂
ϕ̂
∂r̂
∂Er
1 ∂
+ Er ( ·
+
·
)=
+ (1 + 1)Er = 2 r2 Er
∂r
r ∂θ
r sin θ ∂ϕ
∂r
r ∂r
Su di una funzione V che dipende solo da r
∇2 V = (∇ · ∇)V =
1 ∂ 2 ∂
1 ∂2
rV
r
V
=
r2 ∂r ∂r
r ∂r2
In d dimensioni (d = 3 coordinate polari, d = 2 coordinate cilindriche, etc)
∇·E =
1
∂ d−1
r
Er
rd−1 ∂r
∇2 V =
d − 1 ∂V
∂2V
+
∂r2
r ∂r
Rotore Utilizzando in modo analogo il teorema di Stokes si ottiene un’espressione esplicita per il rotore. Il
∇2 è banale


g11 x̂1 g22 x̂2 g33 x̂3
1
g22 g33
1
2


∇×E =
det
∂1
∂1 V + · · ·
∂1
∂2
∂3
,
∇ V =
g11 g22 g33
g11 g22 g33
g11
g11 E1 g22 E2 g33 E3
Ad esempio in coordinate polari
∂ϕ2 V
1
r
∂ 2 (rV ) ∂θ (sin θ∂θ V )
r sin θ
2
∇ V = 2
∂θ V ) + ∂ϕ
∂ϕ V = r
+
+
∂r (r sin θ ∂r V ) + ∂θ (
r sin θ
r
r sin θ
r
r2 sin θ
r2 sin2 θ
2
ed in coordinate cilindriche
1
∂r (r∂r V ) ∂θ2 V
1
∂r (r∂r V ) + ∂θ ( ∂θ V ) + ∂z (r∂z V ) =
+ 2 + ∂z2 V
∇ V =
r
r
r
r
2
Esercizio 28: Laplaciano
2 p
Calcolare ∇ r in d dimensioni spaziali.
bSoluzione: In 1 dimensione r2 = x2 . In 2 dimensioni r2 = x2 + y 2 . In 3 dimensioni r2 = x2 + y 2 + z 2 . In
generale
∂x rp = px rp−2 ,
∂x2 rp = p rp−2 + p(p − 2)x2 rp−4 ,
∇2 rp = p[d + p − 2]rp−2
Il caso d = 3 corrisponde a coordinate polari; il caso d = 2 a coordinate cilindriche.
Il potenziale generato da una carica in d dimensioni è la soluzione singolare a r = 0 di ∇2 ϕ = 0, e cioè per
ϕ ∝ r2−d . Quindi
d=1
d=2
d=3
d=4
d=5
ϕ ∝ r ϕ ∝ ln r ϕ ∝ r−1 ϕ ∝ r−2 ϕ ∝ r−3
E ∝ r0 E ∝ r−1 E ∝ r−2 E ∝ r−3 E ∝ r−4
(il campo generato da una carica puntiforme in d = 1 e d = 2 corrispondono rispettivamente al campo di un
filo e di un piano in d = 3).
La cosa fondamentale non è ϕ ∼ 1/r ma la conservazione del flusso, e cioè E ∝ 1/S o ∇2 ϕ = 0.
Solo d = 3 dà fisica interessante. Per d < 3 non esistono cariche libere (ϕ cresce con r), per d > 3 l’energia
cinetica di rotazione (potenziale effettivo V = L2 /2mr2 ) non basta ad impedire che si spiaccichino ad r = 0.
20
Capitolo 1. Campi e potenziali elettrici
Esercizio 29: Potenziale di Yukawa
2
2
Risolvere ∇ ϕ − λ ϕ = ρ = 0
bSoluzione: La soluzione a simmetria sferica ϕ(r) è
1
(rϕ)00 = λ2 ϕ
r
:
rϕ = e−λr
Per r λ è come l’elettromagnetismo, per r λ la forza va a zero esponenzialmente. Lo Z ha λ ∼ 10−16 cm.
La gravità potrebbe avere λ ∼ 1010 anni luce.
Esercizio 30: Atomo di idrogeno quantistico
Risolvere l’equazione di Schroedinger
bSoluzione: Secondo Schroedinger uno deve: scrivere l’energia H, rimpiazzare p → −ih̄∇, risolvere Hψ = Eψ,
e |ψ|2 è la probabilità. Per un elettrone in un atomo di idrogeno
H=
qe2
h̄2 2 e2
p2
−
→−
∇ −
2m 4π0 r
2m
r
avendo usato e2 = qe2 /4π0 . Quindi, usando u = rψ
−
h̄2
e2
(rψ)00 = (E + )(rψ)
2m
r
risolto da ψ ∝ e−r/a0 ed E = −e2 /2a0 dove a0 ≡ h̄2 /me2 . Quindi secondo la meccanica quantistica la carica
dell’elettrone si distribuisce come ρ(r) = −e e−2r/a0 /πa30 .
Capitolo 2
Conduttori
Le cariche elettriche dentro un conduttore sono libere di redistribuirsi; e finchè E 6= 0 continuano a spostarsi.
Quindi si riaggiustano (dissipando energia termicamente) fino a raggiungere la condizione di equilibrio stabile:
E = 0 dentro il conduttore ed E= = 0 lungo la sua superficie (cioè ϕ = costante nel conduttore). Quindi il
campo elettrico subito fuori da un conduttore vale E⊥ = σ/0 . La pressione sentita da un conduttore è σE⊥ /2.
Esercizio 31: Piano conduttore
Studiare una carica q puntiforme posta a distanza d da un piano conduttore infinito posto a potenziale zero.
bSoluzione: Scelgo le coordinate in modo che il piano è a x = 0 e la carica a (x, y, z) = (d, 0, 0). Usando il
metodo delle immagini si trova il potenziale

1
1
 q
−
per x > 0
ϕ(x, y, z) =
4π0 |x − d| |x + d|

0
per x < 0
La densità superficiale di carica indotta sul piano è
σ(x, y) = 0 E⊥ = −0
1+1
dq
∂ϕ
=−
2
2
∂x
4π (x + y + d2 )3/2
R
e la carica totale è σ dx dy = −q, come si può verificare o facendo esplicitamente l’integrale, o applicando il
teorema di Gauss ad una superficie chiusa ottenuta ‘chiudendo’ il piano a r = −∞ (l’unico contributo al flusso
è sul piano a r ∼ d).
La carica q sente una forza attrattiva Fx = −kq 2 /(2d)2 . Il piano sente una forza opposta, come si può
verificare integrando dF/dS = σE⊥ /2 (non dimenticando il fattore 2: E⊥ /2 è la media del campo elettrico
‘subito fuori’ e ‘subito dentro’ il conduttore)
Z
kq 2
E⊥
dx dy =
F = σ
2
(2d)2
0
Per portare
R ∞ la carica2 da x = d ad x = ∞ (o più in generale, ad un punto x = d ) occorre compiere un lavoro
L = d F dx = kq /4d, uguale a metà della variazione dell’energia potenziale fra carica e carica-immagine
V (∞) − V (d) = kq 2 /2d, in quanto non serve lavoro per spostare la carica immagine.
Se il piano conduttore non è posto a ϕ = 0 ma è isolato non cambia niente. Infatti, se è finito ma grosso
(dimensioni D d) ed ha carica totale zero, la carica q induce una carica −q nella zona ‘vicina’, e quindi una
carica +q concentrata ai bordi lontani (che non si vede se D → ∞).
Esercizio 32: Lastra conduttrice
Si calcoli il campo elettrico in presenza di una carica q situata a distanza d da una lastra conduttrice piana di
spessore finito s.
21
22
Capitolo 2. Conduttori
Figura 2.1: Tentativi di usare il metodo delle immagini.
bSoluzione: È immediato verificare che

 come prima dal lato dove c’è q, i.e. x > 0
0
dentro il conduttore, i.e. −s < x < 0
ϕ(x, y, z) =

0
dal lato opposto, i.e. x < −s
è una soluzione, e quindi è la soluzione. Se d > s la carica immagine −q viene fuori dalla lastra, ma questo è
irrilevante in quanto ϕ è scritto in termini di −q solo sul lato dove c’è q. Dal lato opposto dove non c’e’ la carica
E = 0. Se il conduttore dal lato opposto non fosse piano ma avesse una forma artistica, rimarrebbe sempre
E = 0.
Questo fenomeno è più generale: un conduttore scherma da altre cariche lo spazio che circonda. Se un
conduttore contiene un buco vuoto, allora dentro E = 0. Infatti la ovvia unica soluzione dell’equazione di
Poisson con condizioni al bordo ϕ(bordo) = ϕ0 è ϕ(buco) = ϕ0 . Questo accade perche’ F ∝ 1/rn con n = 2, e
consente di verificare sperimentalmente quanto n è veramente vicino a 2.
Per schermare un campo elettrico (stazionario) non serve racchiudere tutto con un conduttore: una griglia
conduttrice a maglie piccole basta a fare un buon lavoro (per lo stesso motivo discusso in un esercizio analogo
a pag. 73).
Esercizio 33: Metodo delle immagini
Trovare e studiare altri casi simili.
bSoluzione: Usando la linearità è immediato studiate altri casi: se ci sono due cariche q1 e q2 basta sommare
le soluzioni. Se c’e’ un dipolo, si aggiunge un dipolo immagine. Se si vuole studiare un filo carico sospeso sopra
il terreno, si considera un filo immagine sottoterra.
Con una carica posta vicino a semipiani che si intersecano ad un dato angolo, il metodo funziona solo per
angoli speciali. Usualmente riflettendo si trova che servirebbe mettere cariche nella zona vuota (vedere fig. 2.1)
per cui si ottiene una soluzione per 2 o più cariche messe in posti speciali
Il metodo funziona per angolo di 90◦ , e servono 3 cariche immagini (fig. 2.1a). È interessante studiare in
che modo il lavoro necessario a spostare la carica è legato all’energia potenziale fra q e cariche immagini. Per
semplicitè mettiamo la carica lungo l’asse di simmetria a distanza d dai piani: risente una forza attrattiva diretta
lungo l’asse con modulo
√
Z ∞
1
(−4 + 2)kq 2
2 −1
2
+ √
,
L = √ F ds =
F = kq √
8d
2 (2d)2
(2 2d)2
2d
Il lavoro è uguale all’energia potenziale della sola carica ‘vera’, e quindi ad 1/4 dell’energia potenziale di tutte
le cariche (‘vera’ ed ‘immagini’)
k X q1 qi
kq 2 −2
1
k X qi qj
V (1) =
=
+ √
= L,
V (1) + V (2) + V (3) + V (q) =
= 4V.
2
r1i
2 2d
2
rij
2 2d
i6=1
i6=j
Capitolo 2. Conduttori
23
Esercizio 34: Piano carico fra 2 piani conduttori
Due piani conduttori paralleli a distanza ` sono tenuti allo stesso potenziale. Una carica q, distribuita uniformemente lungo un piano, viene messa a distanze δ e δ 0 = ` − δ dai piani. Calcolare le cariche totali indotte q e
q0 .
bSoluzione: Ovviamente q + q 0 = −q. I campi elettrici sono costanti. Siccome i due conduttori sono allo stesso
potenziale, Eδ = E 0 (s − δ). Le cariche indotte sono legate ai campi da E = σ/0 e quindi
σ
E
`−δ
q
= 0 = 0 =
q0
σ
E
δ
Quindi q = −q(1 − δ/`) e q 0 = −qδ/`.
Esercizio 35: Carica fra 2 piani conduttori
Due piani conduttori paralleli a distanza ` sono tenuti allo stesso potenziale. Una carica puntiforme q viene
messa a distanze δ e δ 0 = ` − δ dai piani. Calcolare le cariche totali indotte q e q 0 .
bSoluzione: Si potrebbe usare una serie infinita di cariche immagini ±q situate a x = 2n` ± δ, ma il conto
sarebbe troppo difficile.
RP
PR
(la carica indotta non dipende dalla distanza, ma
6=
). Visto che il problema chiede solo la carica totale
indotta, convene usare un altro trucco. Immaginiamo che q invece di essere puntiforme sia distribuita lungo un
piano parallelo ai due conduttori, come nell’esercizio precedente. La carica totale indotta resta la stessa. Con
lo stesso trucco si potrebbe anche calcolare la forza sentita dalla carica.
Esercizio 36: 1 lastre conduttrice carica
Una lastra di superficie S ha carica totale q. Calcolare i campi elettrici indotti.
bSoluzione: Fisicamente uno si aspetta che la carica q si divida equamente fra le due superfici, generando un
campo elettrico esterno E = (q/2)/S0 ortogonale alla lastra ed uguale sui due lati.
Questo accade non perchè alle cariche piace disporsi simmetrimecamente, ma perchè questa è la configurazione di minima energia, come discusso nel meno semplice problema successivo.
Esercizio 37: 2 lastre conduttrici cariche
Due lastre di superficie S hanno cariche totali q e q 0 . Calcolare i campi elettrici indotti.
bSoluzione: Il problema consiste nel trovare come le cariche si ripartiscono fra le superfici destra e sinistra
delle lastre. In generale le cariche sulle 4 superfici (da sinistra a destra) possono essere
q − Q,
Q
−Q
q0 + Q
dove Q è incognito. I campi elettrici sono
Esinistra =
q−Q
,
0 S
Emezzo =
q
,
0 S
Edestra =
q0 + Q
0 S
L’energia totale è proporzionale all’integrale di E 2 , dominato dal grande spazio a sinistra ed a destra. Quindi
2
2
le cariche minimizzano Esinistra
+ Edestra
. Questo accade nella configurazione simmetrica, Q = (q − q 0 )/2:
Esinistra = Edestra =
q + q0
,
20 S
Emezzo =
q − q0
20 S
24
Capitolo 2. Conduttori
Esercizio 38: Capacitatore cilindrico
Un cavo coassiale è fatto di un filo conduttore interno di diametro d circondato da un guscio metallico di diametro
D. Calcolare la capacità. Sapendo che l’aria può sostenere Emax = 3MV/ m (rigidità dell’aria) trovare quale
valore di d/D consente di avere la massima differenza di potenziale, e quale d/D consente di immagazzinare la
massima energia.
bSoluzione: Usando il teorema di Gauss possiamo immediatamente calcolare il campo elettrico nello spazio
vuoto fra le due armature: 2πr · Er = λ/0 da cui
Er =
∂ϕ
2kλ
=−
r
∂r
:
ϕ = −2kλσ ln r.
Quindi la differenza di potenziale vale V = ∆ϕ = 2kλ ln(D/d) e la capacità per unità di lunghezza vale
2π0 /ln D/d. Imponendo che il massimo campo elettrico E(r = d/2) = 4kλ/d sia uguale a Emax si trova che la
massima differenza di potenziale vale
d D
V = Emax ln
2
d
Per esempio V = 3.45 kV se d = 1mm e D = 1cm. Fissato D, V è massimizzato scegliendo d = D/e.
L’energia immagazzinata in una lunghezza L vale
U=
CV 2
d
D
2
= Lπ0 Emax
( )2 ln
2
2
d
ricalcolabile anche come
Z
U=
0
d
0 E 2
2
dV = L2π( )2 Emax
2
2
2
Z
D/2
d/2
r dr
.
r2
√
Fissato D, l’energia immagazzinata è massimizzata scegliendo d = D/ e.
Esercizio 39: Capacitatore di dimensioni variabili
Si raddoppia la distanza fra i piatti di un capacitatore di capacità C. Quanto lavoro meccanico occorre fare se
(a) le cariche sui piatti sono tenute costanti? (b) una batteria mantiene costante la differenza di potenziale V ?
bSoluzione: Ricordo che Q = CV . Per due piatti conduttori di area A a piccola distanza d
V = Ed = σd/0 = Q · d/A0
cioè
C = 0 A/d
Raddoppiare d dimezza C. All’inizio l’energia vale
U = Ad
0 E 2
A0 V 2
CV 2
Q2
=
=
=
.
2
d 2
2
2C
(a) Alla fine U 0 = Q2 /2C 0 = 2U quindi L = U − U 0 = −U . Infatti il campo elettrico rimane uguale,
ma occupa un volume doppio. I due piatti si attraggono, quindi occorre una forza F = L /(−d) per
allontanarli.
In generale, quando uno modifica un capacitatore variando la capacità di dC tenendo la carica Q costante
F ds = dL = −dU = −
Q2 1
Q2
V2
d =
dC =
dC
2
2 C
2C
2
(b) Alla fine U 0 = C 0 V 2 /2 = U/2 quindi Ltotale = U − U 0 = U/2 > 0. Questo sembra suggerire che i due
piatti si respingano, mentre invece uno si aspetta che si attraggano esattamente come nel caso precedente
(in quanto contengono cariche di segno opposto). Il punto è che Ltotale è il lavoro totale, somma di
due contributi: un contributo meccanico (legato alla forza necessaria per spostare le armature), ed un
lavoro ricevuto dalla batteria mano a mano che le cariche sulle armature diminuiscono. La carica finale
vale Q0 = C 0 V = Q/2. Una carica ∆Q = −Q/2 viene spinta dentro la batteria, che riceve un lavoro
Lbatteria = −QV /2 = U . Lmeccanico = Ltotale − Lbatteria = −U/2 < 0.
Capitolo 2. Conduttori
25
In generale la batteria riceve un carica −dQ e quindi un lavoro Lbatteria = −V dQ = −V 2 dC. L’energia
nel capacitatore varia di V 2 dC/2. Quindi il lavoro meccanico vale
F ds = dLmeccanico = dLtotale − dLbatteria =
V2
dC
2
come nel caso (a).
Quindi in generale la forza è legata alla variazione della capacità C da F = (V 2 /2)(dC/ds), e tende ad aumentare
la capacità. In questo esercizio abbiamo solo ottenuto un risultato atteso in modo complicato. Il prossimo è più
interessante.
Ad esempio se inserisco una barra conduttrice in un condensatore...
Esercizio 40: Conduttore in capacitatore
Un conduttore di spessore d viene parzialmente inserito in un capacitatore quadrato di spessore D e lunghezza
L D mantenuto ad una differenza di potenziale V . Calcolare la forza sentita dal conduttore mobile.
bSoluzione: Il sistema è disegnato in fig. 3.1a. Possiamo vederlo come una capacità C0 = 0 L(L − x)/D in
parallelo con una capacità C1 = 0 Lx/(D − d) (costituita da 2 capacità in serie). Inserire un conduttore è un
po’come ridurre la distanza fra i piatti: per questo la capacità aumenta. Il valore preciso è
0 L2
xd
C = C0 + C1 =
1+
D
L(D − d)
Come visto in precedenza la forza vale
F =
V 2 dC
V 2 dL0
=
.
2 dx
2 D(D − d)
Fisicamente la zona dove agisce la forza è la punta del conduttore: sebbene in quella zona non sappiamo fare i
calcoli, sappiamo calcolare la forza totale. Determiniamo ora il segno dell’effetto. Non mi pare possibile capirlo
in modo intuitivo, ma solo affidandosi al formalismo. Nel caso banale dell’esercizio precedente la forza tende ad
attrarre i piatti, cioè ad aumentare la capacità. Quindi in questo esercizio il conduttore viene attratto dentro il
condensatore, perchè questo aumenta la capacità.
Esercizio 41: Sfera conduttrice a terra
Una carica q è situata a distanza R dal centro di una sfera conduttrice a potenziale zero di raggio r.
bSoluzione: Serve una carica immagine q 0 = −q r/R messa come in figura fig. 2.2, situata a distanza r2 /R dal
centro della sfera.
Un giorno qualcuno notò che il potenziale generato da due cariche ϕ = kq1 /r1 + kq2 /r2 vale zero su di una
sfera. Infatti ϕ = 0 a r1 /r2 = q1 /q2 i.e. r12 q22 + r22 q12 = 0 che è l’equazione della sfera. La sfera è ‘il luogo dei
punti per i quali le distanze fra 2 punti sono in rapporto fisso’.
Torniamo al problema, che proviamo a risolvere usando un sistema di coordinate con origine nel centro della
sfera ed aggiungendo una carica immagine q2 = q 0 a distanza r2 dal centro. Per fissare il tutto basta imporre
ϕ = 0 nei 2 punti del conduttore lungo l’asse:
q1
q2
+
= 0,
R − r r − r2
q1
q2
+
=0
R + r r + r2
sono risolte da r2 = r2 /R e q2 = −q1 r/R. La sfera conduttrice ha carica totale q 0 . Se avesse carica diversa (e.g.
zero) o potenziale diverso, basterebbe aggiungere una ulteriore carica immagine q 00 nel centro.
Usando coordinate polari ρ, θ il potenziale vale
1
r 1
1
r
1
ϕ(ρ, θ) = kq( −
) = kq p
− p
r1
R r2
ρ2 + R2 − 2Rρ cos θ R ρ2 + (r2 /R)2 − 2(r2 /R)ρ cos θ
26
Capitolo 2. Conduttori
Figura 2.2: Fig. 2.2a,b,c: linee di campo in presenza di una carica q a distanze varie da una sfera conduttrice
a terra. Fig. 2.2d: linee di campo per una sfera conduttrice isolata in campo elettrico esterno.
La densità di carica superficiale vale
σ(θ) = 0 Er (ρ = r, θ) = −0
q(R2 − r2 )
∂ϕ =
2
∂ρ ρ=r
4πr(R + r2 − 2rR cos θ)3/2
La carica totale indotta sulla sfera è q 0 , che è diversa da q. La forza attrattiva fra la sfera e la carica q vale
F =k
q1 q2
rR
= −kq 2 2
2
r12
(R − r2 )2
e decresce come 1/R3 per R r. Il lavoro necessario a spostare la carica q rispetto alla sfera da distanza R a
distanza R0 vale L = qq 0 /8π0 (1/R0 − 1/R).
Esercizio 42: Sfera conduttrice isolata
Come l’esercizio precedente, ma la sfera è isolata
bSoluzione: Occorre aggiungere una ulteriore carica immagine q 00 = −q 0 in modo che la ‘carica immagine
totale’ sia zero. Per fare in modo che la sfera rimanga a potenziale costante occorre mettere −q 0 nel centro della
sfera. Per R r la forza fra sfera e carica q decresce come 1/R4 . Il lavoro necessario a spostare la carica q
rispetto alla sfera è uguale a L = − 12 [∆Vqq0 + ∆Vqq00 ] cioè senza includere la variazione di energia potenziale
Vq0 q00 fra le due cariche immagini.
Esercizio 43: Sfera conduttrice in E costante
Una sfera conduttrice isolata di raggio r viene messa in un campo elettrico E0 esterno costante.
bSoluzione: Si può trovare la soluzione in diversi modi, sviluppando ulterioremente esercizi precedenti.
• Partendo dal problema precedente, posso generare un campo elettrico costante usando una carica q a
distanza R dalla sfera nel limite q, R → ∞ tenendo costante E0 = q/4π0 R2 . In questo limite la carica
immagine q 0 = −qr0 /R diverge e si avvicina al centro della sfera, dove si trova la seconda carica immagine
q 00 = −q 0 , ma le due cariche immagine generano un dipolo finito p = −q 0 d = 4π0 E0 r3 . Riassumendo:
fuori dalla sfera
E = E 0 + (campo generato da un dipolo p nel centro della sfera).
Verifichiamo che il potenziale ϕ(ρ, θ) è costante sulla superficie della sfera a ρ = r
r3
p
−
E
ρ
cos θ = ( 2 − ρ)E0 cos θ
ϕ(ρ, θ) =
0
2
4π0 ρ
ρ
Capitolo 2. Conduttori
27
Figura 2.3: (a) Condensatori in serie. (b) Condensatori in serie.
La densità superficiale di carica vale
∂ϕ = 30 E0 cos θ
σ(θ) = −0
∂ρ ρ=r
• Abbiamo quindi ritrovato la situazione studiata a pagina 15: una sfera con carica superficiale σ(θ) =
σ0 cos θ. Avevamo trovato che genera al suo interno un campo elettrico costante E = σ0 /30 , che per
σ0 = 30 E0 è esattamente opposto al campo esterno E0 . In questo modo dentro la sfera si ha E = 0.
Avevamo anche trovato che all’esterno genera il campo di un dipolo p = V σ0 , che per σ0 = 30 E0 vale
p = 4π0 E0 r3 in accordo con il risultato precedente.
Le linee di campo sono disegnate in figura 3.1d.
Esercizio 44: Tetraedro conduttore
4 triangoli equilateri conduttori, mantenuti a potenziali ϕ1,2,3,4 vengono disposti in modo da formare la superficie
di un tetraedro. Quale è il potenziale nel centro?
bSoluzione: In generale deve essere una combinazione lineare dei 4 contributi. Infatti se so risolvere il caso
con solo 1 acceso (ϕ1 6= 0 e ϕ2,3,4 = 0), e poi so risolvere
P il caso con solo il 2 acceso, sommando le due soluzioni
ho risolto anche il caso con 1 e 2 accesi. Quindi ϕ = i ci ϕi .
P
Poi, per motivi di simmetria, la risposta deve essere simmetrica in 1, 2, 3, 4. Quindi ϕ = c ϕi .
Per finire c = 1/4 (cioè ϕ = (ϕ1 + ϕ2 + ϕ3 + ϕ4 )/4) in quanto nel caso ϕ1 = ϕ2 = ϕ3 = ϕ4 il potenziale deve
avere il valore comune costante, siccome le 4 facce formano un tetraedro chiuso.
Esercizio 45: Condensatore sferico
Un condensatore è costituito da una sfere concentriche di raggi r1 ed r2 . Calcolare la capacità e discutere il
limite r2 → ∞.
bSoluzione:
C=
Q
4π0
= −1
∆V
r1 − r2−1
Se r2 r1 il valore di r2 conta poco e si può pensare un’unica sfera come un condensatore di capacità C = 4π0 r1
avente l’altro ‘’piatto’ ad infinito.
Esercizio 46: Condensatori in serie
Due condensatori di capacità C1 e C2 con cariche Q1 e Q2 vengono connessi come in fig. 2.3a. Come si
redistribuiscono le cariche?
bSoluzione: La corrente flusice lungo la resistenza, dissipando energia, fino a che i due condensatori hanno
equali differenze di potenziali. Imponendo
V =
Q01
Q0
= 2
C1
C2
si trova Q0i = Ci (Q1 + Q2 )/(C1 + C2 ).
e
Q1 + Q2 = Q01 + Q02
28
Capitolo 2. Conduttori
Esercizio 47: Effetto delle punte
Due sfere conduttrici cariche di raggi r ed R lontane sono connesse da un filo. Mostrare che il campo elettrico
attorno alla sfera piccola è più grosso che attorno alla sfera grossa.
bSoluzione: Le cariche in un conduttore carico si respingono, e quindi cercano di andare il più possibile lontane
le une dalle altre, generando una forte concentrazione di cariche sulle punte. La sfera piccola schematizza una
punta e consente di fare un calcolo esplicito.
Le cariche q e Q sulle due sfere si determinano imponendo che i potenziali sulle superfici delle due sfere siano
uguali:
q
Q
=
R
r
Questo corrisponde a quanto visto nell’esercizio precedente: Qi ∝ Ci ∝ ri . Quindi il campo elettrico è grosso
attorno alla sfera piccola
E(r)
R
q/r2
= .
=
E(R)
Q/R2
r
Il massimo campo elettrico che l’aria asciutta può sopportare è è qualche MV/m (con campi elettrici più forti
rendono l’aria conduttrice dando luogo a scariche). Mettendo delle punte su di un parafulmine ci si assicura
che una nuvola carica eletricamente si scarichi su di esse.
Esercizio 48: Sfera conduttrice bucata
Una sfera conduttrice scarica contiene, al suo interno ma non al suo centro, un buco con dentro una carica q.
Calcolare il campo elettrico generato.
bSoluzione: Nonostante l’assenza di simmetria sferica, il campo elettrico esterno è uguale a quello generato
da una carica q al centro del conduttore. Infatti, l’unica soluzione dell’equazione di Poisson costante sulla sfera
è ∝ 1/r. Non esiste una soluzione semplice per il campo elettrico nel buco.
Esercizio 49: Carica dentro sfera
(Dal compito del 16/1/2004). Una carica puntiforme q è posta all’interno di un guscio conduttore sferico di
raggio interno R e raggio esterno R0 , a distanza d dal centro. Il guscio conduttore è posto a terra. Calcolare
a) Il potenziale ed il campo elettrico in tutto lo spazio.
b) La forza sulla carica q.
c) Mostrare che la carica totale indotta sulla sfera è pari a −q.
d) Come cambia la risposta a) se il guscio conduttore è isolato?
bSoluzione:
a) È noto che due cariche q e q 0 = −qR/d a distanze dd0 = R2 dal centro di una sfera producono potenziale
zero sulla sfera. Questo è il sistema di cariche immagini che ci serve per calcolare E nella zona interna. Il
fatto che il conduttore abbia spessore finito non complica il problema. Dentro il conduttore e nella zona
esterna E = 0.
b) La forza è attrattiva e vale F = qq 0 /4π0 (d − d0 )2 .
c) Siccome fuori E = 0, la carica totale (q + carica indotta) è zero.
d) La carica totale ora è q. Sulla superficie interna si dispone una carica totale −q distribuita in modo da
schermare, a r > R l’effetto della carica puntiforme. Sulla superfcie esterna si dispone uniformemente una
carica totale q, generando un campo radiale E = q/4π0 r2 . Dentro E rimane come prima.
Capitolo 3
Dielettrici
La densità di polarizzazione indotta da un campo elettrico esterno P = 0 χE induce una densità di cariche di
polarizzazione ∇ · P = ρpol (e sui bordi una densità superficiale σpol = ∆P⊥ ). Separando la carica totale in
ρtot = ρfree + ρpol e definendo D = P + 0 E ≡ E = κP /(κ − 1) il campo D soddisfa a ∇ · D = ρfree . Se
χ è costante un dielettrico è descritto dalle stesse equazioni del vuoto con 0 → = κ0 dove κ = 1 + χ. Se
χ varia bruscamente le condizioni di raccordo su bordi senza cariche libere sono: ∆E= = 0 e ∆D⊥ = 0 (cioè
1 E⊥1 = 2 E⊥2 ).
Esercizio 50: 2 dielettrici in condensatore piano
Calcolare la capacità di un condesatore piano ottenuto mettendo due diversi dielettrici fra due piatti conduttori
come mostrato in figura.
bSoluzione:
σ1
σ
ε1
ε2
σ2
ε1
ε2
1) Chiamo ` lo spessore totale, diviso in `1 ed `2 . Nel primo caso conviene usare il campo D perchè è costante:
D = σ = 1 E1 = 2 E2
Quindi la differenza di potenziale V e la capacità C = Q/V valgono
V = E1 `1 + E2 `2 =
Q `1
`2
( + )
S 1
2
cioè
1
1
1
=
+
C
C1
C2
dove
Ci =
i S
.
`i
Questa geometria corrisponde ad avere due condensatori in serie.
2) Nel secondo caso conviene usare il campo elettrico perchè uguale nelle due zone, visto che ∆E= = 0
lungo il bordo e che la differenza di potenziale è la stessa nelle due zone. Quindi la densità di carica
totale è la stessa nelle due zone; ma ci interessa la capacità che è definita in termini della carica libera
come C = Qfree /V . Chiamiamo σ1 e σ2 al diversa densità di carica libera nelle due zone. Siccome hanno
uguale area la densità media di carica media è σ = (σ1 + σ2 )/2. Esse determinano il campo elettrico come
Ei = σi /i . Imponendo E1 = E2 si ottiene
σi = σ
i
1 + 2
e quindi
V = Ei ` =
Q/S
1 + 2
C = (1 + 2 )
Questa geometria corrisponde ad avere due condensatori in parallelo.
29
S
= C1 + C2 .
`
30
Capitolo 3. Dielettrici
Esercizio 51: N dielettrici in condensatore piano
Ripetere l’esercizio precedente mettendo N dielettrici di egual spessore `/N e costanti dielettriche i = 1 +
(2 − 1 )i/N . Ottenere il risultato nel limite N → ∞.
bSoluzione:
1) Quando sono in serie
N
N
X 1
X ∆z
1
=
=
→
C
Ci
S
i=1
i=1 i
Z
`
0
dz
` ln(1 /2 )
=
(z)S
S 1 − 2
dove (`) = 1 + (2 − 1 )z/`.
2) Quando sono in parallelo, scrivendo S = L2
N
X
Z L
N
X
(x)dx L
S 1 + 2
i L ∆x
C=
Ci =
→
=
`
`
`
2
0
i=1
i=1
dove (x) = 1 + (2 − 1 )x/L.
Esercizio 52: Condensatore in acqua
Un condensatore cilindrico di lunghezza L e diametri esterno ed interno D e d, mantenuto ad una differenza di
potenziale V , viene immerso verticalmente in una bacinella d’acqua, di densità ρ. Calcolare di quanto si innalza
il livello dell’acqua dentro il condensatore rispetto al livello esterno.
bSoluzione: Abbiamo visto che inserendo un dielettrico fra le armature di un condensatore se ne aumenta la
capacità, e che quindi le forze elettriche Fel = dUel /dz tendono a far salire l’acqua dentro il condensatore. Al
contrario la forza gravitazionale Fgrav tende a farla scendere. L’acqua salirà fino ad un livello z tale che queste
due forze si bilanciano. Calcoliamole.
• Se l’acqua entra nel condensatore di un tratto z la capacità vale C(z) = 2π(z + (L − z)0 )/ ln(D/d) =
C(0) + 2π0 zχ/ ln(D/d). avendo definito = (1 + χ)0 . La forza elettrica non dipende da z:
Fel = +
V 2 dC
πV 2 0 χ
=
.
2 dz
ln D/d
• La forza gravitazionale cresce con z:
Fgrav = −m(z)g = −π
D 2 − d2
zρ · g
4
Imponendo Fgrav + Fel = 0 si trova
z=
(D2
4V 2 0 χ
− d2 )gρ ln(D/d)
cioè misurando z si può ricavare χ. Numericamente viene z ∼ mm per D ∼ mm e V ∼ kV.
Esercizio 53: Carica davanti a semipiano dielettrico
Lo spazio è riempito da due semipiani dielettrici aventi a sinistra costante dielettrica 2 , ed a destra 1 . Una
carica si trova a destra. Trovare i campi elettrici.
bSoluzione: Provo: in 1 il campo di vuoto generato da q e da una q 0 immagine. In 2 il campo di vuoto di una
q 00 al posto di q.
E=1 =
(q + q 0 ) cos θ
,
r2
E=2 =
q 00 cos θ
,
r2
E⊥1 =
(−q + q 0 ) sin θ
,
r2
E⊥2 = −
q 00 sin θ
r2
Capitolo 3. Dielettrici
31
L
D
P
d
x
Figura 3.1: Esercizi su forze.
Le condizioni di raccordo sono
q 00 = q + q 0 ,
1 (q − q 0 ) = 2 q 00
:
q0 = q
1 − 2
,
1 + 2
q 00 = q
21
1 + 2
Per 2 → ∞ si ritrova il conduttore. La carica q vale qvera /κ1 .
Se invece 1 2 E⊥1 è piccolo, come intrappolare una carica.
Esercizio 54: Dielettrico in condensatore
Un dielettrico di costante dielettrica relativa κ e spessore d viene parzialmente inserito in un condensatore
quadrato di spessore D e lunghezza L D mantenuto ad una differenza di potenziale V . Calcolare la forza
sentita dal dielettrico.
bSoluzione: Il sistema è disegnato in fig. 3.1a. Possiamo vederlo come una capacità C0 = 0 L(L − x)/D in
parallelo con una capacità C1 , costituita da 2 capacità in serie: C10 = 0 Lx/(D − d) e C100 = κ0 Lx/d. Quindi
la capacità totale vale
C = C0 + C1 = C0 +
1
0 L(L − x)
L0 κ
=
+x
1/C10 + 1/C100
D
d − dκ + Dκ
Come visto in precedenza la forza vale
F =
V 2 dL0 (κ − 1)
V 2 dC
=
2 dx
2 D(d − dκ + Dκ)
Il dielettrico viene attratto dentro il condensatore. Per κ → ∞ si ritrova il ‘conduttore in condensatore’ studiato
a pagina 25, Abbiamo potuto trascurare gli effetti ai bordi e sulla punta del dielettrico mobile sebbene sia lı̀ che
si esercita la forza.
32
Capitolo 3. Dielettrici
Esercizio 55: Forza di conduttore su dielettrico
Un dielettrico di base quadrata a ed altezza h a ha una polarizzazione uniforme P come in figura 3.1b. Viene
appoggiato su di un piano conduttore. Calcolare la forza risentita.
bSoluzione: La polarizzazione genera una densità di carica uniforme +σ sulla cima, e −σ sulla base. (Per
determinare il segno basta ricordare ρpol = −∇ · P = −∂z Pz ). Lo si può risolvere usando un dielettrico
immagine. Ma la forza dominante è quella generata dalle cariche −σ nella base che inducono una carica +σ sul
conduttore, ed un campo elettrico E = σ/0 e quindi una forza attrattiva F = Eσa2 /2 = P 2 a2 /20 . Stimiamo
il contributo delle cariche sul tetto approssimandole come una carica puntiforme q = σa2 . Introducendo una
carica immagine −q essa risente una forza q 2 /h2 4π0 . Quindi la forza totale vale
P 2 a2
a2
F ≈
1+
20
2πh2
Il contributo delle cariche in cima è trascurabile.
Esercizio 56: Dielettrico in campo esterno
Un dielettrico con costante dielettrica κ è immerso in un campo elettrico esterno E ext . Calcolare il campo
elettrico all’interno del dielettrico assumendo che esso abbia forma a) lunga e sottile; b) corta e larga; c) sferica.
bSoluzione:
a) Se il dielettrico è lungo e sottile, la condizione al bordo dominante è ∆E= = 0, e quindi Ein = Eext .
b) Se il dielettrico è corto e largo, la condizione al bordo dominante è ∆D⊥ = 0: dentro il dielettrico il campo
elettrico vale Ein = Din /kappa = Dout /κ = Eout /κ.
c) Se il dielettrico è sferico, verrà una cosa intermedia ma il conto è più compicato. Le equazioni da risolvere
sono, dentro il dielettrico:
E tot = E ext + E pol ,
P = 0 χE tot
(3.1)
cioè la polarizzazione è proporzionale al campo elettrico totale, che comprende un contributo generato
dalla polarizzazione.
Assumiamo che χ ≡ κ − 1 1: in tal caso la polarizzazione è piccola E ext e E tot saranno quasi uguali, e
quindi in prima approssimazione la polarizzazione vale P ' 0 χE ext , ed è quindi costante. Come discusso
a pagina 15) un P costante genera una densità di cariche superficiali σ = P cos θ e quindi, all’interno della
sfera, un campo elettrico E pol = −P /30 uniforme. Il campo elettrico totale vale
E tot = E ext + E pol ' (1 −
χ
)E ext
3
c)0 In generale E ext e E tot differiscono in modo significativo. Proviamo a vedere se una polarizzazione
P uniforme risolve il problema impostato in eq. (3.1). La polarizzazione genera un campo elettrico
E pol = −P /30 = −χE ext /3 uniforme: quindi, dentro la sfera si produce un campo uniforme
E tot = E ext + E pol = E ext −
χ
E tot
3
:
E tot =
E ext
3E ext
=
1 + χ/3
2+κ
minore del campo esterno. Per χ 1 si ritrova l’approssimazione del punto precedente. Per χ 1 il
dielettrico diventa come un conduttore.
Il campo elettrico totale esterno alla sfera è quello esterno più quello di un dipolo: il risultato è disegnato in
figura 3.2.
Capitolo 3. Dielettrici
33
Figura 3.2: Linee di campo (continue) ed equipotenziali (tratteggiate) per una sfera dielettrica in un campo
elettrico esterno costante. Le tre figure correspondono a costanti dielettriche κ = {1, 3, 30}.
Esercizio 57: Buco in dielettrico
Un dielettrico con costante dielettrica κout è immerso in un campo elettrico esterno E ext . Il dielettrico contiene
un buco di forma a) lunga; b) corta; c) sferica. Calcolare il campo elettrico dentro il buco.
bSoluzione:
a) Ein = Eext .
b) Ein = Din = Dout = κout Eout .
c) Consideriamo il problema generale di un oggetto dielettrico di costante in immerso in un dielettrico
esterno di costante out . Le condizioni al bordo che devono essere soddisfatte sulla superficie della sfera
sono
⊥
⊥
=
=
out Eout
= in Ein
,
Eout
= Ein
Cioè conta solo il rapporto κ = in /out . Le soluzioni ottenute ai punti a) e b) di questo esercizio e di
quello precedente soddisfano a questa proprietà generale.
Per trovare la soluzione basta quindi sostituire κ → 1/κout nella soluzione c)0 dell’esercizio precedente.
La stessa soluzione è riottenuta tramite un calcolo esplicito nell’esercizio successivo.
Esercizio 58: Sfera dielettrica in dielettrico
Un dielettrico di costante dielettrica out contiene un buco sferico di raggio r e costante dielettrica in . Si studi
il sistema in presenza di un campo elettrico esterno Eext .
bSoluzione: Proviamo a trovare una soluzione assumendo che il campo interno sia Etot = Ein costante, e che
il campo esterno sia Etot = Eout = Eext + campo generato da un dipolo P . Ci sono 2 incognite: Etot e P . Le
condizioni al bordo che devono essere soddisfatte sulla superficie della sfera sono
r
r
out Eout
= in Ein
,
θ
θ
Eout
= Ein
Esplicitamente
out (Eext − 2
kP
) cos θ = in Ein cos θ,
r3
da cui
Ein =
3Eext
,
2 + in /out
(Eext +
kP
) sin θ = Ein sin θ
r3
kP
out − in
= Eext
r3
2out + in
34
Capitolo 3. Dielettrici
Esercizio 59: Attrazione fra dielettrici
Perchè un pettine attrae pezzettini di carta?
bSoluzione: Storicamente fu una delle prime manifestazioni dell’elettricità (èlectron non è inglese ma greco,
e vuol dire ambra. Anche un pettine di plastica va benissimo). Il fatto che la carta attratta rimanga poi
appiccicata indica che c’entrano i dielettrici. Se fossero invece cariche libere si neutralizzerebbero appena si
toccano.
Un dielettrico in un campo uniforme non sente nessuna forza. Questo è ovvio per geometrie semplici (e.g.
cubo o cilindro orientato lungo il campo), è stato verficato in precedenza nel caso di una sfera, ed in generale è
dovuto al fatto che il dielettrico si polarizza lungo il campo. Un dipolo in campo elettrico costante non sente
forze.
Un dielettrico in un campo elettrico non uniforme viene attratto verso campi grossi: lo abbiamo visto nel
caso particolare del condensatore piano, dove erano gli effetti ai bordi a generare la forza attrattiva ∝ E 2 . La
seconda potenza non è specifica di questa geometria semplice, ed è dovuta al fatto che F = σpol E è che la carica
di polarizzazione è a sua volta ∝ E.
In generale il dielettrico viene attratto verso campi grossi da una forza che è complicato calcolare in dettaglio.
Ma gli argomenti precedenti consentono di dire che in generale la forza è del tipo F ∝ ∇E 2 .
Un pettine sfrutta l’effetto delle punte per generare un campo elettrico abbastanza grosso ed abbastanza
dipendente dalla posizione in modo da generare una forza abbastanza grande da attrarre pezzetti di carta.
Capitolo 4
Correnti
E = ρj dove ρ è la resistività e σ = 1/ρ viene chiamata conducibilità. Per effetto Joule viene dissipata una
potenza W = j · E = ρj 2 . È utile introdurre la corrente totale I e definire la resistenza R in modo che V = IR.
Compiti rilevanti: Compitino del 19 dicembre 2003 es. 1. Compitino del 17 gennaio 2003, es. 3.
Esercizio 60: Capacitatore piano imperfetto
Calcolare il tempo di scarica di un condensatore piano (area A, distanza tra i piatti d) contenente un materiale
di conducibilità σ e costante dielettrica .
bSoluzione: Attenzione: σ qui non indica la densità superficiale di cariche. Si può ragionare in due modi:
1. Usando le equazioni fondamentali. Il campo elettrico E = Q/A genera una corrente J = σE e quindi
σ
Q̇ = −AJ = − Q
Q(t) = Q(0)e−t/τ
risolta da
dove
τ=
.
σ
2. Usando le formule valide per circuiti. La capacità vale C = A/d, la resistenza R = d/Aσ. Quindi
τ = RC = /σ.
Notare che τ non dipende da A e d, cioè da quanto è grosso il condensatore. Questo rende più semplice il
funzionamento delle cellule: il tempo di scarica non varia quando la membrana diventa più spessa o grossa.
Esercizio 61: Scarica di sfera carica
Calcolare il tempo di scarica di una sfera di raggio a in un dielettrico di conducibilità σ e costante dielettrica .
bSoluzione: Esce una corrente radiale. Siccome ha divergenza zero le cariche flusicono verso distanza infinita
senza accumularsi. Facendo il calcolo direttamente ottengo:
flusso di J
flusso di E
z }| {
z }| {
σ
dQ
= − 4πr2 Jr = − 4πr2 Er σ = − Q
dt
e quindi Q(t) = Q(0)e−t/τ con τ = /σ. Per t → ∞ il materiale si è comportato come un conduttore, per
t τ = /σ come un dielettrico.
Posso riscrivere la corrente i = Q̇ come i = V /R dove V = Q/4πa è la differenza di potenziale e R = 1/4πaσ.
Avevamo visto che una sfera ha una capacità C = 4πa. Quindi possiamo schematizzare il sistema come un
circuito (chiuso all’infinito) con costante tempo τ = RC = /σ.
Esercizio 62: Resistenza fra sfere concentriche
Calcolare la resistenza fra due sfere concentriche di raggi a e b in un materiale di resistività ρ.
35
36
Capitolo 4. Correnti
bSoluzione: Posso calcolarla indirettamente interpretando il sistema come una serie infinita di resistenze:
come visto nell’esercizio precedente la resistenza di un guscio di spessore dr vale dR = ρ dr/4πr2 . Sommando
le resistenza di tutti i gusci in serie viene
Z
ρ 1 1
R = dR =
( − )
4π a b
Se a b conta solo il primo termine: l’integrale è dominato dalla zona vicino alla sfera piccola.
Esercizio 63: Sonda marina
Due sfere di raggi a vengono calate in mare a distanza d e connesse da un filo conduttore. Calcolare la resistenza
del circuito.
bSoluzione: Domina la zona vicino alle sfere (o attorno alla sfera piccola, se avessero dimensioni diverse):
quindi l’elettrostatica consente e.g. di misurare localmente la salinità del mare. Se passa un branco di pesci fra
le sfere non me ne accorgo. Per pescare servirà l’elettrodinamica.
Si può schematizzare il sistema come due resistenze R = ρ/4πa in serie. Per a = 25 cm e ρ = 25 ohm cm
viene 2R = 0.27 ohm.
Esercizio 64: Semipiano dielettrico imperfetto
iωt
Un dipolo oscillante p = p0 e
e conducibilità σ.
viene posto nel vuoto a distanza d da un semispazio x < 0 di costante dielettrica
bSoluzione: In analogia all’esercizio a pagina 30, provo una soluzioni con ‘dipoli immagini’
(p a x = d)+(p0 a x = −d) per x > 0, zona 2
E=
(p00 a x = d)
per x < 0, zona 1
Questo soddisfa le equazioni
∇ · D = ρfree
∇×E =0
∇ · J = −ρ̇free
J = σE
D = E
per x < 0 e x > 0, dove l’unica carica è il dipolo p. Sul bordo (chiamdo w la densità di carica)
1
2
E=
= E=
,
2
1
E⊥
− κE⊥
= w/0
1
ẇ = −σE⊥
cioè
(ṗ + ṗ0 − κṗ00 ) = −
p − p0 = p00
2
1
1
Ė⊥
− κĖ⊥
= −σ1 E⊥
/0
:
σ 00
p
0
Eliminando p00 trovo una equazione per p0
ṗ0 =
σ/0
1−κ
(p − p0 ) −
ṗ
1+κ
1+κ
Assumendo p0 = p00 eiωt (dopo un transiente) trovo
p0 = p
1 − iω0 (1 − κ)/σ
1 + iω0 (1 + κ)/σ
Un dipolo ruotante è descritto da p ∝ (1, i, 0).
Esercizio 65: Diodo termoionico
Gli elettroni escono dal catodo con velocità nulla a V0 = 0. Calcolare come la corrente dipende dalla differenza
di potenziale V .
Capitolo 4. Correnti
37
2
bSoluzione: Gli elettroni acquistano velocità m
2 v = eV (x) e generano una densità di corrente J = ρv. Se J è
piccola e non modifica V , abbiamo finito. A basso V la corrente è limitata dalla densità di carica degli elettroni.
L’intasamento massimo si ha quando gli elettroni schermano completamente il campo elettrico esterno dando
E = 0 al catodo (se E < 0 nulla esce e gli elettroni fuori vengono spazzati via). Utilizzando l’equazione di
Poisson −ρ/0 = V 00 si ottiene
r
2e
00
V
J = ρv = −V 0
m
In condizioni stazionarie J è costante in x. Ottengo una equazione differenziale per V :
√
j
d V 02
00
V =√
:
− 2j V = 0
dx 2
V
p
(j = −J m/2e/0 ). La costante di integrazione vale zero in quanto V 0 = −Ecatodo =. Integrando ancora
0
p
V = 2 jV 1/4
Z
V
:
dV V
−1/4
Z
x
p
2 j dx
=
0
0
:
p
4 3/4
V
= 2 jd
3
ed, inserendo il valore di j,
V
3/2
3d2 J
=
20
r
m
2e
cioè J ∝ V 3/2 : non segue la legge di Ohm.
Calcoli simili permettono di studiare giunzioni fra semiconduttori, usati per costruire diodi più moderni.
Esercizio 66: Piatto dielettrico
Un piatto di dielettrico con costante dielettrica e resistività ρ viene messo in un campo elettrico esterno Eext ,
che viene improvvisamente rimosso. Studiare cosa succede. Stessa domanda per un campo elettrico esterno
Eext = E0 eiωt lentamente variabile.
bSoluzione:
• Dopo un po’di tempo si forma una densità di carica σ = 0 Eext sul bordo destro e −σ su quello sinistro.
Subito dopo che il campo Eext è stato rimosso rimane la σ che genera un campo E = −Eext all’interno
del conduttore e quindi una corrente j = E/ρ che inizia a riequilibrare le cariche. Siccome ∇ · E = 0
non si generano cariche di volume. Da ρ̇ = −∇ · j, tenendo che j = 0 fuori dal conduttore segue che
σ̇ = +j = −σ/0 ρ da cui σ(t) = σ(0)e−t/τ con τ = ρ. In questo primo problema il segno giusto è fissato
da ovvie considerazioni fisiche.
• Come prima ρ = 0 dentro il conduttore, e si accumulano cariche ±σ ai due bordi, lasciando un campo
elettrico interno Ein . Le condizioni di raccordo sono
0 Eext = Ein + σ
σ̇ = j = Ein /ρ
:
σ + τ σ̇ = 0 Eext
In condizioni stazionarie la soluzione è
σ = σ0 eiωt
con
σ0 =
0 E0
1 + iωτ
e quindi
Ein =
0 E0
1
1 + 1/iωτ
Se ω = 0 Ein = 0 (conduttore perfetto). Se ωτ 1 Ein = E0 /κ (dielettrico perfetto). Per valori intermedi
si ha un campo con modulo intermedio che oscilla con ritardo di fase ϕ = arctan ωτ . Il tutto corrisponde
a modificare il conto statico usando una costante dielettrica complessa κ → κ̂ = κ + 1/iωρ0 .
Esercizio 67: Sfera dielettrica
Un sfera di raggio r composta da un materiale con costante dielettrica e resistività ρ viene messa in un campo
elettrico esterno Eext = Êext Re (1, i, 0)eiωt lentamente ruotante.
38
Capitolo 4. Correnti
a) Mostrare che la carica libera sta solo sul bordo.
b) Scrivere le condizioni di raccordo.
c) Verificare che la sfera acquista una polarizzazione P (t) uniforme ruotante con ritardo di fase.
d) Trovare il momento delle forze M , ed il valore di ω per il quale M è massimo.
bSoluzione:
a) Dentro il dielettrico J ∝ D. Prendendo la divergenza segue ρ̇ ∝ ρ: se all’inizio ρ = 0 la carica fluisce
senza accumularsi.
b) In generale (con fuori il vuoto)
E=in = E=out ,
E⊥out = κE⊥in + σ/0
σ̇ = E⊥in /ρ
Assumendo un campo Ein costante ed un campo Eout = Eext + (campo generato da un dipolo P )
(Ein − Eext −
kP
) sin θ = 0
a3
(κEin − Eext + 2
kP
σ
) cos θ =
3
r
0
σ̇ =
Ein
cos θ
ρ
c) Assumendo condizioni stazionarie P = P̂ (1, i, 0)eiωt , derivando la prima equazione e sostituendo σ̇ la
riscrivo in forma analoga all’equazione statica
(κ̂Êin − Êext − 2
P̂
) cos θ = 0
r3
κ̂ ≡ κ +
1
iωρ0
Quindi la soluzione è analoga a quella ottenuta nel caso statico
k P̂
1 − κ̂
−1
= Êext
= Êext
r3
2 + κ̂
3 + iωτ
avendo messo per semplicità κ = 1. [Il segno non viene]
d) M = Re P ×Re E massima per ωτ = ±1 (a meno della riduzione nel modulo di P ) (nel sistema Terra-Luna
le maree avvengono con ritardo di fase provocando un trasferimento di momento angolare).
Capitolo 5
Circuiti
Risolvere le equazioni di Maxwell è complicato. È facile ottenere soluzioni approssimate che descrivono alcune
situazioni di interesse pratico (‘circuiti’).
Esercizio 68: Resistenze in parallelo
Verificare che la corrente si ripartisce minimizzando la dissipazione per effetto Joule.
bSoluzione: L’enegia dissipata vale W = R1 I12 + R2 I22 . Chiamando I la corrente totale sia ha I2 = I − I1 .
Quindi
dW
R1 R2
= 2R1 I1 + 2R2 (I1 − I) = 0
quindi
R1 I1 = I
= R= I
dI1
R1 + R 2
Che sia un minimo e non un massimo, lo si può vedere nel caso R2 R1 : tutta la corrente flusice nella resistenza
piccola minimizzando l’effetto Joule.
Esercizio 69: Resistenze su cubo
Calcolare la resistenza totale del circuito in fig. 5.1a, assumento che le singole resitenze abbiano un valore
comune R.
bSoluzione: È un esempio di circuito non decomponibile come combinazioni di serie e paralleli. Lo si potrebbe
risolvere scrivendo le equazioni di Kirchoff, però si fa prima a dare la risposta ad occhio sfruttando la simmetria
del problema. Per motivi di simmetria le correnti si dividono come in figura. Quindi la differenza di potenziale
fra i due capi vale V = IR( 13 + 16 + 13 ) = I · 56 R.
Se invece uno collegasse due spigoli opposti sulla stessa faccia del cubo, non passerebbe corrente nelle due
linee verticali degli altri due spigoli opposti...
Esercizio 70: Ponte di Wheatstone
Risolvere il circuito in fig. 5.1b.
bSoluzione: È un esempio di circuito non decomponibile: cioè non si può evitare di applicare le leggi di
Kirchoff vedendolo come combinazioni di serie e paralleli. In pratica lo si usa con: R3,4 resistenze fisse note,
R2 resistenza ignota da misurare, R1 resistenza variabile nota, da scegliere in modo tale che la differenza di
potenziale VBD = 0, cioè che non passi corrente attraverso R.
Iniziamo dal caso R = ∞: VBD = 0 (cioè il circuito è ‘bilanciato’) se R2 R4 = R1 R3 . In generale
VDB = V
R1
R4
−
R1 + R 2
R3 + R4
39
=V
R 1 R 3 − R2 R4
(R1 + R2 )(R3 + R4 )
40
Capitolo 5. Circuiti
B
I/3
I/3
R1
R2
A
C
R
I/6
R4
R3
D
I/3
Figura 5.1: (a) Circuito a cubo. (b) Ponte di Wheatstone.
In condizioni di bilanciamento R2 /R1 = R3 /R4 = r. Per studiare la sensitività dello strumento (e quindi
calcolare l’incertezza sperimentale su R2 ) consideriamo una variazione delle resistenze Ri → Ri + δRi : essa
produce
δR1
δR2
δR3
δR4
r
−
+
−
V.
VDB = 0 +
(1 + r)2 R1
R2
R3
R4
Passiamo al caso di R finito. In pratica R compare perchè un qualunque strumento che misura VDB lo fa
introducendo una resistenza R < ∞ fra i capi B e D. Risolviamo quindi il circuito completo utilizzando il
metodo delle maglie: ci sono 7 incognite: VA , VB , VC , VD e le tre correnti di maglia I1 (a sinistra, circolante in
direzione A → B), I2 (a destra in direzione C → B) ed I0 (sotto, in direzione A → D → C). Siccome contano
solo le differenze di potenziale si può scegliere VA = 0, riducendo il numero di incognite a 6. Ci sono poi 6
equazioni, una per ogni tratto di circuito:
VB − VA = R1 I1 ,
VD − VA = R4 (I0 − I1 )
VB − VC = R2 I1 ,
V C − VA = 0
VD − VC = −R3 (I0 + I2 )
VD − VB = R(I1 + I2 ).
Dopo calcoli noiosi si ottiene
VDB =
(R1 R3 − R2 R4 )V
.
(R1 + R2 )(R3 + R4 ) + (R1 R2 R3 + R1 R2 R4 + R1 R3 R4 + R2 R3 R4 )/R
Vediamo quindi che per calcolare il valore centrale di R2 non serve tenere conto di R, che invece modifica la
sensibilità dello strumento.
Esercizio 71: Impedenze
Mostrare che per una corrente oscillante I = I0 eiωt vale ZR = R
ZC =
1
iωC
ZL = iωL.
bSoluzione: Le cadute di potenziale ai capi di resistenza, condensatore, induttanza valgono
RI
Q
I
=
C
iωC
LI˙ = iωLI
Esercizio 72: Potenza dissipata
Calcolare la potenza dissipata su di una generica impedenza Z.
bSoluzione: La potenza dissipata vale W = V I dove V = ZI. Il prodotto non è una operazione lineare:
Re(z1 z2 ) 6= Re(z1 )Re(z2 ). Bisogna tornare ai numeri reali: scrivendo Z = R + iY
hW i = h(I0 R cos ωt − I0 Y sin ωt) · (I0 cos ωt)i =
cioè solo la parte resistiva dissipa energia.
Z cos φ 2
R 2
I0 =
I0
2
2
Capitolo 5. Circuiti
41
Esercizio 73: Filtro che taglia frequenze alte
Costruirlo.
bSoluzione: Ad esempio metto RC in serie e leggo il voltaggio ai capi di C: a grosso ω la sua impedenza
Z = 1/ıωC decresce e la maggior parte della caduta di potenziale avviene su R.
VC 2
1
VC
1
=
=
r = Vin
1 + iωRC
Vin 1 + (ωRC)2
Per R = 200 kΩ si ha una riduzione di 3 dB a ν = 1 Hz (r = 10−0.3 = 0.512 ≈ 1/2) se C = 0.8 µF, che a ν = 50
Hz corrisponde a r = 0.4 10−3 i.e. 34 dB.
Esercizio 74: Filtro che taglia frequenze basse
Costruirlo.
bSoluzione: Metto RC in serie e leggo il voltaggio ai capi di R, oppure metto RL e leggo ai capi di L.
Esercizio 75: Pendolo accoppiato
Mostrare che due pendoli connessi da una molla k soddisfano alle stesse equazioni di due maglie LC con in
comune una capacità C 0 .
bSoluzione: Usando le correnti di maglia I1 e I2 le equazioni sono
Q1 − Q2
Q1
+
C
C0
Usando le impedenze e risolvendo si trova
V1 = LQ̈1 +
I2 =
0 = LQ̈2 +
Q2
Q2 − Q1
+
C
C0
iωV1 C 2
V1
ω22 − ω12
=
iω
(CLω 2 − 1)(CC 0 Lω 2 − 2C − C 0 )
2L (ω 2 − ω12 )(ω 2 − ω22 )
C
I2
=
0
I1
C + C − CC 0 Lω 2
Avendo definito ω12 = 1/LC e ω22 = 1/LC + 2/LC 0 . Alla media delle frequenze I1 = 0.
Le equazioni del moto del pendolo sono
x2
x1
+ k(x1 − x2 ) = F
mẍ2 + mg
+ k(x2 − x1 ) = 0
mẍ1 + mg
`
`
che hanno la stessa forma con 1/LC ↔ g/`, 1/LC 0 = k/m.
Per trovare i modi normali provo una soluzione xi = x̂i eiωt
x1
−ω 2 + g/` + k/m
−k/m
=0
−k/m
−ω 2 + g/` + k/m
x2
Il determinante vale zero per
1. ω 2 = ω12 = g/`, x1 = x2
2. ω 2 = ω22 = g/` + 2k/m, x1 = −x2
Mettendo F (t) = F̂ eiωt viene
x2 =
F
F
ω22 − ω12
=
k − (k − mω 2 + gm/`)2 /k
2m (ω 2 − ω12 )(ω 2 − ω22 )
x2
k`
ω12 − ω22
=
=
x1
k` + mg − m`ω 2
2ω 2 − ω12 − ω22
che corrispondono alle soluzioni per i modi normali
x1 + x2 = −
F
1
m ω 2 − ω12
x1 − x2 = −
F
1
m ω 2 − ω22
Nel caso del circuito, il modo normale I1 = I2 (niente corrente su C 0 ) vede un’impedenza iωL + 1/iωC che vale
zero per ω = ω1 ; il modo I1 = −I2 vede un’impedenza 21 (iωL + 1/iωC) + (1/iωC 0 ) che vale zero per ω = ω2 .
42
Capitolo 5. Circuiti
R1
R1
R2
R1
R2
R1
R2
R1
R1
R2
R2
R2
R
=
Figura 5.2: Fig. 5.2a: circuito attenuatore. Fig. 5.2b: resistenza equivalente.
Esercizio 76: Attenuatore
Si determini la resistenza totale R del circuito infinito in fig. 5.2 ed i potenziali nei vari punti
bSoluzione: Siccome ∞ = ∞ + 1 per trovare la resistenza R equivalente al circuito si impone che R sia eguale
ad un passo della catena seguito da R:
p
R1 + R12 + 4R1 R2
1
(5.1)
R = R1 +
:
R=
1/R2 + 1/R
2
Per trovare il potenziale dopo il primo passo, rimpiazziamo tutte le resistenze successive con la resistenza
equivalente R, ottenendo un circuito con 3 resistenze R1 , R2 ed R, di resistenza totale R: circola una corrente
totale I = V /R
La differenza di potenziale V 0 ai capi di R vale
V0 =V
R2
R + R2
Nei passi successivi la differenza di potenziale decresce in modo geometrico: V (n) = V /(1+R/R2 )n . Ad esempio,
per dimezzare la differenza di potenziale ad ogni passo serve R = R2 , cioè R2 = 2R1 .
In pratica non si può costruire una catena con un numero infinito di passi: per terminare il circuito dopo
un numero finito di passi senza scompensarlo, basta terminarlo con una resistenza R.
Esercizio 77: Catena LC
Come nella fig. 5.2, con R1 → Z1 = iωL e Z2 = 1/iωC.
bSoluzione: Pre semplificare la formula (5.1) divido ogni L in 12 L + 12 L e metto L/2 a sinistra:
Zeff = Z −
p
Z1
= (Z1 /2)2 + Z1 Z2 =
2
p
2
se ω < ω0
p(L/C) − (ωL/2)
2
i (ωL/2) − (L/C) se ω > ω0
√
dove ω0 = 2/ LC. La cosa sorprendente è che a basse frequenze un circuito con solo L e C si comporti come
una resistenza; il motivo fisico è che l’energia sembra scomparire in quanto viene trasmessa attraverso la catena.
Abbiamo già visto che il potenziale varia lungo la catena come
 p
(L/C) − (ωL/2)2 − i(ωL/2)



p
= eiδ se ω < ω0

2 + i(ωL/2)
Z
Z
−
Z
(L/C)
−
(ωL/2)
2
1
p
Vn = α n V0
α=
=
=

(ωL/2)2 − (L/C) − (ωL/2)
Z + Z2
Z


<1
se ω > ω0
 p
(ωL/2)2 − (L/C) + (ωL/2)
p
A bassa frequenza tutto funziona se si termina la catena finita con R = L/C. (Se si scambia L ↔ C è come
cambiare ω → 1/ω: la catena risultante taglia le frequenze basse invece di quelle alte).
Se le differenze di potenziale e di corrente fra due elementi vicini sono piccole, valgono
∆V = LI˙
∆I = ∆Q̇ = C ∆V̇
R
Capitolo 5. Circuiti
43
cioè
dV
L dI
=
dx
∆x dt
dI
C dV
=
dx
∆x dt
:
L C d2 V
d2 V
=
2
2
dx
|∆x{z∆x} dt
v2
La rete LC fornisce una descrizione approssimata di una linea di trasmissione. Ad esempio per un cavo coassiale
L = ∆x
µ0
`
2π
C = ∆x
2π0
`
v=√
1
=c
0 µ0
(` ≡ ln
R2
).
R1
Per ∆x → 0 ω0 → ∞: una linea ideale trasmette tutto, ed il cavo equivale ad una resistenza
r
µ0 `
R = lim Z =
= 60 Ω · `.
∆x→0
0 2π
Parte II
Magnetostatica
Capitolo 6
Campi magnetici
Le equazioni di base sono F = q(E + v × B) e le equazioni di Maxwell (µ0 = 4πkm , km = 10−7 Tesla m/A)
∇ × B = µ0 j
∇·B =0
che implicano ∇ · j = 0. In forma integrale
I
B · ds = µ0 Φ(j) = µ0 i
Φ(B) = 0
Campo magnetico generato
B=
µ0
r
µ0
qv × 3 =
4π
r
4π
I
i ds ×
r
r3
Forza prodotta da un campo magnetico: F = qv × B = j × B = i ds × B. Un circuito chiuso in un campo
magnetico costante percorso da una corrente costante i sente F = 0 e momento M = m × B dove m = iSn
L’energia potenziale vale U = −m · B = −iΦ(B). Se il campo magnetico non è costante F ' ∇(m · B).
Unità di misura. Il campo magnetico viene misurato in Tesla = N/Am (Gauss = 10−4 Tesla). Il campo
magnetico terrestre sulla superficie vale circa 0.1 Gauss. Il massimo campo producibile è circa 10 Tesla. Il flusso
di B viene misurato in Weber = Tesla· m2 = Volt · sec.
Esercizio 78: Forza fra 2 cariche
Due elettroni si muovono parallelamente lungo traiettorie rettilinee a distanza a con velocità costante v c.
Calcolare la forza elettromagnetica
bSoluzione: La forza è diretta lungo la congiungente. La forza elettrica respinge e quella magnetica attira
F =
e2
µ0
e2
v2
− ev ·
ev =
(1 − 2 )
4π0
4π
4π0
c
c2 =
1
0 µ0
Il risultato sopra è sbagliato. Esistono altri effetti relativistici di O(v 2 /c2 ). Nel sistema in cui le cariche sono
in quiete F0 = e2 /4π0 e quindi la relatività dice che Fv = F0 /γ, invece che Fv = F0 /γ 2 Infatti i campi E e B
generati da cariche in moto vanno moltiplicati per γ.
Una carica può avere velocità media ∼ cm/s ∼ 10−10 c e quindi il suo campo magnetico è una correzione di
ordine 10−20 . Questa soppressione può venire compensata se ci sono NA ∼ 6 1023 cariche che si muovono nello
stesso verso (formando una corrente), messe assieme ad altrettante cariche di segno opposto (in modo che i loro
campi elettrici si cancellano). Siccome la materia è fatta in questo modo, ha senso studiare la magnetostatica.
Esercizio 79: Disco di Rowland
Un disco di raggio r = 20 cm con carica σ = 10−6 C/ m2 fa 200 giri al secondo. Stimare il campo magnetico
generato
45
46
Capitolo 6. Campi magnetici
bSoluzione: Quindi la corrente vale I ∼ 200 Q/ sec ∼ 10−5 A e, ad una distanza di 1 cm, genera un campo
magnetico
km I
B∼
∼ 10−10 T
cm
(km = µ0 /4π = 10−7 ) 105 volte minore del campo magnetico terrestre (circa costante attorno al disco). Il
grosso campo elettrico E ∼ σ/0 ∼ 105 V/m (0 ∼ 10−11 C/Vm) genera una forza repulsiva di circa 1 N e veniva
schermato con un conduttore. Sui condensatori si può accumulare Q ∼ C ma...??
Esercizio 80: Attrazione o repulsione?
Perchè due cariche uguali si respingono, mentre due fili con correnti uguali si attraggono?
bSoluzione: Non esiste un modo semplice di vederlo sapendo solamente che u = 0 E 2 /2 + B 2 /2µ0 . Invece di
cariche e fili consideriamo piani che rendono la geometria più semplice.
• La pressione elettrostatica su di un piano con densità di carica superficiale σ = 0 (E2⊥ − E1⊥ ) vale
±p = σ
E1⊥ + E2⊥
0 2
= ∆( E⊥
) = ∆uE
2
2
uE =
0 2
E
2
avendo usato il fatto che E= è continuo. Per determinare il segno pensiamo a due piani xy paralleli con
densità di cariche ±σ: essi si attraggono, tendendo a minimizzare l’energia uE .
• La pressione magnetostatica su di un piano con densità di corrente superficiale Υ = (B=2 − B=1 )/µ0 vale
±p = Υ
B2
B1= + B2=
= ∆( = ) = ∆uM
2
2µ0
uM =
B2
2µ0
avendo usato il fatto che B⊥ è continuo. Per determinare il segno pensiamo a due piani xy paralleli con
densità di corrente ±Υx : essi danno luogo ad un campo magnetico By nella zona interna, ma si respingono.
Quindi il segno è opposto nei due casi. La pressione è data non da ∆u dove u = uE + uM ma da ∆L dove
L = uE −uM . La notazione L indica che questa sarà la Lagrangiana Lorentz-invariante dell’elettromagnetismo:
è noto che L = K − V mentre H = K + V . La ragione fisica del ribaltamento del segno nel caso magnetico è
che mantenere correnti costanti costa energia, mentre le cariche rimangono automaticamente costanti.
Esercizio 81: Filo rettilineo
Calcolare il campo magnetico generato da un filo rettilineo di raggio a percorso da una densità di corrente
costante j.
bSoluzione: B ha solo una componente radiale. Fuori dal filo (r > a) la ‘legge di Ampere’ fornisce 2πr Br = µ0 i
dove i = j πa2 . Dentro il filo (r < a) i = jπr2 e quindi Br = µ0 jr/2 = µ0 ir/2πa2 .
Esercizio 82: Cavo coassiale
Calcolare il campo magnetico generato da un filo rettilineo di raggio a percorso da una densità di corrente
costante j, circondato da un cilindro di raggi b e b0 > b lungo cui scorre uniformemente una corrente totale
opposta.
bSoluzione: Per r < b è come nell’esercizio precedente. Per r > b0 si ha B = 0. Per b < r < b0 si ha
µ0 j 2
r 2 − b2
Br =
πa 1 − 02
2πr
b − b2
Capitolo 6. Campi magnetici
47
Esercizio 83: Spira circolare
Calcolare il campo magnetico generato da una spira circolare di raggio a nel piano xy percorso da una corrente
i.
bSoluzione: Lungo l’asse è facile integrare: B ha solo una componente lungo z
Bz =
µ0 ia
µ0
µ0 i
ia2
2πa
cos
θ
=
cos
θ
=
4π r2
2 r2
2 (a2 + x2 )3/2
ds ed r sono ortogonali, e ds × r forma un angolo θ con l’asse z. Nel centro θ = 0 e B = µ0 i/2a.
Quindi l’induttanza della spira vale circa L ∼ µ0 a. È complicato fare un calcolo preciso, che dipende dallo
spessore del filo.
Esercizio 84: Due spire circolari
Calcolare il campo magnetico generato da due spira circolare di raggio a a distanza d nel piano xy percorse da
una corrente N i.
bSoluzione:
Bz =
i
µ0 N ia2 h 2
(a + x2 )−3/2 + (a2 + (d − x)2 )−3/2
2
Il campo magnetico è molto uniforme fra le due spire quando d = a (spire di Helmholtz): infatti per tale valore
si ha d2 Bz /dx2 = 0 a x = d/2 e Bz (x = d/2) = (4/5)3/2 µ0 N i/a.
Assumiamo che il campo magnetico sia zero fuori e costante dentro. Un elettrone entra camminando lungo
l’asse. Dentro percorre un arco di circonferenza con raggio di curvatura r = mv/eB e quindi viene deflesso di
sin θ = a/r.
p
p
Se deve essere θ = 45◦ per v = 2eV /m con V = 25 kV e i = 2 A servono n = (1.4/µ0 i) mV /e ≈ 200
avvolgimenti per spira.
Esercizio 85: Filo a U
Calcolare il campo magnetico nel centro del semicerchio generato da un filo percorso da una corrente i che forma
una U di raggio a.
bSoluzione: Raddoppiando i due mezzi fili ed il semicerchio, Bz è la metà del Bz prodotto da 2 fili rettilinei
infiniti e da un cerchio:
1 µ0 i
2 µ0 i
+
Bz =
2 2πa 2 2a
Notare che la configurazione di fili riflessa produce lo stesso B, in quanto esso è uno vettore assiale, non un
vettore.
Esercizio 86: Piano a U
Come prima, con il filo rimpiazzato da un piano.
bSoluzione: Nel centro equivale alla metà di due piani e di un solenoide. La cosa interessante è che il campo
magnetico è costante nella regione interna.
48
Capitolo 6. Campi magnetici
Esercizio 87: Solenoide rettilineo infinito
Calcolare il campo magnetico generato dentro un solenoide rettilineo infinito.
bSoluzione: Fuori vale zero, dentro è costante orientato come il solenoide. La legge di Ampere fornisce
B = µ0 ni dove n è il numero di spire per unità di lunghezza.
Il flusso concatenato ad un solenoide abbastanza lungo da poter trascurare effetti ai bordi vale Φ = BN S =
LI (dove N = n` è il numero di spire e S la loro area) con L = µ0 SN 2 /`. Quindi l’energia necessaria per
˙ Quindi la potenza vale W = −V I =
accendere una corrente i vale U = LI 2 /2 (dimostrazione: V = −Φ̇ = LI.
1
2
˙
LI I = d( 2 LI )/dt). La si può calcolare anche come l’integrale della densità di energia u = B 2 /2µ0 sul volume:
U = (µ0 nI)2 S`/2µ0 .
Esercizio 88: Solenoide rettilineo finito
Dire qualcosa sul campo magnetico generato dentro un solenoide rettilineo semi-infinito.
bSoluzione: Usando la forza bruta, integro il campo di una spira, mettendo n spire per unità di lunghezza da
−` a 0:
Z 0
ia2
µ0 ni
`+x
x
0 µ0
p
√
Bz (x) =
n dx
=
−
2 (a2 + (x − x0 )2 )3/2
2
a2 + x2
a2 + (` + x)2
−`
|
{z
}
1 per ` → ∞
Il principio di sovrapposizione consente di dire che: 1) Sul bordo B vale la metà che in un punto interno, come
vedo immaginando di completare aggiungendo l’altra metà del solenoide. In generale Bz (x) + Bz (−x) = Bz∞ . 2)
La linea di campo di B che sfiora il bordo esce perpendicolare al solenoide, perchè’ quando completo la somma
deve fare zero; le componenti orizzontali si sottraggono ma le eventuali componenti verticali si sommerebbero
(B è uno pseudovettore) e quindi devono valere zero. Chiaro
√ con un disegno. 3) La linea di campo che sfiora il
bordo, dentro il solenoide sta ad una distanza limite a/ 2 dal centro (il raggio che contiene metà del flusso),
con relazioni analoghe per le linee interne.
Esercizio 89: Solenoide toroidale
Calcolare il campo magnetico generato dentro un solenoide toroidale.
bSoluzione: B è circolare in quanto spire disposte simmetricamente rispetto al punto dove viene calcolato B
danno contributi che si cancellano alle componenti non radiali. Quindi, per la legge di Ampere, fuori B = 0 e
dentro 2πrBr = µ0 N i dove N è il numero totale di spire.
Esercizio 90: Sfera ruotante
Sulla superficie di una sfera omogenea di massa M e raggio R è distribuita uniformemente una carica Q. La
sfera ruota con velocità angolare ω. Calcolare il momento magnetico. Scrivere l’equazione del moto in presenza
di un campo magnetico uniforme B.
Capitolo 6. Campi magnetici
49
bSoluzione: Un anello fra θ e θ + dθ ha raggio r = R sin θ superficie dQ = Q dS/S con dS = 2πr · R dθ e
ruota con velocità v = ωr. Quindi trasporta una corrente
di =
dQ v
Qω
=
sin θ dθ
2πr
4π
Il momento magnetico vale
Z
µ=
o anche
QωR2
πr di =
4
2
Z
0
π
sin3 θ dθ =
QR2
ω
3
5
2M R2
Q
L
dove
g=
e
L=
ω
2M
3
5
è il momento angolare. Una sfera uniformemente carica avrebbe g = 1. Elettrone e muone hanno ge =
2.002319304374.. e gµ = 2.0023318406...
µ=g
Capitolo 7
Moto in campo magnetico esterno
Esercizio 91: Trottola magnetica
Trovare un modo di sospendere un dipolo magnetico in aria.
bSoluzione: La forza di un dipolo in un campo magnetico esterno è data da un’equazione analoga al caso di
un dipolo elettrico: U = −µ · B e F = −∇U , M = µ × B.
Discussione preliminare. Avevamo visto che F = −∇(−p·E) non funzionava per un dipolo elettrico orientato
lungo z in un Ez ∝ x (perchè tale E ha rotore diverso da zero, e quindi non può esistere in elettrostatica):
pensando il dipolo come 2 cariche è facile vedere che la forza deve essere zero. Nel caso magnetico (dove B può
avere rotore diverso da zero) la formula funziona: la forza è prodotta dal fatto che il lato della spira dove Bz è
più grosso sente una forza maggiore.
Applicata invece ad un dipolo orientato lungo B, e con un Bz (z), la formula dice che il dipolo è attratto
verso zone dove B è maggiore. Pensando al dipolo come ad una spira, è immediato vedere che non c’è nessuna
forza. La formula sballa perchè non esiste un campo magnetico con solo Bz (z): per avere ∇ · B = 0 serve anche
un Br (r, z). A livello grafico, il campo magnetico deve ‘incurvarsi’ verso l’esterno. È la componente Br che
genera la forza, anche se non appare in U .
Un’applicazione è la trottola magnetica. Se si mette un dipolo sopra il campo magnetico generato da una
spira, questo si allinea con la spira e ne viene attratto. Quindi non rimane sospeso.
Se invece l’ oggetto con dipolo magnetico µ (e.g. la sfera dell’esercizio precedente) viene anche fatto girare,
il possedere un momento angolare L ∝ µ gli impedisce di allinearsi a B. Esiste un momento delle forze che
produce la seguente equazione del moto:
dL
Q
=µ×B =g
L×B
dt
2M
che dice che L precede attorno al campo magnetico con frequenza di precessione ωp = g QB/2M , mantenendo in
media temporale la sua orientazione iniziale. Quindi se all’inizio µ viene messo anti-parallelo a B il magnetismo
genera una forza repulsiva che può tenere sospeso l’oggetto. Alla lunga l’attrito rallenta la rotazione, e l’oggetto
si allinea e casca. È grazie all’attrito che una bussola si allinea al campo magnetico terrestre.
Esercizio 92: Cilindro su piano inclinato
Un cilindro di raggio r e lunghezza `, appoggiato orizzontalmente su di un piano inclinato (di angolo α), è
percorso da una corrente i uniforme lungo la sua lunghezza. È presente un campo magnetico verticale. Per
quale valore di i il cilindro rimane fermo se a) l’attrito tra piano e cilindro è trascurabile b) l’attrito è tale che
il cilindro può rotolare senza strisciare.
bSoluzione: Il campo magnetico produce una forza orizzontale, diretta verso il piano inclinato. La corrente in
un angolo dθ produce dF = `B di = `Bi dθ/2π.
a) La componente della forza magnetica totale lungo il piano inclinato vale F = `Bi cos α. La componente
della forza gravitazionale lungo il piano inclinato vale F = mg cos α. Sono uguali se i = mg tan α/`B.
50
Capitolo 7. Moto in campo magnetico esterno
51
b) Il momento rispetto all’asse di appoggio deve essere zero. La gravità produce M = mgr sin α. Il campo
magnetico produce
Z
Z 2π
dθ
M = ∆y · dF =
r(cos α + sin θ)`Bi
= i`Br cos α
2π
0
Quindi serve la stessa i di prima (le due forze sono uniformi).
Esercizio 93: Ago magnetico
Una ago di momento magnetico µ è situato sopra la congiungente due fili paralleli orizzontali a distanza d.
I due fili sono percorsi da correnti i e −i. La distanza fra ago e ciascun filo è r. L’ago può ruotare in un
piano ortogonali ai fili, con momento di inerzia I. Calcolare la posizione di equilibrio ed il periodo delle piccole
oscillazioni.
bSoluzione: Il campo magnetico è verticale
Bz = 2
µ0 i h
2πr d/2
Il momento delle forze vale M = µ × B e l’equazione del moto è I θ̈ = µB sin θ (la posizione
di equilibrio è
p
θ = 0 o θ = π/2 a seconda del segno di B, il dipolo vuole allinearsi con B) da cui T = 2π I/µ|B|.
Esercizio 94: Carica in B costante
Studiare il moto di una particella di carica q in un campo B costante
bSoluzione: Risolvendo l’equazione del moto
mv̇ = qv × B
→
v̇ = v × ω B
ωB =
qB
m
si trova che v ha una componente costante v = parallela a B, ed una componente v ⊥ ortogonale a B che ruota
con frequenza ωB . Quindi fa una spirale. Chiamando a il raggio dell’orbita, si ha v⊥ = aωB i.e.
p⊥ = aqB = MeV 3 10−4
a q B
cm e Gauss
• • • • • • • • • • • • • • • • • •
Nei problemi successivi studieremo moti in campi magnetici più generali, che tendono a dare moti complicati:
le particelle spiraleggiano attorno a qualche traiettoria media. La cosa interessante da calcolare è la traiettoria
media, cioè la ‘velocità di drift’. Da questo punto di vista il moto in un campo magnetico costante è quindi
semplicemente un moto a velocità costante lungo le linee del campo, a meno di girellamenti.
Esercizio 95: Campo magnetico galattico
Stimare quando è grande, assumendo che l’energia cinetica in particelle sia comparabile all’energia in campo
magnetico.
bSoluzione: La galassia contiene, in media, un atomo di idrogeno per cm3 con velocità v ∼ km/sec.
ρ
mv 2
erg
B2
∼ 10−14
∼
2
cm2
2µ0
da cui B ∼ 10−10 Tesla = 10−6 G. Essendo debole viene un raggio r = p/eB grosso (scritta in termini di
p = mv la formula rimane valida anche per particelle relativistiche, per le quali p = mvγ), che ha comunque
52
Capitolo 7. Moto in campo magnetico esterno
effetti importanti perchè anche la galassia ha una grossa dimensione d ∼ Mpc: un protone di impulso p viene
deflesso
d
deB
d
B 3 1019 eV
θ≈ =
'
−9
r
p
1 Mpc 10 G
E
Quindi si può (o meglio potrà) fare astronomia solo con i pochi protoni che arrivano con energia abbastanza
grande. Gli altri spiraleggiano nella galassia e la loro direzione di arrivo non dà nessuna informazione sulla
sorgente che li ha generati.
Esercizio 96: Spettrometro
bSoluzione: Permette di selezionare particelle con dato q/m se uno ha un fascio monoenergetico.
Esercizio 97: Carica in B ed E costanti
Studiare il moto di una particella di carica q in campi E e B costanti.
bSoluzione: In generale possiamo assumere B = (0, 0, Bz ) e E = (0, Ey , Ez ). L’equazione del moto (nel limite
non relativistico), scritta in componenti è
mv̇x = qvy Bz
mv̇y = q(Ey − vx Bz )
mv̇z = qEz
La soluzione è
vx = vrot cos(ωB t) +
Ey
Bz
vy = −vrot sin(ωB t)
vz = qEz t
(ωB =
qBz
)
m
(7.1)
Il moto lungo z (in generale, lungo B) non si mescola con gli altri ed è ovvio. Lungo x, y la carica oltre a
ruotare fa un drift costante lungo x (in generale lungo E × B). La rotazione dipende da q, ma il drift no.
(Sub-esercizio: discutere il limite q → 0). Il moto globale è illustrato in fig. 7.1a per cariche con segno opposto.
Se particelle cariche entrano in una zona dove i campi elettromagnetici sono costanti ed hanno solo componenti
Ey e Bz , quelle con vx = Ey /Bz viaggiano imperturbate. Sfruttando questo fenomeno si può ottenere un fascio
monoenergetico (ad esempio utilizzabile nello spettrometro dell’esercizio precedente).
• • • • • • • • • • • • • • • • • •
Per ottenere in modo facile ed istruttivo la soluzione (7.1) conviene passare ad un nuovo sistema di riferimento
S 0 dove E e B sono paralleli, avendo solo una componente lungo z, tramite una trasformazione Galileiana
vx = vx0 + Ey /Bz (in generale con un boost B × E/B 2 ). In tale sistema S 0 la particella gira attorno al campo
magnetico costante.
Teoricamente questo equivale a dire che sotto una trasformazione galileiana v = v 0 + β i campi elettromagnetici trasformano come
E0 = E + β × B + · · · ,
B0 = B + · · ·
(E · B = E 0 · B 0 )
Queste trasformazioni sono corrette solo all’ordine più basso in β. Ad ordini superiori il risultato non sarebbe
consistente perchè le equazioni di Maxwell sono Lorentz-invarianti mentre quella di Newton è Galileo-invariante.
Einstein modificò F = ma in modo da renderla relativisticamente invariante, permettendo di discutere i termini
di ordine più alto in β, qui indicati con · · · Se non è verificata la condizione E/B c questi termini mancanti
diventano importanti. Se E/B > c vedremo che si può invece passare ad un sistema dove B = 0. Fisicamente,
questo corrisponde ad avere un campo elettrico cosı̀ forte che B non ce la più ad incurvare l’orbita.
Esercizio 98: Fotomoltiplicatore in B, E
Studiare di quanto viene ridotta l’efficienza di un fotomoltiplicatore con E = kV/cm se viene posto in un campo
magnetico B = Tesla a 45 gradi (figura 7.1).
Capitolo 7. Moto in campo magnetico esterno
53
d
L
z
B
y
x
γ
E
Figura 7.1: Fig. 7.1a: esempio di moto in E, B costanti di due particelle con carica opposta. Fig. 7.1b:
fotomoltiplicatore in campo magnetico.
bSoluzione: Un fotomoltiplicatore è un aggeggio simile ad un televisore che accelera elettroni (generati da
ionizzazioni γ, . . . ) facendoli sbattere su di uno schermo in modo da renderli rilevabili. Nei rivelatori di
particelle a volte si mettono campi magnetici, che incurvano le traiettorie di particelle ed anti-particelle in
direzioni opposte e con raggi che dipendono dalla loro massa, in modo da poterle distinguere.
Il campo elettrico di un fotomoltiplicatore è orientato in modo da accelerare gli elettroni verso lo schermo. Ma se c’è un campo magnetico gli elettroni non vanno nella direzione desiderata. Capire dove vanno è
un’applicazione dell’esercizio precedente. All’inizio gli elettroni hanno velocità zero, quindi nella (7.1) mettiamo
vrot = −Ey /Bz . Una particella che parte da (x, y, z) = (0, 0, 0) segue il percorso
x=
Ey
(tωB − sin tωB ),
Bz ωB
y=
Ey
(1 − cos tωB ),
Bz ωB
z=
eEz 2
t
2me
Numericamente la frequenza ωB ed il raggio di spiraleggiamento a valgono
ωB =
eB
= 1.7 1011 Hz,
me
vrot =
Ey
m
= 7 104 ,
Bz
s
a=
vrot
= 4 µm
ωB
cioè a è miscoscopico. Quindi in pratica gli elettroni
(A) ruotano su circonferenze di raggio trascurabile;
(B) driftano lungo x con velocità vrot ∼ 2 10−4 c: è un effetto trascurabile.
(C) accelerano raggiungendo v ∼ 0.1c lungo B e non più lungo E. Questo è l’effetto importante.
A causa di (C) una frazione d/L degli elettroni (cioè quelli che vengono ionizzati nella zona in cima a destra del
fotomoltiplicatore in fig. 7.1) vanno a sbattere sulle pareti laterali invece di venir rilevati sullo schermo. Ridurre
il campo magnetico o aumentare quello elettrico non migliora la situazione, fino a quando a d.
Esercizio 99: Ciclotrone
Mostrare che una particella libera di muoversi in un campo magnetico Bz ed in un campo elettrico oscillante
Re Ex e−iωt = Ex cos ωt viene accelerata lungo x.
bSoluzione: Se E = 0 la carica gira nel piano xy: riottengo questa cosa nota usando un primo trucco
matematico. Quando c’è roba che gira è utile introdurre z ≡ x + iy (attenzione: questo z non ha niente a che
fare con l’asse z) in quanto girare nel piano xy con frequenza ω e raggio r viene descritto in modo più compatto
come z = re−iωt .1 In termini di z le equazioni del moto diventano
mẍ = q ẏBz
mÿ = −q ẋBz
:
z̈ = −iq żBz
1 Questo trucco viene usato in calcoli più complicati (meccanica quantistica,. . . ), per cui è utile vederlo all’opera in questo
problema più semplice, dove non dà una grande semplificazione. Quindi se confonde le idee, conviene rifare i conti ritornando ad x
ed y.
54
Capitolo 7. Moto in campo magnetico esterno
e quindi ż(t) = ż(0)e−iωB t con ωB = qBz /m. Aggiungendo il campo elettrico l’equazione del moto diventa
mz̈ = −iq żBz + qEx e−iωt
:
ż =
iEx q/m −iωt
e
ω − ωB
dove abbiamo usato un secondo trucco matematico: abbiamo assunto z(t) ∝ e−iωt il che, come noto, trascura
il transiente e fornisce solo l’orbita limite.
Per ω = 0 si ritrova il drift a velocità costante vy = Ex /Bz precedentemente discusso: grazie alla i esso è
diretto lungo y.
Per ω 6= 0 la carica q gira, acquistando una grossa velocità se ω = ωB . Intuitivamente la particella gira nel
campo magnetico, ed ad ogni mezzo giro il campo elettrico viene riorientato in modo da essere sempre lungo il
moto della particella, che quindi viene accelerata lungo una spirale. Come al solito questa tecnica trascura il
transiente e Si può verificare che i raggi limiti ottenuti dalle seguenti similazioni numeriche sono in accordo con
il valore atteso:
ω / ωB = 0.8
ω / ωB = 0.9
ω / ωB = 1
ω / ωB = 1.1
ω / ωB = 1.2
Quindi il ciclotrone è un modo di accelerare particelle in uno spazio ridotto. Aggiungendo un termine di attrito
(che può essere causato da vari effetti fisici e.g. l’irraggiamento) il denominatore diventa ω − ωB + iγ, risonante
ma finito per ω = ωB .
La frequenza di rotazione ωB non dipende dall’energia della particella; questa semplificazione non è più
vero quando la particella acquista un’energia relativistica: in tale caso diventa necessario variare ω in modo
appropriato per ogni bunch.
Uno potrebbe pensare che sia più complicato ma anche più conveniente mettere un campo elettrico che gira
con la particella, in modo che E sia sempre diretto
√ lungo v. In termini di numeri complessi un E ruotante nel
piano xy si scrive come E = Ee−iωt (1, ±i, 0)/ 2. Il segno ± specifica la direzione di rotazione. L’equazione
del moto diventa
1±1
mz̈ = −iq żBz + q √ Ee−iωt .
2
Cioè se metto senso di rotazione sbagliato non accelero nulla, e se lo metto giusto non guadagno quasi nulla.
Infatti il campo elettrico oscillante può essere
come sovrapposizione lineare di due campi che
√ decomposto √
ruotano in direzioni diverse: (1, 0, 0) = (1, i, 0)/ 2 + (1, −i, 0)/ 2: uno è in risonanza e l’altro non ha effetto.
Esercizio 100: Carica in B con direzione non uniforme
Moto in un B la cui direzione varia lentamente: studiare il moto di una particella di carica q in un campo
magnetico con Bθ costante.
bSoluzione: Le equazioni del moto in coordinate cilindriche (ρ, θ, z), dove B ha una componente Bθ costante,
sono
aρ = ρ̈ − ρθ̇2 = −ωB ż,
aθ = ρθ̈ + 2ρ̇θ̇ = 0,
z̈ = ωB ρ̇
dove ωB ≡ qBθ /m. In prima approssimazione procede lungo le linee del campo facendo una spirale di raggio
a. Quindi ρ = R e θ̇ = v|| /R. Scriviamo la soluzione al primo ordine perturbativo in a/R: la 1a equazione del
moto fornisce ż = v||2 /ωB R. Avendo assunto R a la velocità di drift è piccola: ż/v|| ∼ a/R.
Lo si può capire in modo intuitivo notando che per far curvare la traiettoria media lungo le linee del campo
serve una forza diretta lungo ρ̂. Abbiamo visto all’esercizio precedente che il moto in B ed E ortogonali è un
drift lungo B × E. In questo caso ρ̂ × θ̂ = ẑ.
Capitolo 7. Moto in campo magnetico esterno
55
Esercizio 101: Carica in B con modulo non uniforme
Moto in un B il cui modulo varia lentamente in direzione ortogonale a B. Studiare il moto di una particella di
carica q in un campo magnetico B = (0, 0, Bz (x)).
bSoluzione: Per semplicità assumiamo due valori costanti B1 < B2 = B1 + ∆B nei due semispazi x > 0 e
x < 0 La particella si muove lungo semicirconferenze di raggi ri = mv/qBi con frequenza ω = qB/m (dove
B ∼ (B1 + B2 )/2). Ad ogni giro si sposta di ∆y = 2(r2 − r1 ) e quindi drifta con velocità
vydrift ≈
2
ω
v⊥ B2 − B1
v⊥ r B2 − B1
mv⊥
2(r2 − r1 ) ≈
=
≈
∇B
2π
π
B
π B
r
qB
In generale la direzione del drift è B × ∇B 2 ; il verso dipende da q ma la velocità di drift no.
• • • • • • • • • • • • • • • • • •
In generale ∇ × B = 0 implica che nè il modulo nè la direzione di B sono costanti, per cui si ha l’effetto
combinato dei due drift discussi in questo esercizio e nel precedente. Nel caso del campo magnetico di un filo,
Bθ ∝ 1/ρ entrambi gli effetti producono un drift lungo z, con velocità
v drift =
2
/2
v||2 + v⊥
ωB ρ
ρ̂ × B̂
dove il versore ẑ è stato scritto in modo complicato in modo che la formula sia valida in generale per il moto
con a ρ in un B 2-dimensionale (i.e. nel problema concreto B non dipende da z).
Esercizio 102: Carica in B(t) uniforme
Studiare il moto di una particella di carica q libera di ruotare nel piano xy in un campo magnetico Bz (t) che
viene lentamente variato da B0 a B1 .
bSoluzione: Assumiamo che B vari di poco in un giro: quindi le orbite sono approssimabili come circonferenze.
Il loro raggio a è determinato da mv 2 /a = qvB. All’inizio la carica ha velocità v0 e quindi gira con raggio
a0 = mv0 /qB0 . Alla fine avrà velocità v1 (da calcolare) e quindi girerà con raggio a1 = mv1 /qB1 .
La forza magnetica non accelera le particelle, ma un campo magnetico Bz (t) induce un campo elettrico
Eθ (t). Dall’equazione ∇ × E = −Ḃ segue 2πrEθ = Φ̇ = πr2 Ḃ. Denotando con v il modulo della velocità, la
particella viene accelerata secondo
mv̇ ' qEθ =
qa(t)
v Ḃz
Ḃz = m
2
2 Bz
cioè
d v2
' 0.
dt Bz
(7.2)
Quindi v 2 /Bz (o equivalentemente il flusso Φ ∝ Bz a2 , o equivalentemente il dipolo magnetico µ = qav/2)
sono invarianti adiabatici. La parola ‘adiabatico’ ed il ' ricordano che il tutto è vero solo nel limite di campo
lentamente variabile.
• • • • • • • • • • • • • • • • • •
Riotteniamo lo stesso risultato in modo alternativo, approssimando la carica ruotante come un dipolo
magnetico. Questa approssimazione è possibile solo se la carica ruota veloce, cioè stiamo anche qui facendo
l’approssimazione adiabatica. Una carica produce una corrente i = qω/2π e quindi un momento magnetico
µ = πa2 i =
q 2
q
a ω=
L
2
2m
dove
L = mav = ma2 ω
è il momento angolare.
La forza su di un dipolo magnetico dipende solo dal campo magnetico, secondo
L̇ = µ × B =
e
L × B.
2m
56
Capitolo 7. Moto in campo magnetico esterno
Una particella libera in un campo magnetico ruota sempre attorno ad esso, quindi L e B sono sempre paralleli,
quindi L rimane costante in accordo con i nostri risultati precedenti.
• • • • • • • • • • • • • • • • • •
Nel limite opposto, in cui µ non dipende da B, l’equazione del moto dice che L gira attorno a B con
frequenza ωL = eB/2m. In un linguaggio meno sofisticato, questo accade perchè il campo magnetico fa girare
più veloce la carica, con frequenza ω + ωL (o più lenta: il segno sarà tale che la variazione di moto genera un
campo magnetico che si oppone alla variazione del B esterno). Studiamo un caso particolare importante per la
teoria del magnetismo nella materia.
Esercizio 103: Atomo in B(t) uniforme
Studiare come reagisce un atomo di idrogeno quando viene acceso lentamente un campo magnetico esterno.
bSoluzione: Il problema è analogo a quello precedente, con la differenza che l’elettrone gira risentendo anche
della forza di Coulomb, non solo del campo magnetico esterno. Possiamo quindi utilizzare ancora l’eq. (7.2),
tenendo conto che ora il legame fra a(t) e v(t) è dato da mv 2 /a = −evB + e2 /4π0 a2 . Per semplicità, assumiamo che il campo magnetico dia una piccola correzione ad v e quindi al raggio dell’orbita a, che in prima
approssimazione rimane costante: questa è l’unica differenza rispetto al caso precedente. Quindi
ea
Ḃ
→
ω(t) ≡
2
dove ωL = eB/2m è detta frequenza di Larmour, che stiamo
E.g. per B = Tesla viene ωL = 0.9 1011 Hz, mentre gli atomi
mv̇ =
v(t)
= ω(0) + ωL (t)
a
assumendo essere una correzione piccola, ωL ω.
hanno frequenze tipiche ω ∼ 1016 Hz.
Esercizio 104: Carica in B non uniforme
Calcolare in approssimazione adiabatica la velocità di drift parallela al campo magnetico.
bSoluzione: Nei vari casi precedentemente studiati abbiamo trovato ‘nuovi’ effetti: drifts ortogonali al campo
magnetico. Ora studiamo in dettaglio il ‘vecchio’ moto lungo le linee del campo magnetico, che in molti casi
rimane il moto principale. Se il campo magnetico è uniforme, la carica procede con v|| costante. Vogliamo
vedere come procede in generale. Il risultato è:
2
v 2 = v||2 + v⊥
= costante
e
2
v⊥
/B = invariante adiabatico
dove la prima costante del moto è l’energia (ovvio!). L’esistenza di un invariante adiabatico la si può capire
intuitivamente dai problemi precedenti, dove la stessa espressione era invariante adiabatico. Nel caso precedente
B dipendeva dal tempo (producendo un E), mentre ora dipende dallo spazio: ma dal punto di vista di una
particella che cammina è come se B dipendesse dal tempo.
2
Avendo capito intuitivamente perchè v⊥
/B è invariante adiabatico, vogliamo dimostrarlo in modo rigoroso.
Mettendo l’asse z lungo la linea di B si ha che una variazione di Bz (z) è accompagnata da un Bρ = −ρBz0 (z)/2,
in quanto questo è necessario per avere ∇ · B = 0. Per verificarlo basta usare la divergenza in coordinate
cilinidriche, o imporre che il flusso di B su di un appropriato cilindretto sia zero.
La componente Bρ contribuisce all’equazione del moto della carica q lungo z:
z̈ = −
qvθ Bρ
v2
' ⊥ Bz0
m
2Bz
2
avendo usato vθ ' v⊥ e ρ ' mv⊥0 /qBz0 in approssimazione adiabatica. Verifichiamo che v⊥
/Bz è costante:
2
d v⊥
Ḃz 2
1 d 2
B0 2
2ż z̈
ż
v2 B 0
2
= − v⊥
−
(v − v=
) = − z żv⊥
−
= −2 (z̈ + ⊥ z ) = 0
dt Bz
Bz
Bz dt
Bz
Bz
Bz
2 Bz
in quanto l’ultima espressione coincide con l’equazione del moto.
• • • • • • • • • • • • • • • • • •
Una conseguenza è l’intrappolamento magnetico: se Bz (z) aumenta ad un certo punto v|| = 0 e la particella
deve tornare indietro. Nel campo magnetico terrestre questo fenomeno di intrappolamento tiene particelle
cariche nelle fascia di Van Allen.
Capitolo 7. Moto in campo magnetico esterno
57
Esercizio 105: Carica in quadrupolo magnetico
Studiare il moto di una carica q con velocità v quasi parallela all’asse z in un campo magnetico B = ∇(bxy).
bSoluzione: Avendolo scritto come gradiente di un ‘potenziale magnetico’ Vm il campo magnetico soddisfa
automaticamente alla IV equazione di Maxwell nel vuoto, ∇ × B = 0. Siccome ∇2 Vm = 0 soddisfa anche a
∇ · B = 0. Sperimentalmente è ottenibile nel seguente modo. Il ferro ha µ ∼ 1000 e quindi le linee di B
escono praticamente perpendicolari al materiale. Quindi lo si costruisce in modo che segua la forma delle linee
equipotenziali di xy. [Spostare nel capitolo 10]. Vale
Bx = by
By = bx,
Bz = 0
L’equazione del moto è
mẍ = −qvbx,
mÿ = +qvby
Assumendo che la particella si muova circa lungo l’asse z e venga deflessa poco possiamo approssimare d/dt =
v d/dz ottenendo
d2 x
d2 y
qb
2
=
−k
x,
= +k 2 y
dove
k2 =
2
2
dz
dz
mv
la cui soluzione è
1
1
x(z) = x0 cos kz + x00 sin kz,
y(z) = y0 cosh kz + y00 sinh kz.
k
k
Quindi un fascio di particelle viene focalizzato lungo l’asse x e defocalizzato lungo l’asse y. Più precisamente,
assumendo che ` sia la lunghezza della zona dove B 6= 0 un fascio avente x00 = 0 passa per x(z) = 0 a
z − ` = cot(k`)/k: cioè il campo magnetico si comporta come una lente di distanza focale f = 1/k sin k`. (La
differenza fra cot e 1/ sin è questione di definizione dovuta allo spessore finito della lente: il piano principale sta
a distanza −(1 − cos k`)/k sin k` dall’uscita del quadrupolo).
Esercizio 106: Carica in quadrupoli magnetici
Studiare cosa succede quando un fascio di particelle attraversa due quadrupoli magnetici, ruotati di 90◦ l’uno
rispetto all’altro.
bSoluzione: Abbiamo visto che un quadrupolo magnetico focalizza un fascio lungo una direzione ma defocalizza
lungo l’altra. Il segno diverso è sgradito ma inevitabile, perchè dovuto a come deve essere fatto un campo
magnetico nel vuoto. In molti esperimenti è invece necessario focalizzare un fascio lungo entrambi gli assi, in
modo da concentrarlo. Infatti, incrociando due fasci, il numero di urti fra particelle aumenta se i fasci sono
stretti, analogamente a come il numero di incidenti aumenta quando una strada diventa più stretta.
Scopo di questo ‘esercizio’ [Christofilos, 1950; Courant et al., 1952] è mostrare che mettendo due quadrupoli
magnetici ruotati di 90◦ uno dopo l’altro si riesce a focalizzare in entrambe le direzioni.
Conviene riscrivere l’effetto di un singolo quadrupolo usando la formulazione matriciale dell’ottica geometrica. In tale formulazione si studia l’evoluzione con z di (x, x0 ) e di (y, y 0 ). Cioè x è la distanza dall’asse ottico,
ed x0 è l’inclinazione rispetto all’asse ottico di un raggio. Nel caso dell’ottica si studiando raggi di luce, qui fasci
di particelle. Attraversando un magnete quadrupolare di lunghezza ` si ha
x(`)
x(0)
y(`)
y(0)
= Mx
= My
x0 (`)
x0 (0)
y 0 (`)
y 0 (0)
dove le matrici di trasferimento sono
cos k`
sin(k`)/k
Mx =
,
−k sin k`
cos k`
My =
cosh k` sinh(k`)/k
k sinh k`
cosh k`
Per k → 0 si ritrova lo spazio vuoto.
La formulazione matriciale dell’ottica è conveniente perchè per studiare l’effetto combinato di sistemi diversi
basta moltiplicare le loro matrici. Quindi attraversando due quadrupoli ruotati di 90◦ a distanza d fra loro si
58
Capitolo 7. Moto in campo magnetico esterno
x
1
2
3
4
a
z
b
Figura 7.2: Lente convergente.
ha, assumendo per semplicità ` d, 1/k (lente sottile), le matrici di propagazione per raggi inclinati lungo x e
lungo y sono, rispettivamente date dai prodotti Mx · M (d) · My e My · M (d) · Mx , che valgono
1
`∼0
1 d
1
`∼0
1 + ··· d + ···
·
·
'
−d`2 k 4 1 − · · ·
±k 2 `
1
0 1
±k 2 `
1
Si ha focalizzazione in quanto l’elemento 21 è negativo.
Senza assumere d ` si troverebbe
(Mx · M (d) · My )21 ' −k 4 `2 (3d + 2`)/3
(My · M (d) · Mx )21 ' −k 4 `2 (` − 3d)/3
cioè che serve d > `/3, e che la lente è astigmatica (fx 6= fy ).
Esercizio 107: Ottica geometrica matriciale
Viene riassunto l’utilizzo delle matrici come formalismo conveniente per problemi di ottica geometrica.
bSoluzione: L’ottica geometrica studia come si propagano raggi di luce. Un raggio che si propaga lungo ‘l’asse
ottico’ z nel piano xz è descritto da due quantità: la posizione x(z) e l’inclinazione x0 (z).
Attraversando una distanza ` vuota si ha ovviamente
x(`)
x(0)
1 `
= M (`)
M (`) =
x0 (`)
x0 (0)
0 1
Attraversando una lente sottile di focale f cambia solo l’inclinazione del raggio. Esso varia di una quantità
costante, chiamata ‘distanza focale inversa’ 1/f :
x
x
1
0
= Mlente
M=
x0 dopo
x0 prima
−1/f 1
Per il resto serve solo sapere come moltiplicare matrici: se X4 = M34 X3 e X3 = M23 X2 e X2 = M12 X1 ,
allora X4 = M34 M23 M12 cioè le matrici vengono moltiplicate ‘in ordine inverso’: la prima (M34 ) è quella che
corrisponde all’ultimo passo.Consideriamo ad esempio il sistema standard in fig. 7.2. La matrice di propagazione
da 1 a 4 è
1 b
1
0
1 a
1 − b/f a + b − ab/f
M14 = M (b) · Mlente · M (a) =
·
·
=
0 1
−1/f 0
0 1
−1/f
1 − a/f
I raggi disegnati convergono in un punto unico se l’elemento 12 della matrice totale vale zero, cioè se 1/a + 1/b =
1/f . Quando questa relazione è soddisfatta si ha xC = (1 − b/f )xA : quindi 1 − b/f è il fattore di ingrandimento
della lente. Se esso è negativo, l’immagine viene rovesciata.
Quindi l’ottica matriciale consente di riprodurre i risultati noti per una lente. L’ottica matriciale è utile
perchè consente di ottenere risultati non noti per sistemi arbitrari di lenti: basta moltiplicare le matrici delle
singole componenti.
Capitolo 8
Induzione magnetica
Esercizio 108: Circuito allungato
Un circuito rettangolare di reststenza R e lati ` fisso ed x variabile è immerso in un campo magnetico B
ortogonale. La lunghezza x viene variata x = vt. Calcolare la corrente indotta, la forza esterna F e la potenza
W necessarie, e la potenza dissipata nella resistenza.
bSoluzione:
a. Φ = BLvt e quindi I = E/R = BLv/R.
b. Fext = BIL = B 2 L2 v/R in direzione opposta al moto.
c. Wext = F v.
d. W = I 2 R = Wext . Tutto il lavoro fatto viene dissipato tramite R.
Esercizio 109: Cilindro ruotante
Un cilindro conduttore viene fatto ruotare a velocità angolare ω parallela ad un campo magnetico costante B.
Un contatto strisciante di resistenza R connette il bordo con il centro. Quanta corrente vi passa?
bSoluzione:
Esercizio 110: Generatore
Come si produce la ‘corrente’ ?
bSoluzione: Per trasformare energia meccanica in energia elettrica, si fa ruotare una spira in un campo
magnetico costante (generato e.g. da un magnete) o far ruotare un campo magnetico attorno ad una spira
ferma. In entrambi i casi vale E = −Φ̇, ma questa formula segue da due diverse equazioni di base.
Nel caso in cui la spira è ferma segue direttamente dalla seconda equazione di Maxwell. Nel caso in cui il
campo magnetico è fermo, segue dalla forza di Lorentz. Per vederlo mettiamo B lungo z e facciamo girare una
spira di lati ∆x e ∆y attorno all’asse x:
Bz
x
θ
59
y
60
Capitolo 8. Induzione magnetica
La forza magnetica ha una componente lungo il circuito solo sui lati esterni, i cui punti hanno coordinata
d
y = 12 ∆y cos θ(t) e quindi Fx /e = (v × B)x = vy Bz che integrata vale E = 22 dt
∆y · ∆x sin θ = −Φ̇. (la fem
preduce una vx per cui per far girare la spira occorre compiere lavoro uguale all’energia elettrica prodotta).
E = −Φ̇ non vale sempre:
• Ondulatore
• Macchina unipolare: una sfera magnetizzata verticalmente contiene
un B ∝ M viene fatta ruotare. Fra
R
polo e contratto strisciante su equatore si crea una fem E ∝ drωrB, calcolabile anche come variazione
del flusso di un circuito ruotante
Esercizio 111: Generatore in orbita
Il campo magnetico terrestre all’equatore vale B0 = 0.5 G. Un satellite ruota con v = 7 km/ s su un orbita di
raggio R = 8000 km. Dal satellite pende un filo lungo L = 200 m. Calcolare la ddp ai suoi capi.
R
bSoluzione: vB = Lv(R0 /R)3 ≈ 35 V. Facendo oscillare la lunghezza del filo si genera una corrente alternata.
Esercizio 112: Trasformatore
Discutere i trasformatori.
bSoluzione: Per trasmettere una data potenza W = V I da una centrale elettrica al luogo di consumo conviene
utilizzare un grosso V ed una piccola I, in modo da ridurre la potenza RI 2 dissipata per effetto Joule lungo la
linea di trasmissione. Ad esempio per trasmettere 10 kW a 100 V serve I = 100 A; se invece la trasmetto a 10
kV basta I = 1 A, riducendo di un fattore 104 la potenza dissipata nel filo.
Per motivi di sicurezza l’energia elettrica va però venduta a basso voltaggio. È quindi essenziale che un
trasformatore sappia convertire il voltaggio senza perdite. Un trasformatore si basa sull’induzione magnetica,
che richiede correnti variabili. Per questo motivo le correnti alternate sono più usate di quelle continue.
Le equazioni di base di un trasformatore sono
V1 − Φ̇1 = V1 − L1 I˙1 − M I˙2 = R1 I1
− Φ̇2 = −L2 I˙2 − M I˙1 = R2 I2
e la quantità interessante da calcolare è il fattore di conversione V2 /V1 = R2 I2 /V1 .
Nel limite ideale R1 = 0 ed R2 → ∞ viene I2 → 0 e quindi V2 /V1 = M/L1 = N2 /N1 . Se I2 6= 0 il secondo
circuito reagisce sul primo. Per calcolare come assumo una fem alternata V̂1 = V1 eiωt ed uso il metodo delle
impedenze
V1 = (R1 + iωL1 )I1 + iωM I2
(R2 + iωL2 )I2 + iωM I1 = 0
da cui
V2
−iωM R2
=
V1
(R1 + iωL1 )(R2 + iωL2 ) + ω 2 M 2
W2
R2
M 2 ω2
=
W1
R2 + IωL2 (R1 + iωL1 )(R2 + iωL2 ) + ω 2 M 2
[i moduli quadri andrebbero presi con cura] La potenza trasmessa vale 1 se R1,2 → 0 e se il termine ω 2 (M 2 −
L1 L2 ) al denominatore vale zero. Questo
√ corrisponde ad avere un trasformatore che non disperde flusso. Il
massimo valore possibile di M è M = L1 L2 .
Assumendo accoppiamento perfetto e che R1 sia trascurabile
r
V2
L2
N2
=−
=−
V1
L1
N1
È chiaramente importante avere ωL R, altrimenti W2 /W1 ∼ L2 /R2 . Nel vuoto si ha L ∼ N µ0 d dove d è la
dimensione del trasformatore ed N è il numero di spire. Per ω ∼ 100 Hz e d ∼ m viene ωL ∼ N 10−7 Ω. Il ferro
migliora la situazione di un fattore µ/µ0 ∼ 104 .
Capitolo 8. Induzione magnetica
61
Esercizio 113: Trasformatore con due spire
Non si usa il ferro. Due spire concentriche, di raggi A ed a.
bSoluzione: L ∼ µ0 a, MAa ' µ0 a2 /2πA. MaA = MAa in modo non ovvio.
Esercizio 114: Induzione
Due circuiti hanno autoinduttanze L1 ed L2 e M . Studiare cosa succede quando si connettono i fili in vario
modo.
bSoluzione: In generale
E1 = −L1 I˙1 − M I˙2
E2 = −L2 I˙2 − M I˙1
con M > 0, avendo scelto stesse direzioni per I ed E. (E = −Φ̇, posso fare lo stesso esercizio con Φ).
• Se li attacco in serie dritti E = E1 + E2 e I = I1 = I2 . Quindi ottengo un unico circuito con induttanza
L = L1 + L2 + 2M .
• Se li attacco in serie a rovescio E = E1 − E2 e I = I1 = −I2 . Quindi ottengo un unico circuito con
induttanza L = L1 + L2 − 2M . Deve essere L > 0 e quindi M < (L1 + L2 )/2.
• Se li attacco in parallelo dritti E = E1 = E2 e quindi
L2 − M
L1 − M
L1 + L2 − 2M
,
I2 = −E
,
I1 + I2 = I = −EL
L=
L1 L2 − M 2
L1 L2 − M 2
L1 L2 − M 2
√
√
da cui M < L1 L2 , che è una condizione più stringente. Notare che è possibile avere |M | L1 L2 ,
quando M < 0.
I1 = −E
• Se li attacco in parallelo a rovescio viene L = (L1 + L2 + 2M )/(L1 L2 − M 2 ).
• Se M = 0 le induttanze si combinano come le resistenze.
Esercizio 115: Trapano
Inventare un trapano
bSoluzione: Iniziamo da un esercizio ideale: supponiamo di avere un campo magnetico con simmetria cilindrica
Br = B0 (r0 /r). Verifica ∇ · B = 0 per r 6= r0 .
Una spira di lunghezza `, nella quale viene fatta passare una corrente continua i, è libera di ruotare radialmente. La forza di Lorentz sul lato superiore vale F+ = `iB(r+ ) e tende a far girare la spira; tuttavia sul
lato inferiore si esercita una forza F− = −(r+ /r− )F+ tale che il momento totale è zero. Se a uno piace complicarsi la vita e pensare in termini di ‘flusso tagliato’ questo accade perchè il flusso tagliato dal filo superiore
(Φ = B(r+ )`r+ ∆θ) è uguale ed opposto a quello tagliato da filo inferiore.
Quindi, per ottenere un trapano funzionante, si elimina il filo inferiore rimpiazzandolo con il contatto mobile
disegnato in figura
F
F
62
Capitolo 8. Induzione magnetica
Studiamo adesso come si fa ad ottenere il campo magnetico assunto. Non disponendo di un filo di monopoli
magnetici lo si può costruire con un magnete non su tutta la circonferenza ma solo su di una parte. Questo è
ottenibile usando un magnete permanente. Nella parte di sotto il campo magnetico ‘tira dritto’ e produce un
momento che tende a frenare la spira. Per evitare questo si può invertire il segno della corrente i quando fa il
mezzo giro nella parte inferiore, utilizzando quindi una corrente alternata di frequenza uguale alla frequenza di
rotazione. Questa non è una buona soluzione, in quanto una presa di corrente da invece una corrente alternata
di frequenza fissa, 60 Hz. Conviene quindi avvolgere un solenoide attorno ai magneti in modo da fargli creare un
campo magnetico nella direzione giusta quando ci passa la spira, in modo che venga sempre accelerata. Siccome
far percorrere ad un magnete il suo ciclo di isteresi costa energia (a seconda di quanto è largo il ciclo di isteresi),
si mettono due o più spire con correnti circolanti in senso opposto in modo da usare il campo magnetico quando
punta in entrambe le direzioni.
• • • • • • • • • • • • • • • • • •
La rotazione del filo produce una vθ e quindi una forza di Lorentz lungo il filo, e quindi una fem indotta
E = −qr+ Bω` che ad alta frequenza riduce la fem iniziale frenando la rotazione della spira Poi la spira produce
a sua volta un campo magnetico: si ha anche un effetto di auto-induzione
E = −qBr+ `ω = Ri + L di/dt
I ω̇ = Br+ `i
da cui, senza fem esterna,
L
dando oscillazioni smorzate.
(Br+ `)2
dω
d2 ω
+R
+q
ω=0
2
dt
dt
I
Capitolo 9
Forze magnetiche fra circuiti
Hanno importanza soprattutto come esercizi in compiti.
Esercizio 116: Due circuiti lunghi
Si considerino i due circuiti rigidi in figura, con L d. a) Determinare il coefficiente di mutua induzione. Due generatori di
d
corrente mantengono le correnti I1 ed I2 costanti. Le resistend
ze elettriche sono trascurabili. b) Determinare le forze esterne
necessarie a mantenere i circuiti fermi come in figura. c) Lasciand
do libero uno dei circuiti di muovesi, determinare la sua energia
cinetica a distanza infinita.
I1
I2
bSoluzione:
a) Dominano le forze fra i fili lunghi. Chiamandoli 1,2,3,4 (dall’altro in basso)
µ0 L I1 I2 1 2
µ0 L I1 I2
F = F14 + (F13 + F24 ) + F23 =
− +1 =
2π d 3 2
2π 3d
La forza è repulsiva.
b) M < 0 in quanto i due circuiti sono ‘esterni’: uno prende il flusso dell’altro nella regione in cui B ‘torna
indietro’. Lungo il piano che contiene i due circuiti, a distanza x dal filo 2
Z 2d
µ0 I1
1
1
µ0 L 4
B=
− +
,
Φ(B) = L
B dx = M I1
M =−
ln
2π
x x+d
2π
3
d
c) Sfruttiamo la conservazione dell’energia: UI + L = UF + K. K è l’energia cinetica che si vuole calcolare.
UF ed UI sono le energie magnetiche, in generale date da U = Lij Ii Ij . Alla fine L12 = M = 0 mentre
all’inizio M ha il valore calcolato al punto b). L è il lavoro fatto dai generatori per mantenere costanti
le correnti
L =
2 Z
X
i=1
Ii Ei dt =
2 Z
X
i=1
2
Ii
X
dΦi
dt =
Ii [ΦiF − ΦiI ] = I1 [0 − M I2 ] + I2 [0 − M I1 ] = −2M I1 I2
dt
i=1
Quindi K = (UF − UI ) + L = (1 − 2)M I1 I2 = −M I1 I2 > 0, in accordo con il fatto che la forza è repulsiva.
c0 ) Si può anche procedere in modo meno intelligente intergando la forza ricalcolata a distanza r > d generica:
µ0 L
1
2
1
µ0 L
2d2
F = F14 + (F13 + F24 ) + F23 =
I1 I2
−
+
=
I1 I2
2π
2d + r d + r
r
2π
r(d + r)(2d + r)
A
R ∞grande distanza r d deve ridursi alla forza fra due dipoli magnetici µi = LdIi . Integrando K =
F dr si ritrova il risultato precedente
d
63
64
Capitolo 9. Forze magnetiche fra circuiti
Esercizio 117: Rotazione di due spire circolari
[Dal compito del 19/9/03] Due spire conduttrici circolari coplanari diposte nel piano xy hanno raggi diversi in
modo trascurabile, sezione del filo trascurabile, resistenza R e coefficiente di autoinduzione L. La spira interna
è libera di ruotare attorno all’asse x e inizialmente percorsa da una corrente i0 , mentre la spira esterna è fissa
e collegata ad un generatore ideale di corrente che eroga la corrente I. Le correnti scorrono nello stesso verso.
All’istante t = 0 ed in un tempo trascurabile rispetto al tempo caratteristico del sistema, la spira interna viene
ruotata di 90◦ e fermata.
a) Si calcoli la relazione tra il coefficiente di autoinduzione L delle spire e quello di mutua induzione M sia
al tempo t < 0 (quando le spire sono coplanari) sia a t > 0 (dopo aver effettuato la rotazione).
b) Supponendo trascurabile la caduta ohmica nella spira interna durante la rotazione, si calcoli la corrente
che circola in essa all’istante dell’arresto.
c) Si calcoli il lavoro fornito dal generatore di corrente durante la rotazione.
d) Sapendo che l’energia dissipata nella spira interna per effetto Joule durante il tempo transitorio di scarica
successivo alla rotazione della spira è LJ , si determini L.
e) Si calcoli il lavoro meccanico Lmecc speso per far ruotare la spira.
bSoluzione:
a) All’inizio M0 = L. Dopo la rotazione, M = M1 = 0, come si può vedere da considerazioni di simmetria.
b) Durante la rotazione la f.e.m. ai capi della spira interna è: E = −dΦ/dt = Ri. Siccome la rotazione
avviene molto velocemente possiamo trascurare Ri, e quindi Φ = M I + Li rimane costante. Quindi
Li1 = Li0 + M0 I cioè i1 = i0 + I.
c) La conservazione dell’energia consente di calcolare il lavoro totale Lgen + Lmecc = U1 − U0 , ma non le
singole componenti.
Il lavoro compiuto dal generatore di corrente si ottiene facilmente integrando la potenza dissipata sia a
causa della presenza della resistenza, sia dovuta alla forza elettromotrice indotta:
Lgen =
Z
∆t
RI 2 − EI dt = RI 2 ∆t + I∆Φ = RI 2 ∆t + I∆(M i)
0
Per ∆t → 0 la dissipazione ohmica diventa trascurabile. Essendo ∆(M i) = −M0 i0 = −Li0 , risulta che
Lgen = −Li0 I. Verifica del segno: il sistema da solo tenderebbe ad aumentare I per opporsi alla variazione
di Φ, che decresce. Siccome invece I viene mantenuta costante, il generatore riceve energia.
d) Durante il transiente successivo alla rotazione l’energia immagazzinata nell’induttanza si dissipa per effetto
Joule, quindi
L
L
LJ = i21 = (i0 + I)2 .
2
2
e) Trascurando la dissipazione Joule durante la veloce rotazione, dalla conservazione dell’energia segue che
L 2
L
L
(i + I 2 ) − (i20 + I 2 ) − M0 Ii0 + LIi0 = I(I + 2i0 )
2 1
2
2
R
Calcolare il lavoro meccanico come integrale Lmecc = M dθ del momento delle forze sarebbe più
complicato, ma consente di vedere subito che vale zero per I = 0.
Lmecc = U1 − U0 − Lgen =
Capitolo 9. Forze magnetiche fra circuiti
65
Esercizio 118: Una spira ed un dipolo
[Dal compito del 6/2/04] Un dipolo magnetico con momento magnetico
µ
~ è posto al centro di una spira circolare di raggio a. Il sistema di assi
cartesiani è fissato in modo che l’origine sia nel centro della spira e gli assi
x e y nel suo piano. Il dipolo viene fatto ruotare con velocità angolare
costante ω nel piano x, z (vedi fig. 1)
θ=ω(τ)
z
a) Si determini la corrente che scorre nella spira sapendo che essa ha una resistenza elettrica R (si trascuri il suo coefficiente di
autoinduzione).
a
y
b) Si determini il momento meccanico esterno necessario a mantenere
il dipolo in rotazione.
x
c) Si mostri che la potenza meccanica media fornita è eguale alla
potenza dissipata per effetto Joule.
Fig.1
La spira sia connessa a un generatore di corrente costante I.
d) Se il dipolo ha massa m e si può muovere lungo z, si determini l’orientamento relativo della corrente nella
spira e del dipolo perche’ la forza lungo z sia di richiamo attorno al punto z = 0 e la frequenza delle sue
piccole oscillazioni.
bSoluzione:
a) Il dipolo magnetico è equivalente ad una spiretta di superficie s percorsa da corrente i tale che µ = si. La
corrente nella spira grossa dipende dalla variazione del flusso di mutua induzione della spiretta sulla spira.
È complicato calcolarlo, mentre è facile calcolare il coefficiente di mutua induzione M (θ) dalla spira sulla
spiretta. Per via di un teorema generale i due coefficienti sono uguali. Ricordando che il campo magnetico
nel centro della spira vale Bz = µ0 I/2a abbiamo
Φs (θ)
µ0 s
=
cos θ.
I
2a
Quindi il flusso indotto dalla spiretta (dipolo) sulla spira è
M (θ) =
µ0 µ
µ0 s · cos θ
i=
cos ωt
2a
2a
e non dipende dalla superficie s arbitraria della spiretta. La corrente I è data da
ΦS (θ) = i · M (θ) =
I=
E(t)
1 ∂ΦS (t)
1 µ0 µω
=−
=
sin ωt
R
R ∂t
R 2a
b) Il momento meccanico esterno M deve essere opposto al momento delle forze dovuto all’interazione tra
dipolo e campo della spira, M = −µ × B, che ha solo componente
My = µx Bz = µBz sin ωt =
µ2 µ20 ω
sin2 ωt
4Ra2
c) La potenza W sviluppata dal momento delle forze esterne è
W = My ω =
µ2 µ20 ω 2
sin2 ωt
4Ra2
eguale alla dissipazione Joule WJ = E 2 /R.
d) Affinchè la forza sia di richiamo il potenziale U (z) = −µ · B deve essere minimo a z = 0, quindi µ e B
devono essere concordi. La forza è:
∂Bz
∂
µ0 Ia2
3 µ0 Ia2 z
3 µ0 Iz
=µ
=
−
'−
2
2
3/2
2
2
5/2
∂z
∂z 2(a + z )
2 (a + z )
2 a3
p
da cui la frequenza delle piccole oscillazioni, ω = 3µ0 I/2ma3 .
Fz = mz̈ = µ
66
Capitolo 9. Forze magnetiche fra circuiti
Esercizio 119: Monopolo magnetico
Studiare come reagisce una spira circolare di raggio a quando lungo il suo asse passa, a velocità v costante a)
un dipolo b) un monopolo magnetico.
bSoluzione: Entrambi inducono un flusso indotto e quindi una fem E = −Φ̇. Se la spira ha auto-induttanza
L trascurabile e resistenza R, misuro una corrente I = −Φ̇/R. Se invece la spira ha R trascurabile ed autoinduttanza L ho E = LI˙ e quindi misuro una corrente I = I0 − Φ(t)/L. In generale succedono cose più
complicate. Nel seguito studio questi due casi particolari.
a) Dipolo magnetico. Usiamo lo stesso trucco usato alla domanda a) dell’esercizio precedente. Il coefficiente di mutua induzione della spirona sulla spiretta (dipolo) è
M=
2(a2
µ0 a2
s cos θ,
+ z 2 )3/2
ΦD = iM =
2(a2
µ0 a2
µ cos θ
+ z 2 )3/2
dove θ è l’angolo che fa il dipolo rispetto alla spira e z = vt.
b) Monopolo magnetico. Un monopolo magnetico qM produrrebbe un campo magnetico a simmetria
sferica, Br = qM /r2 (la definizione della normalizzazione di qM è arbitraria). Non occorre fare l’integrale
per calcolare il flusso raccolto dalla spira, che è semplicemente dato da qM per l’angolo solido sotteso dalla
spira, 2π[1 − cos θsotteso ]. Quindi
z
√
ΦM = 2πqM 1 −
.
z 2 + a2
fem
flusso
La figura mostra l’andamento temporale di Φ e Φ̇ nei due casi.
tempo
tempo
Misurando una di queste quantità si ricava la velocità dell’oggetto che passa, e si distingue se è un dipolo
(linea continua blu) o un monopolo (linea tratteggiata rossa) Il monopolo magnetico produrrebbe un segnale
caratteristico, che nessuno ha mai visto.
Notare che i due risultati sono legati da dΦM /dz = ΦD /d: infatti due monopoli magnetici qM a distanza
d = µµ0 cos θ/4πqM piccola formano un dipolo magnetico µ. 1
Esercizio 120: Traslazione di due spire circolari
[Dal compito del 25/9/02] Una spira circolare conduttrice giace vincolata su di un piano parallelo a distanza z
1 Teorici ritengono che i monopoli debbano esistere, abbiano carica q
16
M ∼ h̄/2qE e forse massa M ∼ 10 mp , ma che dopo
l’inflazione ne siano rimasti troppo pochi per essere osservati. Un elettrone sull’anello aquista impulso
Z
pr =
qE Er dt =∼ qE
∆Φ
2qE qM
∼
2πa
a
Imponendo la quantizzazione del momento angolare L = apr = h̄ si trova qE qM = h̄/2. Torna ma non sono sicuro sia corretto,
bisogna capire il significato di ∆Φ 6= 0. Probabilmente per rendere consistenti le eq. di Maxwell modificate bisogna aggiungere una
JM
Capitolo 9. Forze magnetiche fra circuiti
67
dal piano di una seconda spira. La resistenza, il coefficiente di autoinduzione e quello di mutua induzione della
spira superiore siano R, L2 , M . Il coefficiente di autoinduzione della spira inferiore sia L1 . Si invii nella spira
inferiore una corrente
It/τ per t < τ
i1 (t) =
I
per t > τ
Si determini
a) La corrente i2 (t) nella spira superiore;
b) L’energia totale dissipata nella resistanza R
c) La carica Q che attraversa R
Nell’ipotesi che la spira superiore di massa m sia libera di muoversi verticalmente lungo il suo asse parallelo al
campo gravitazionale g, che R = 0 e che τ sia abbstanza piccolo in modo che M resti costante per 0 < t < τ si
determini, ripetendo l’immissione della corrente i1
d) la quota massima raggiunta dalla spira superiore, sapendo che in questa posizione il suo coefficiente di
mutua induzione è M 0 .
bSoluzione:
a) Per t < τ i2 è data dall’equazione
0
Τ2 ` Τ
di1
di2
L2
+M
+ Ri2 = 0,
dt
dt
i2 (0) = 0
Τ2 = Τ
risolta da
M I −t/τ2
(e
− 1)
Rτ
dove τ2 = L2 /R. Per t > τ si ha di1 /dt = 0 e quindi
i2 (t) = i2 (τ )e−(t−τ )/τ2 . La figura mostra il risultato per
diversi valori di τ /τ2 .
i2 (t) =
Τ2 = ¥
-IM/L 2
0
Τ
2Τ
b) L’energia dissipata vale
Z
W =
∞
dt R i22 (t) =
0
I 2M 2
[τ − τ2 + e−τ /τ2 τ2 ]
Rτ 2
Per τ2 τ si ha i2 (τ ) ' −IM/Rτ e W ' Rτ (M I/Rτ )2 → 0 dominata da 0 < t < τ .
Per τ2 τ si ha i2 (τ ) ' −IM/Rτ2 = −IM/L2 e W ' L2 i22 (τ )/2 dominata da τ < t <
∼ τ2 .
c) La carica totale che attraversa la resistenza è calcolabile come
Z ∞
Q=
dt i2 = −IM/R.
0
Il risultato è semplice e lo si può alternativamente ottenere senza nessun calcolo usando i2 = −E/R =
−Φ̇/R dove Φ(t) = M i1 (t) è il flusso del campo magnetico generato dalla spira inferiore su quella superiore.
Si ottiene quindi Q = (Φi − Φf )/R = −M I/R.
d) Per R = 0 si ha E = 0 = Φ̇ quindi Φ = M i1 +Li2 è costante. Siccome all’inizio Φ = 0 e siccome i1 raggiunge
velocemente il valore costante i1 (t > τ ) = I alla massima quota si ha i2 = −M 0 I/L2 . Imponiamo adesso
la conservazione dell’energia, tenendo conto che un generatore genera la corrente i1 . L’energia magnetica
vale
L1 2 L2 2
L1 2 M 2 (t) 2
1
I2
Umag =
i1 +
i2 + M (t)i1 i2 =
I −
I ,
∆Umag = (M 2 − M 02 )
2
2
2
2L2
2
L2
L’energia fornita dal generatore vale
Z
Z
d
I2
Lgen = i1 E1 dt = I
(M i2 + L1 i1 ) = (M 2 − M 02 )
dt
L2
3Τ
68
Capitolo 9. Forze magnetiche fra circuiti
L’energia gravitazionale vale ∆Ugrav = mg ∆z. Imponendo ∆Umag + ∆Ugrav = Lgen si ottiene
∆z =
M 2 − M 02 I 2
.
2
mgL2
Come al solito il generatore contribuisce −2 volte la variazione di Umag .
Capitolo 10
Campi magnetici nella materia
La densità di corrente totale J viene divisa in corrente libera J L (usualmente chiamata J , con abuso di
notazione) e corrente di magnetizzazione J M = ∇ × M . Per motivi storici si scrive B = µ0 (H + M ) = µH in
modo che la IV equazione di Maxwell diventa
∇ × H = J + JS
La magnetizzazione è approssimativamente legata ad H da M = χm H (e quindi µ = µ0 (1+χm )). La precessione
di Larmour dà il diamagnetismo, χ < 0. L’allineamento dei dipoli elementari dà il paramagnetismo, χ > 0.
Le condizioni di raccordo sul bordo fra due materiali sono: B⊥ continuo, H= continuo quando non ci sono
correnti.
Esercizio 121: Sbarra magnetizzata
Una cilindro di raggio r ha una magnetizzazione uniforme M lungo l’asse. Calcolare B. Se ne taglia una fetta
trasversale di spessore δ r. Calcolare B nel centro del buco.
bSoluzione: Senza buco: la magnetizzazione M descrive una corrente J m = ∇ × M . Nel caso del problema
JM è una densità di corrente superficiale Υθ = M , come si ottiene applicando Stokes ad un circuitino lungo
parallelo al materiale. Risolvendo ∇ × B = µ0 J M usando lo stesso circuitino si trova B = µ0 M .
In modo più matematico, ma anche più veloce, dobbiamo risolvere ∇ × B = µ0 ∇ × B cioè ∇ × H = 0.
Quindi mettendo H = 0 (niente correnti libere) si ottiene B = µ0 B. (I materiali ferromagnetici possono avere
avere M 6= 0 in assenza di un B esterno, realizzando in pratica questo esercizio).
Per tenere conto dell’aggiunta del buco possiamo vederlo come l’aggiunta di una spiretta di corrente i = −M δ,
che nel centro genera B = µ0 i/2r. Quindi B = µ0 M (1 − δ/2r). (B⊥ è continuo lungo il taglio, ma saperlo non
risolve il problema, in quanto tagliare il materiale modifica anche il campo all’interno del materiale, non solo
nel buco.).
Esercizio 122: Materiali ferromagnetici
N spire con corrente I sono avvolte attorno a materiali ferromagnetici (i.e. µ µ0 ) secondo le 3 geometrie
disegnate in figura. Trovare B nel piccolo traferro.
69
70
Capitolo 10. Campi magnetici nella materia
bSoluzione: Il ‘ferro’ ha µ = µr µ0 con µr ∼ 103 ÷ 105 (alcune leghe hanno µr più alto, alltre hanno cicli di
isteresi stretti e dissipano meno energia). Per risolvere problemi su materiali ferromagnetici occore sfruttare il
fatto che, in ottima approssimazione, questi intrappolano le linee del campo magnetico, tenendo costante il flusso
in
out
di B. All’uscita dalle imboccature il campo magnetico è circa ortogonale al materiale, in quanto H=
= H=
e
out
in
quindi B= = B= /µr ∼ 0.
a) Senza buco il campo H = B/µ è dato da 4LH = N I, cioè B = µN I/4L. Nel vuoto si avrebbe B ∼
µ0 N I/L: aggiungere il ferro ha lo stesso effetto di ridurre la dimensione: L → Lr = L/µr .
Aggiungendo un buco (o ‘traferro’) per un tratto d L le equazioni diventano (4L−d)B/µ+dB/µ0 = N I.
Notare che anche un piccolo buco d L può ridurre B in modo significativo: e.g. un d ∼ 4L/µr ∼ L/10000
lo dimezza circa. Il motivo è che la lunghezza ‘effettiva’ del circuito varia da L/µr a circa d + L/µr : il
tratto nel vuoto non è soppresso da 1/µr
b) Nel restringimento il flusso di B rimane costante. Se la superficie diventa 4 volte più piccola, B diventa
4 volte più intenso.
c) Invece di applicare il formalismo delle ‘riluttanze’ provo a ragionare. Nella regione con le spire è come il
caso a). Nelle altre regioni bisogna capire come si divide il flusso del campo magnetico alle biforcazioni.
La condizione è che la circuitazione di H lungo la ‘parte destra’ del circuito valga zero. Quindi B non
puó e.g. passare tutto nella sbarra intermedia, ma deve dividersi: 3/4 nella sbarra intermedia, ed 1/4 fa
il giro lungo. Quindi il campo magnetico nell’interferro è 4 volte più debole che nel caso a).
Esercizio 123: Due bacchette
Ci sono due bacchette di ferro. Una e’magnetizzata per la sua lunghezza, l’altra no. Come si può scoprire quale
delle due è magnetizzata?
bSoluzione: Chiedere all’Ill.mo Prof. [email protected].
Esercizio 124: Correnti parassite
[dal compito del 19/9/2003] Un lungo solenoide cilindrico è costituito da N spire per unità di lunghezza avvolte
su un nucleo di ferro di raggio R e lunghezza L R. Il ferro ha permeabilità magnetica µ e conducibilità
elettrica σ. Nelle spire si fa passare la corrente alternata I = I0 cos ωt.
a) Calcolare il campo magnetico all’interno del solenoide.
b) Calcolare il campo elettrico indotto all’interno del solenoide.
c) Si spieghi perchè il nucleo di ferro si riscalda e si calcoli la potenza dissipata per unità di lunghezza.
bSoluzione:
a) B = µH = µN I
b) Siccome ∇ × E = −Ḃ viene Eθ = 12 rµnI0 ω sin ωt
c) E induce una corrente J = σE e quindi una potenza dissipata JE per unità di volume.
In generale un campo magnetico B(t) induce un campo elettrico E(t) e quindi delle correnti J = σE che
dissipano energia e sono quindi dette ‘correnti parassite’. A frequenze alte bisogna tenere conto che B non è
più uniforme: questo viene fatto nel prossimo esercizio, nel quale si sceglie la geometria più semplice possibile
ma si deriva un risultato generale.
Capitolo 10. Campi magnetici nella materia
71
Esercizio 125: Correnti parassite
Un blocco di metallo ha permeabilità magnetica µ e conducibilità
σ. Viene applicato un campo magnetico oscillante di frequenza ω.
All’esterno del blocco B è parallelo alla superficie: By = B0 cos ωt.
Determinare la lunghezza di penetrazione del campo nel ferro e la
potenza dissipata dalla corrente indotta. (Si assuma che il blocco
occupi il semipiano x > 0 e si trascuri la corrente di spostamento).
Ez
x
By
bSoluzione: Il campo magnetico By (x, t) = µHy (x, t) genera un campo elettrico Ez (x, t) come dettato dalle
equazioni di Maxwell
∂x Hy = (∇ × H)z = Jz = σEz ,
∂x Ez = −(∇ × E)y = Ḃy = µḢy
Eliminando Ez , si trova che Hy soddisfa all’‘equazione di diffusione’
∂Hy
∂Hy
= µσ
2
∂x
∂t
Assumendo Hy ≡ h(x)e−iωt si riduce a h00 = −iωµσh,
risolta, nel semipiano x > 0 dove µ
p
±kx
2
2
h(x) = h(0)e
con k = −iδ /2 dove δ =
2/ωµσ viene chiamato ‘lunghezzapdi pelle’
differisce di un fattore 2 da quella usata in un altro esercizio). Quindi k = (i − 1) ωµσ/2.
soluzione che diverge per x → ∞, si ottiene che il campo penetra per una lunghezza dell’ordine
è costante, da
(la definizione
Eliminando la
di δ:
Hy (x) = ReHy (0)e(i−1)x/δ e−iωt = Hy (0)e−x/δ cos(x/δ − ωt)
La continuità di Hy al bordo x = 0 dà la condizione al contorno Hy (0) = H0 = B0 /µ0 . Il campo elettrico vale
H0 −x/δ
∂x Hy
=−
e
cos(x/δ − ωt) − sin(x/δ − ωt)
Ez =
σ
σδ
ed è piccolo a ‘basse’ frequenze: Ez /cHy ∼ 1/σδc 1. Esso genera correnti parassite J = σE. La potenza
media dissipata per unità di volume è
dW
µω 2 −2x/δ
= hJz Ez it = σhEz2 it =
H e
dV
2 0
Integrando su x > 0 si trova la potenza dissipata per unità di superficie:
Z ∞
dW
dP
µωδ B02
=
dx =
dS
dV
µ0 µ0
0
Se δ è piccolo, viene dissipata poca potenza.
Esercizio 126: In pratica
Avvolgo N spire su di un tubo di ferro, ed accendo la corrente (220 V, 50 Hz). Sul tubo appoggio un anello. Si
muove o la forza magnetica è trascurabile?
bSoluzione:
Parte III
Elettrodinamica
Capitolo 11
Corrente di spostamento
La conservazione della carica ρ̇ = −∇ · j aggiunge l’ultimo termine alle equazioni di Maxwell
∇ · E = ρ/0
∇ × E = −Ḃ
∇·B =0
∇ × B = µ0 j + µ0 0 Ė
Il nuovo termine j s ≡ 0 Ė viene chiamato ‘corrente di spostamento’. Una conseguenza è la presenza di onde
√
elettromagnetiche che viaggiano alla velocità della luce c = 1/ 0 µ0 , e.g.
Ez = sin(y − ct),
Bx = cos(y − ct),
tutto il resto = 0
Esercizio 127: Scarica di un filo
Un filo rettilineo va da z = 0 a z = ` di area S ha una densità uniforme di carica uniforme ρ(t) = ρ0 e−t/τ che
si scarica al capo z = `. Calcolare il campo magnetico.
bSoluzione: Iniziamo a calcolare la corrente j. L’equazione di continuità ∂jz /∂z = −ρ̇ equivale a ∂i/∂z = −λ̇
(dove λ = ρS) e quindi i(t, z) = −z λ̇. È come svuotare un canale d’acqua aprendo una chiusa: la corrente è
forte vicino alla chiusa e debole al capo opposto.
Siccome i dipende da z, se fosse ∇ × B = µ0 j la corrente concatenata dipenderebbe da quale superficie uno
sceglie nell’applicare il teorema di Stokes. La corrente di spostamento mette tutto a posto. La prima equazione
di Maxwell dice che Ez = zρ/0 (il campo elettrico vale zero a z = 0). La corrente di spostamento quindi vale
isz = S0 E˙z = +z λ̇. Non nasce nessun campo magnetico.
Esercizio 128: Piano con carica ondulata
a) Trovare il potenziale generato da un piano con densità di carica σ(x) = σ0 cos kx. b) Le cariche sono lasciate
libere di muoversi sul piano con conducibilità σ. Calcolare la loro evoluzione, la corrente, ed il campo magnetico
generato.
bSoluzione: a) Invece di integrare provo a risolvere ∇2 ϕ = −ρ/0 . Tento la soluzione
ϕ(x, z) = F (z) cos kx,
∇2 ϕ = [F 00 − k 2 F ] cos kx
e quindi F (z) = c+ ekz + c− e−kz . La soluzione con le condizioni al contorno ϕ(∞) = 0 e ∆E⊥ = −∆φ0 = σ/0 è
ϕ=−
σ0 −k|z|
e
cos kx
2k0
Per k → 0 (piano uniformemente carico) si riduce a ϕ ∼ cte − σ0 |z|/20 , in accordo con l’esercizio a pag. 8.
• • • • • • • • • • • • • • • • • •
Questo esercizio è più importante di quanto sembra, in quanto una qualunque distribuzione di cariche
può essere decomposta come somma di coseni con diversi k (trasformata di Fourier). Usando il principio di
sovrapposizione, abbiamo una soluzione per il problema generico.
73
74
Capitolo 11. Corrente di spostamento
Ad esempio una griglia di fili a distanza a avrà un trasformata di Fourier diversa da zero per k ∼ 1/a. Ad
una distanza |z| a i termini esponenziali diventano piccoli, e si ottiene il campo elettrico uniforme generato
dal ‘modo k = 0’, cioè dalla carica totale della griglia. Quindi è facile generare un campo elettrico uniforme.
L’illuminazione è descritta dalle stesse equazioni dell’elettrostatica. Quindi una griglia di tubi al neon
produce una illuminazione costante.
• • • • • • • • • • • • • • • • • •
b) Il campo elettrico Ex = −∂x ϕ = σ0 sin kx/20 genera una corrente Jx = σEx che redistribuisce le cariche.
La soluzione è σ(t) = σ0 e−t/τ cos kx, come si vede da
ρ̇ = −∇ · J = −σ∇ · E = −
σ
ρ
0
:
τ=
0
σ
Quindi anche il campo elettrico decade esponenzialmente. Non viene generato nessun campo magnetico in
quanto la corrente di spostamento compensa la corrente
J + 0 Ė = σE − 0
E
=0
τ
Vediamo quindi che questo accade in generale quando cariche sbilanciate sono libere di redistribuirsi secondo
J = σE. Il prossimo esercizio mostra che questa cancellazione è più generale.
Esercizio 129: Sfera radioattiva
Una sfera uniforme isolata di raggio a emette isotropicamente positroni da decadimento β: n → peν̄ con velocità
v0 . Il tempo di decadimento della radioattività è τ . Genera un campo magnetico?
bSoluzione: Il numero di neutroni liberi di decadere diminuisce come N = N0 e−t/τ e quindi la sfera acquista
una carica Q(t) = e[N0 − N ] > 0.
Iniziamo assumendo v costante e τ r/v, cioè un decadimento cosı̀ lento da essere approssimabile ad un
processo costante. I elettroni generano una corrente Jr (r) = (−e)(−Ṅ )/4πr2 , che non dipende da v. Il campo
magnetico è zero in quanto la corrente di spostamento cancella Jr : Jsr = 0 Ėr = −eṄ /4πr2 .
Per capire se questa cancellazione è un accidente del caso semplificato che abbiamo considerato, o se è invece
dovuta a qualche motivo più profondo, consideriamo casi progressivamente meno semplici.
Se il decadimento è veloce, τ ∼ r/v0 , in generale Jr (r, t) = eṄ (t − r/v)/4πr2 : il numero di elettroni che
attraversano una superficie a distanza r al tempo t dipende da quanti ne erano stati emessi al tempo t − r/v0 .
La cancellazione fra J e Js rimane perfetta in quanto Er (r) è determinato dalla carica totale dentro una sfera
di raggio r (che contiene la sfera radioattiva ed una nuvola di elettroni), eguale a e[N0 − N (t − r/v)]. Si ha
ancora J + Js = 0.
Il calcolo diventa ancora più complicato se si tiene in conto che v non è costante, in quanto la forza di
Coulomb rallenta i positroni. Il calcolo è complicato, e potrebbe essere fatto con una tecnica analoga a quella
utilizzara per studiate il diodo termoionico. È facile vedere che J e Js si cancellano ancora, in quanto entrambe
proporzionali a Ṅ calcolato a qualche istante ritardato. Il ritardo non è più r/v ma è dato da qualche formula
complicata che non è necessario calcolare.
Sebbene venga qualche i(r) = 4πr2 Jr (r) complicata si ha sempre B = 0: è quindi naturale domandarsi quale
sia il motivo generale. Una corrente a simmetria sferica non può generare un campo magnetico, che dovrebbe
avere solo una componente Br (r), ma questa dà rotore zero. Prendendo la divergenza della IV equazione di
Maxwell si ottiene ∇ · (J + J s ) = 0: la corrente di spostamento deve quindi cancellare ∇ · J , e l’unico modo
che ha di farlo è cancellare Jr .
Esercizio 130: Scarica di un condensatore
Un condensatore di area S = πa2 e distanza fra i piatti d a si scarica con costante tempo τ . Calcolare il
campo magnetico e la sua energia.
Capitolo 11. Corrente di spostamento
75
bSoluzione: La carica vale q(t) = q0 e−t/τ i.e. i = q̇ = −q/τ . Il campo elettrico vale 0 E = σ generando una
densità di ‘corrente di spostamento’ uniforme js = σ̇ = −q/Sτ . Notare che la corrente i entra in un piatto ed
esce dall’altro; la corrente totale di spostamento vale is = i. Quindi j s genera un campo magnetico ruotante
Bθ =
πr2 µ0 js
µ0 ri
=
2πr
2S
r<a
(A grande distanza r a il campo magnetico generato dalla corrente (che non si interrompe) i + is è circa
radiale Bθ = µ0 i/2πr). L’energia nel campo magnetico (usando i = −S0 /τ )
Z
1 a
di2 µ0
0 E 2
UB
a2 0 µ0
B2
= ( )2
=
UE = V
=
UB = dV
2µ0
16π
2
UE
8τ 2
8 cτ
è trascurabile a meno che a ∼ cτ , in generale a meno che E vari significativamente nel tempo che la luce impiega
ad attraversare l’apparato.
Esercizio 131: Condensatore in alternata
Studiare una capacità (costituita da due cerchi codnuttori a distanza d) alla quale viene applicata una differenza
di potenziale oscillante a frequenza ω. Si trascurino gli effetti di bordo e l’irraggiamento.
bSoluzione: Il campo elettrico oscillante Ez = E0 eiωt genera un campo magnetico lungo θ Bθ = B0 eiωt :
∇×B =
Ė
c2
:
B0 =
iωE0 πr2
iωE0 r
=
2
c 2πr
2c2
che a sua volta genera una correzione al campo elettrico E0 → E0 + E1
Z r
ω 2 r2
∇ × E = −Ḃ
:
E1 = iω
dr0 B0 (r0 ) = −E0 2
4c
0
avendo definito E0 come il campo a r = 0. A sua volta E1 genera un campo magnetico B0 → B0 + B1 1
Z
iω 1 r
−iω 3 r3
B1 = 2
E0
dr0 r0 E1 =
c r 0
16c4
che genera un campo elettrico
Z
r
E2 = iω
dr0 B1 (r0 ) =
0
ω4 r4
E0
64c4
che genera
B2 =
iω 5 r5
E0
64 · 6
E3 =
Quindi
iωt
Ez = (E0 + E1 + E2 + E3 + · · ·)e
iωt
= E0 e
−ω 6 r6
E0
64 · 62 c6
2n
∞
X
ωr (−1)n ωr
≡ E0 eiωt J0
2
(n!)
2c
c
n=0
La fig. 11.1a mostra J0 (x) confrontata con la sua espansione in serie ad ordine 0,2,4,6,8: J0 (x) = 1 − x2 /4 +
x4 /64 − x6 /2304 + · · ·. J0 (x) = 0 a x = ωr/c ' 2.4: J0 (2.4) ' 1 − 1.44 + 0.52 − 0.08 = 0.
• • • • • • • • • • • • • • • • • •
Riotteniamo la stessa cosa usando le eq. di Maxwell in forma differenziale. Assumendo Ez = Eeiωt e
Bθ = Beiωt dove E e B dipendono da e nelle equazioni di Maxwell in coordinate cilindriche si trova
−
1 Quando
∂E
= −iωB
∂r
1 ∂
iω
(rB) = 2 E
r ∂r
c
il gioco si fa duro conviene usare il rotore in coordinate cilindriche
(∇ × ẑ Ez (r))θ = −
∂Ez
,
∂r
(∇ × θ̂ Bθ (r))z =
1 ∂(rBθ )
.
r ∂r
76
Capitolo 11. Corrente di spostamento
2
1
0.8
1.8
|J0 (i1/2 x)|
J0 (x)
0.6
0.4
0.2
0
− 0.2
1.6
1.4
1.2
− 0.4
1
0
2
4
x
6
8
0
0.5
1
1.5
x
2
2.5
3
√
Figura 11.1: J0 (x) e |J0 ( ix)| (linee nere) confrontate con la loro espansione in serie attorno a x = 0 (linee
tratteggiate, che corrispondono ad includere mano a mano ordini successivi).
e quindi, sostituendo nella seconda il B preso dalla prima, ed usando come variabile x = ωr/c
r
∂ ∂E
r2 ω 2
(r
)=− 2 E
∂r ∂r
c
:
x(xE 0 )0 = −x2 E
:
E 00 +
E0
= −E
x
Se non ci fosse il secondo termine si avrebbe E 00 = −E 0 , la cui soluzione è una funzione speciale chiamata
“cos(x)” che si trova su tutti i computer (o tavole). Con il secondo termine, che è di tipo attrito, la soluzione è
chiamata “J0 (x)” che si trova su molti computer (o tavole) ed assomiglia un cos(x) che si smorza.
Si può fare la stessa cosa più in generale. Prendo il rot della II equazione di Maxwell
−∇2 E + ∇(∇ · E) ← ∇ × (∇ × E) = −
da cui
(∇2 −
∂
Ë
∇ × B = −µ0 J̇ − 2
∂t
c
1 ∂2
1
)E = µ0 J̇ + ∇ρ
2
2
c ∂t
0
(11.1)
Nel vuoto, assumendo che E abbia solo una componente Ez (r) che oscilla a frequenza ω, riscrivendo in coordinate
cilindriche
1 ∂2
1 ∂ ∂Ez
ω2
(∇2 − 2 2 )E = 0
i.e.
r
+ 2 Ez = 0
c ∂t
r ∂r ∂r
c
è risolta da Ez ∝ J0 (ωr/c). La funzione di Bessell J0 compare perchè siamo in simmetria cilindirica.
Esercizio 132: Cavità risuonante
bSoluzione: (Se racchiudo il condensatore formando una lattina aggiungendo la parete laterale dove E = 0
il campo elettrico interno risolve le equazioni di Maxwell. Cioè la cavità risuona alle frequenze ω = 2.405r/c,
5.52r/c... Ci sono altri modi con E orizzontale, come si vedrebbe più facilmente per una lattina cubica. Vedremo
che si trasmettono campi che dipendono anche da z).
Esercizio 133: Effetto pelle
Ad un filo di resistività ρ e raggio a viene applicata una differenza di potenziale oscillante a frequenza ω,
ottenendo una corrente alternata ed un campo elettrico parallelo al filo E(t) = E0 eiωt . Mostrare che la corrente
si sposta sul bordo, e che questo tende ad aumentare la resistenza effettiva.
bSoluzione: Oltre alla corrente J = σE c’è la corrente di spostamento Js = 0 Ė = iω0 E. A bassa frequenza
la corrente normale è più importante della corrente di spostamento (Js /J ∼ ωτ con τ = 0 /σ) che quindi
trascuriamo. Possiamo calcolare come si redistribuisce la corrente calcolando come si redistribuisce il campo
elettrico, in quanto J = σE. Assumendo che l’effetto sia piccolo, procediamo perturbativamente.
Capitolo 11. Corrente di spostamento
77
La J iniziale uniforme genera un campo magnetico Bθ (r) = 21 µ0 reiωt Jz , che per induzione genera una
correzione al campo elettrico E1 parallelo ed opposto a quello iniziale. Utilizzando coordinate cilindriche
r
2
ωσµ0 r
r
dEz
dove
δ≡
= −Ḃθ = −i
Ez0 = −i 2 Ez0
∇ × E = −Ḃ
:
dr
2
δ
ωσµ0
viene chiamata skin depth. Il campo elettrico, scritto in termini del suo valore Eext = Ez (r = a), è
Ez (r) = Eext
1 − ir2 /2δ 2
.
1 − ia2 /2δ 2
L’approssimazione perturbativa E1 E0 vale se δ a. Nel rame σ/0 ∼ 1018 sec−1 . Per a ∼ mm si ha δ a
fino a ω 105 Hz.
È interessante calcolare l’impedenza per unità di lunghezza, definita come Z = Eext /I, dove I è la corrente
totale:
Z a
1 − ia2 /4δ 2
.
I=
Jz 2πr dr = πa2 σEext
1 − ia2 /2δ 2
0
Per ω → 0 si ha δ → ∞ e si ritrova Z → R0 = 1/σπa2 . In generale Z = R + iωL ha una parte complessa (che
corrisponde all’impedenza dovuta a Bθ ) ed una parte reale maggiore di R0 :
Z = R0
1 − ia2 /2δ 2
1 + (a/2δ)2 − i(a/2δ)2
= R0
.
2
2
1 − ia /4δ
1 + a2 /8δ 2
Fisicamente, questo è dovuto al fatto che la corrente si concentra verso l’esterno aumentando l’‘intasamento’ e
quindi la resistenza. Quindi conviene lavorare a frequenze abbastanze basse che δ a.
• • • • • • • • • • • • • • • • • •
Se E1 E0 l’esercizio finisce qui. A grandi ω questo potrebbe non essere vero; in tal caso o si continua
lo sviluppo perturbativo E0 , E1 , E2 , . . ., oppure si risolvono le equazioni di Maxwell in forma differenziale.
Nell’equazione d’onda ricavata all’esercizio precedente metto J = σE, trascuro ρ ed il termine Ë dovuto alla
corrente di spostamento, assumo Ez eiωt , e riscrivo per un Ez (r) in coordinate cilindriche:
√ 1 ∂ ∂Ez
Ez0
2i
r 2i
00
˙
(r
) = µ0 Jz = iωσµ0 Ez
:
Ez +
= 2 Ez
:
Ez (r) ∝ J0
r ∂r ∂r
r
δ
δ
Abbiamo nuovamente ottenuto un’equazione differenziale ‘alla Bessel’ ma questa volta il coefficiente numerico
è immaginario. Infatti anche la correzione al primo ordine ad Ez era immaginaria. La funzione completa è
mostrata in fig. 11.1b ed è qualitativamente simile al risultato al primo ordine.
Consideriamo il limite δ a, opposto a quello studiato con il metodo approssimato. Per δ a si può
trascurare il termine Ez0 /r nell’equazione differenziale, per cui la soluzione approssimata diventa un’esponenziale:
Ez (r) ' Eext e−r(1+i)/δ . Fisicamente, significa che la corrente è grossa solo ai bordi del filo, e penetra per uno
spessore δ.
9
Come vedremo in seguito esistono le onde. A frequenze abbastanza grandi, ω >
∼ 10 Hz, hanno lunghezza
d’onda ‘umana’ ed è possibile utilizzarle per trasportare energia dentro cavità metalliche (‘guide d’onda’). Il
fatto che i campi penetrino dentro il metallo solo per un piccolo spessore δ e che quindi le correnti ‘parassite’
J = σE dentro al metallo siano trascurabili diventa un vantaggio. In questo modo si riesce a trasportare grandi
potenze con poca dissipazione di energia per effetto Joule. Tecnologicamente risulta più semplice trasportare la
corrente a basse frequenze in cavi di rame che ad alte frequenze in tubi di rami.
Esercizio 134: Filo conduttore interrotto
[Dal compito del 4/4/2003] Un filo conduttore rettilineo e cilindrico, di resistività ρ, raggio a e lunghezza ` a,
viene connesso ad un generatore, in modo che nel filo passa la corrente I = I0 cos ωt.
a) Si calcoli il campo magnetico in tutto lo spazio (assumendo un filo di lunghezza infinita) e il campo
elettrico per r < a nell’assunzione di corrente lentamente variabile. Si discuta a posteriori la condizione
necessaria a questa approssimazione.
78
Capitolo 11. Corrente di spostamento
Si taglia via un tratto h a del filo e si regola di nuovo il generatore in modo che passi la stessa corrente di
prima.
b) Si risponda di nuovo alla domanda a); come cambiano i campi prima e dopo l’interruzione del filo?
c) L’interruzione del filo si può schematizzare come l’inserimento in serie di una impedenza Z. Si stimi e
discuta il valore di Z in funzione di ω.
bSoluzione:
a) Per ω → 0 si ha un campo elettrico uniforme E0 = ρI/πa2 . La corrente genera un campo magnetico
Bθ = µ0 rI/2πa2 per r < a. Come già visto in esercizi precedenti, vi sono correzioni di ordine relativo
ωρ0 , che diventano significative ad alte frequenze.
b) Prendendo la divergenza della IV equazione di Maxwell ∇ × B = µ0 (J + 0 Ė) si impara che la somma
delle correnti elettrica e di spostamento si conserva, anche alla superficie di discontinuità. Quindi il
campo magnetico non cambia rispetto al punto a). La conservazione della corrente ‘totale’ consente di
determinare il campo elettrico Ev nella regione di vuoto: da 0 Ėv = J + 0 Ė0 segue Ev = E0 [1 + i/(ωρ0 )].
Infatti, si può schematizzare il sistema come un condensatore inserito fra due resistenze: sulle superfici si
deposita una densità di carica σ tale che σ̇ = j, per cui Ev = E0 + σ/0 .
c) La parte vuota si comporta come un condensatore di capacità C = 0 πa2 /h. Al crescere della frequenza
questo “condensatore” ha una induttanza parassita L ' µ0 h/8π. Per trovare L si può calcolare l’energia
magnetica contenuta nel condensatore
Z
Z a
LI 2
Bθ2
(µ0 rI/2πa2 )2
µ0 hI 2
≡
UM = dV
=
dr 2πr
=
2µ0
2µ0
16π
2
0
Quindi, l’impedenza associata è Z = iωL + 1/iωC.
L’induttanza diventa rilevante solo ad alte frequenze, quando
√ la nostra
√ approssimazione di corrente lentamente
variabile cessa di valere. Ad esempio Z = 0 per ω = 1/ LC = 2 2c/a: per valori di ω cosı̀ alti, I varia in
modo significativo nel tempo che la luce impiega ad attraversare il filo. A frequenze cosı̀ alte ci sono effetti
addizionali:
il filo irraggia; la corrente non è più uniforme a causa dell’effetto pelle. Infatti, ωρ0 1 implica
p
δ = 2 c0 ρ/ω c/ω, che per Z = 0 è comparabile al raggio a del filo.
Esercizio 135: Due cilindri cavi
[Da un compito del 1987] Due cilindri cavi coassiali di raggi a < b sono percorsi da correnti uguali I(t) = I0 sin ωt
in verso opposto, distribuite uniformemente sulle superfici. a) Trascurando la corrente di spostamento calcolare
B. b) Mostrare che E può avere una sola componente non nulla. c) Calcolare E assumendo che valga zero sul
cilindro esterno. d) Calcolare la corrente di spostamento Is . e) Discutere come deve essere calcolato B nei tre
casi Is I0 , Is < I0 e Is ≈ I0 . f) Caso numerico: a = 1 mm, b = 1 cm, I0 = 2 A, ω = 1000 Hz.
bSoluzione:
a) B = 0 per r < a ed per r > b. Nella zona intermedia 2πrBθ = µ0 I.
b) Per simmetria cilindica E può dipendere solo da r. Siccome non c’è carica Er = 0. Siccome Bz = 0 si ha
2πrEθ = 0. Quindi l’unica componente è Ez (r), generata da ∇ × E = −Ḃ.
c) Riscrivendo in componenti la II equazione di Maxwell si ottiene −∂Ez /∂r = −∂Bθ /∂t = −µ0 ωI0 cos(ωt)/2πr
per a < r < b. La soluzione con Ez (b) = 0 è

0
r>b
1 
µ0 I0 ω cos(ωt) ln(r/b) a < r < b
Ez (r) =
·
2π 
µ0 I0 ω cos(ωt) ln(a/b) r < a
d) La corrente di spostamento j s ≡ 0 Ė vale
Z
Is = 0
b
Ėz 2πr dr =
0
b2 − a2 ω 2
I
4
c2
Capitolo 11. Corrente di spostamento
79
e) Se Is I0 il conto perturbativo fatto finora è accurato. Se Is < I0 si può iterare B → E → B →
E → B → . . . aggiungendo i termini perturbativi successivi. Se Is ≈ I0 occorre risolvere le equazioni di
Maxwell. Da un punto di vista matematico questo è analogo a calcolare 1/(1 − ) epandendolo attorno ad
= 0. Se 1 bastano pochi termini della serie di Taylor 1 + + 2 + · · ·. Se <
∼ 1 ne servono tanti. Se
≥ 1 la serie perturbativa non funziona.
f) Siccome bω/c 1 siamo nel caso Is I0 .
Esercizio 136: Carica in moto
Una carica q si muove lungo l’asse z con velocità v c costante. Calcolare il campo magnetico che essa genera.
Spiegare in che modo tante cariche q che formano una corrente i continua producono approssimativamente un
campo magnetico che non dipende dal tempo.
bSoluzione: Senza includere la corrente di spostamento si ha j 6= 0 solo in coincidenza della carica, ed il
problema non ha senso. Includendo j s , essa è l’unica sorgente di B in tutto lo spazio vuoto. Qui risolviamo
il problema con un calcolo esplicito. Un modo alternativo di risolvere il problema consiste nel notare che le
equazioni di Maxwell complete sono relativisticamente invarianti, calcolare i campi nel sistema rispetto al quale
la carica è ferma, ed applicare le trasformazioni di Lorentz dei campi.
• Il campo magnetico ha solo componente Bθ (z − vt, r). È sufficiente calcolarlo a z = 0 per t ed r generici.
Integriamo la IV equazione di Maxwell, ∇×B = µ0 0 Ė lungo la superficie di un anello di raggio r giacente
a z = 0. Si ottiene
I
µ0 is
B · ds = µ0 0 Φ̇(E)
i.e.
Bθ (r) =
2πr
dove is è la corrente di spostamento che attraversa l’anello:
is = Φ(j s ) = 0 Φ̇(E) =
d q
q d
vt
qva2
√
(1 − cos θ(t)) =
=
dt 2
2 dt a2 + v 2 t2
2(a2 + t2 v 2 )3/2
Quindi Bθ (z, r, t) = µ0 qvr/4π(r2 + (z − vt)2 )3/2 . Per z = 0 è massimo a t = 0, cioè mentre q sta passando.
• Una successione di cariche con densità λ produce in media una corrente continua i = λv. Infatti il campo
magnetico generato vale
Z +∞
λ dz µ0 vr
µ0 i
Bθ =
=
2 + (z − vt)2 ]3/2
2πr
4π[r
−∞
che è la ben nota formula per il campo magnetico generato da una corrente i.
Capitolo 12
Onde e oscillazioni
Un’onda elettromagnetica piana polarizzata linearmente nel vuoto è descritta da
E = E 0 sin(k · r − ωt),
B = B 0 sin(k · r − ωt),
c=
E
ω
=
B
k
con E 0 , B 0 , k ortogonali. Frequenza: ν = ω/2π. Periodo: T = 1/ν. Lunghezza d’onda; λ = 2π/k = c/ν.
Densità e il flusso di energia:
0
u = (E 2 + c2 B 2 )
S = 0 c2 E × B
2
Quindi, in media
c
1
hSi = 0 E02 = chui
hui = 0 E02 ,
2
2
Esercizio 137: Sorgenti di onde
D
Quattro sorgenti identiche disposte come in figura emettono onde di
lunghezza d’onda λ. Due ricevitori sono situati a distanza r λ come
in figura. Calcolare il rapporto fra le potenze ricevute dai ricevitori.
Cosa cambia se B viene spenta? Se D viene spenta?
λ/2
R1
A
λ/2
B
λ/2
C
R2
bSoluzione: Si ha IE ∝ |EA + EB + EC + ED |2 :
√2 2
|eik(r−λ/2) + eikr + eik(r+λ/2) + eik r +λ /4 |2 ∝ | − 1 + 1 − 1 + 1|2
√2 2
√2 2
I2 ∝ |eik r +λ /4 + eikr + eik r +λ /4 + eik(r+λ/2) |2 ∝ |1 + 1 + 1 − 1|2
p
avendo usato kλ = 2π, e±ikλ/2 = −1, e r2 + λ2 /4 ' r, Quindi I1 /I2 = 0/4. Se B viene spenta I1 /I2 = 1/1.
Se D viene spenta I1 /I2 = 1/9. Quindi il ricevitore 1 non sa dire se viene spenta B o D, mentre il ricevitore D
può dirlo.
I1
∝
Esercizio 138: Ricevitore di onde
Calcolare la f.e.m. attraverso un cicuito quadrato di lato λ/4 disposto nell’asse xy, quando viene attraversato
da un’onda elettromagnetica che si propaga lungo x di lunghezza d’onda λ ed ampiezza E0 polarizzata lungo y.
bSoluzione: Ci sono 2 modi. La circuitazione del campo elettrico vale
I
λ
E = E · ds = E0 [sin(k0 − ωt) − sin(k(0 + λ/2) − ωt)] = −E0 λ sin ωt
2
Il flusso del campo magnetico vale
Z
Z
λ λ/2
B 0 λ2
Φ = dSBz =
dxB0 sin(kx − ωt) =
cos ωt
2 0
2π
80
Capitolo 12. Onde e oscillazioni
81
quindi E = −Φ̇ viene uguale a prima, come si verifica usando ω = 2πc/λ e E0 = cB0 .
I due E devono venire uguali, in quanto un’onda piana è soluzione della II equazione di Maxwell ∇×E = −Ḃ.
Esercizio 139: Luce solare
Calcolare i valori numerici del vettore di Poynting, i campi elettrici e magnetici, densità di energia, pressione
della luce solare, sapendo che essa fornisce un’energia K = 1366 J m−2 s−1 .
bSoluzione: La media del vettore di Poynting è uguale alla costante solare: hSi = K . Quindi pressione e
densità di energia valgono
hpi = chgi = hui =
N
J
hSi
= 4.5 10−6 3 = 4.5 10−6 2 .
c
m
m
Il campo elettrico vale E0 = 1000V/m (usando 0 = 8.85 10−12 C2 /N m2 e N · m = V · C). Il campo magnetico
vale B0 = E0 /c = 3.36 10−6 Tesla.
Il sole emetta luce ‘gialla’ di frequenza ν ≈ 0.5 1015 Hz e quindi ha λ = c/ν ≈ 0.6µm. La luce è composta da
‘fotoni’, ciascuno dei quali ha energia hν ∼ 10−19 J (la costante di Planck vale h = 6.62 10−34 J/ s): quindi la
luce del sole ne contiene circa 1022 / m2 s. Dalla relazione hν ∼ kB T si ricava la temperatura del sole. Nota la
temperatura TS = 5800 K ed il raggio RS = 7 108 m del sole, la potenza emessa può venir calcolata teoricamente,
sapendo che un corpo nero a temperatura T irraggia per unità di superficie una potenza
dW
= σT 4
dS
dove
σ=
4
π 2 kB
W
−8
3 2 = 5.670 10
m2 K4
60h̄ c
è detta ‘costante di Stefan-Boltzmann’. Quindi il sole irraggia una potenza WS = SS · σTS4 = 4πRS2 σTS4 =
4 1026 W. La terra è a distanza d ≈ 1.5 1011 m dal sole e riceve frazione Ω/4π = πR2 /4πd2 = 6.8 10−5 = 1/14500
della potenza totale. La potenza per unità di superficie vale K = (RS /d)2 σTS4 = 1366W/m2 .
• • • • • • • • • • • • • • • • • •
È interessante proseguire calcolando la temperature dei pianeti, anche se questo non è solo un esercizio di
elettromagnetismo. Anche la terra è approssimabile come un corpo nero a temperatura TE , calcolabile sapendo
che in condizioni di equilibrio irraggia tutta l’energia
p che riceve dal sole: tenendo conto che la terra è una sfera
πr2 K = 4πr2 σTE4 da cui TE = TS (Ω/4π)1/4 = RS /2dTS ≈ 280 K, che è una buona approssimazione. È
buona perché per via di un teorema generale, emissività e riflettività sono uguali e quindi si cancellano. Non è
perfetta per via dell’effetto serra: le zone interne dell’atmosfera sono più calde. La luna ha
√ la stessa temperatura
media TS della terra, ma non ha atmosfera: quando il sole è a picco raggiunge Tday = 2TE ≈ 400 K, di notte
scende fino a T Tday .
Marte è dista dal sole il 50%
√ in più della terra, e quindi la sua temperatura è circa 25% più bassa, TM ≈ 230K.
Ma a mezzogiorno raggiunge 2TM ≈ 320 K e l’acqua si può scongelare.
• • • • • • • • • • • • • • • • • •
È interessante calcolare l’energia ottenibile da pannelli solari. Come detto sopra il sole a picco produce
K = 1.366kW/m2 : tenendo conto che pannelli solari hanno efficienza ≈ 0.1, la potenza prodotta vale
W = K = 1kW/8m2 . L’energia totale prodotta in un anno vale E = K epsilon · yr/2/3 = 200kWh/m2 , dove
1/2 tiene conto che di notte non c’e’il sole, ed 1/3 dell’inclinazione del sole hcos2 θi < 1 e delle nuvole. Siccome
un kW h costa circa 0.15Euro, il risparmio prodotto è di circa 30Euro/yr·m2 (se l’energia viene utilizzata). Il
costo di installazione è di circa 1000Euro/m2 .
Esercizio 140: Vettore di Poynting
Verificare che il vettore di Poynting descrive veramente la variazione di energia in varie geometrie.
bSoluzione:
82
Capitolo 12. Onde e oscillazioni
1. Una capacità cilindrica (lato a, spessore h) a frequenze non troppo alte ha campo elettrico Ez (t) e la
corrente di spostamento genera un campo magnetico lungo θ dato da Bθ = πr2 Ėz /c2 /2πr = rĖz /2c2 .
L’energia contenuta in un raggio r vale
U ' (πr2 h)
0 2
E
2
U̇ = πr2 h0 Ez Ėz
Il vettore di Poynting vale
Sr = −0 c2 Ez Bθ =
I
r
0 Ez Ėz
2
Sr = −2πr hSr = −U̇
in accordo con u̇ + ∇ · S = 0, che possiamo anche verificare usando ∇ · S = (1/r)d(rSr )/dr.
2. Un filo resistivo di lunghezza h ha un campo elettrico costante Ez = V /h. La corrente j = σE dissipa
una potenza U̇ = h · πr2 jE e genera
un campo magnetico Bθ = µ0 j πr2 /2πr. Quindi il flusso del vettore
H
di Poynting Sr = −rjE/2 vale Sr = −h πr2 jE = −U̇ .
3. Un cavo coassiale porta corrente continua a tensione V su di una resistenza R, che congiunge le due
armature. Quindi ci passa una corrente i = V /R, che dissipa una potenza W = iV . Al suo interno
contiene un campo elettrico ed un campo magnetico dati da
Er =
V
r ln r2 /r1
µ0 i
2πr
Bθ =
Il flusso del vettore di Poynting vale
I
Z
Sz = 0 c2 Er Bθ =
iV
2πr2 ln r2 /r1
r2
Sz =
Sr 2πr dr = iV = W
r1
4. Un solenoide rettilineo infinito contiene un campo magnetico Bz = µ0 nI e quindi un campo elettrico
2
2
2
E
H r = −Ḃz r/2. Il vettore di Poynting vale Sr = −Bz Ḃz r/2µ0 . U = πr Bz /2µ0 , U̇ = πr Bz Ḃz /µ0 ,
Sr = 2πr Sr = −U̇ .
5. Paradosso di Feynman. Un disco libero di ruotare lungo l’asse z contiene una carica q ed un solenoide
percorso da una corrente continua che genera un campo magnetico Bz . Si interrompe la corrente: la
variazione di Bz genera un Er che mette in rotazione il sistema. Che ne è della conservazione del momento
angolare?
6. Un elettrone in un atomo di idrogeno ruota a distanza a dal protone con velocità determinata da
me v 2 /a = qe2 /4πa2 0 cioè
v2
qe2
1
r2
1
=
≡ e2 =
2
2
2
c
4π0 me c a
a
137.2
per
a = 0.53Å.
La lunghezza re è detta “raggio classico dell’elettrone” (ma non è il raggio dell’elettrone), in quanto è il
raggio che avrebbe un elettrone se la sua massa fosse dovuta all’elettromagnetismo:
U∼
qe2
= me c2
4π0 re
per
re =
qe2
= 2.82 10−13 cm
4π0 me c2
(per abbreviare spesso si usa e2 ≡ qe2 /4π0 invece di qe ). L’elettrone, ruotando attorno al protone, genera
anche un campo magnetico campo magnetico di tipo dipolare, B ∼ µ0 µ/r3 dove il dipolo magnetico vale
µ = πa2 i = πa2 (qe ω/2π) = (qe /2m)L (L = ma2 ω). Quindi Sθ ∼ Er Bz /µ0 . La densità di momento vale
g = S/c2 . Il momento angolare elettromagnetico vale
Z
Lem ∼
dV r × g ∼ a4 g(a) ∼
qe2 L
re
v2
L
∼
L
∼
L∼
2
2
mc 0 a
a
c
1372
ed è quindi una frazione trascurabile del momento angolare meccanico.
Capitolo 12. Onde e oscillazioni
83
Esercizio 141: Rilfessione di onde in una corda
Si connettono due corde con diverse densità lineari λ e λ0 = n2 λ (attenzione a non confondere λ con la lunghezza
d’onda). Studiare cosa succede quando un onda trasversale arriva al punto di congiunzione.
bSoluzione: Metto la corda lungo l’asse x. Un’onda produce una distorsione y(x, t) rispetto al valore di
equilibrio y = 0. La legge del moto per y(x, t) è
ma = F
:
λ
∂2y
∂2y
=τ 2
2
∂t
∂x
(compare y 00 perchè la tensione
non richiama una corda dritta, con y 0 costante). La tensione τ è la stessa
p
dovunque. La velocità v = τ /λ(x) = k/ω cambia imrovvisamente sul punto di congiunzione, che mettiamo a
x = 0. Siccome v 0 = v/n, nell’analogo elettromagnetico n sarà l’indice di rifrazione.
Non serve studiare un pacchetto d’onda. Cerco la soluzione di onda piana: oltre all’onda incidente y =
a ei(kx−ωt) aggiungo una componente Riflessa ed una T rasmessa
i(kx−ωt)
e
+ R ei(−kx−ωt) per x < 0
y=a
i(k0 x−ωt)
Te
per x > 0
È intuitivamente ovvio che al punto di congiunzione fra le due corde y e y 0 sono continui. Matematicamente
segue dal fatto che l’equazione d’onda contiene due derivate rispetto a x (e quindi rimarrà vero in casi meno
intuitivi, come nelle equazioni di Maxwell o Schroedinger). Imponendo la continuit’à si trova
1+R=T
k(1 − R) = k 0 T
→
T =
2k
2
=
k + k0
1+n
R=
k − k0
n−1
=
k + k0
n+1
Si ha T > 1 se n < 1: questo è sensato, e dovuto al fatto che nel passare da una corda ‘grossa’ and una corda
‘piccola’ l’ampiezza dell’onda si amplifica.
Non si deve amplificare la sua energia. La potenza media dell’onda incidente (che corrisponde al vettore di
Poyinting nell’analogo e.m.) vale WI = vu = 12 vλ(aω)2 : infatti 14 λ(aω)2 è la densità di energia cinetica media,
che è in media eguale all’energia potenziale. La potenza riflessa vale WR = WI R2 e quindi quella trasmessa deve
valere WT = WI − WR .1 Infatti, facendo il calcolo esplicito sfruttando il fatto che W ∝ v · λ · a2 ∝ (1/n) · n2 · a2
si ottiene
WT
4n
= T 2n =
<1
WI
(1 + n)2
Come giusto viene WT /WI ≤ 1, massimo per n = 1.
Esercizio 142: Rifrazione
Fare incidenza normale, mostrando che la cons. energia richiede un onda riflessa. Calcolare forza sentita da
separazione vuoto/dielettrico I − T + R mostrando che si ottiene 2 : 1 : 0 per riflettente:assorbente:trasmettente
bSoluzione:
Esercizio 143: Rifrazione
Due dielettrici sono separati dal piano x = 0. Un’onda elettromagnetica incide lungo k = (kx , ky , 0) con E
polarizzato lungo z. Cosa succede?
bSoluzione: In un dielettrico i campi e.m. si propagano in modo simile al vuoto, con l’unica differenza che
c → c/n:
1
1
c2
E
c
ω
c2 =
→
≡ 2,
=c→ =
0 µ0
µ0
n
B
n
k
1 Nella zona dove ci sono due onde, la potenza media è la somma delle potenze media delle due onde: W = W −W . Fisicamente,
I
R
questo è chiaro in quanto usare onde piane è un modo utile per descrivere un problema nel quale si invia un pacchetto d’onde, che
viene parzialmente trasmesso e riflesso. Per vederlo matematicamente con onde piane, occorre notare che il termine di interferenza
fa zero, se mediato sullo spazio e sul tempo, usando formule tipo 2 sin(kx − ωt) sin(−kx − ωt) = cos(2kx) − cos(2ωt).
84
Capitolo 12. Onde e oscillazioni
Le condizioni al bordo fra due dielettrici sono B 1 = B 2 e E1= = E2= , 1 E1⊥ = 2 E2⊥ . Assumo che il piano di
separazione sia il piano yz a x = 0, e che l’onda si propaghi nel piano xy con il campo elettrico polarizzato
lungo z. Per prima cosa, le condizioni al bordo, dovendo essere vere per ogni t, implicano che l’onda incidente,
l’onda riflessa (nel mezzo 1) e quella trasmessa (nel mezzo 2) hanno la stessa frequenza. Dovendo poi essere
vere per ogni y, si ha che tutte le onde hanno lo stesso ky : ky ≡ kyi = kyr = kyt . Siccome i moduli dei vettori k
sono dati da ki = ni ω/c si ha ki /kt = n1 /n2 e quindi si impara che l’onda trasmessa si inclina:
sin θt =
ky
kt
sin θi =
ky
ki
:
n2 sin θt = n1 sin θi
Per quanto riguarda le ampiezze dell’onda le condizioni di raccordo sono
Ei + Er = Et
kx Ei − kx Er = kxt Et
(da E o da Bx )
(da Ey0i + Ey0r = Ey0t o da Byi + Byr = Byt usando B = k × E/ω)
e quindi
Er =
kx − kxt
Ei
kx + kxt
Et =
2kx
Ei
kx + kxt
che è identica a quanto ottenuto nell’esercizio precedente su onde due corde con un capo in comune.
Allo stesso modo funziona anche il rapporto fra potenze trasmesse ed incidente. La potenza trasmessa vale
WT = vu dove u è la densità media di energia. In un dielettrico essa vale
B2
u = h E2 +
i = h [E 2 + B 2 v 2 ]i = E 2
2
2µ0
2
2
Tenendo conto che v ∝ 1/n e che u ∝ E 2 ∝ n2 E 2 si ha nuovamente
WT
4n
< 1.
= T 2n =
WI
(1 + n)2
Esercizio 144: Riflessione da un metallo
Un’onda piana di frequenza ω con polarizzazione lineare parallela alla superficie incide sulla superficie di un
metallo di conducibilità σ.
a) Determinare la forma dell’equazione delle onde elettromagnetiche nel metallo.
b) Studiare la penetrazione dell’onda nel metallo.
c) Calcolare la riflettività del metallo (rapporto tra intensità riflessa ed incidente)
d) Trovare il flusso di energia verso l’interno del metallo e confrontarlo con la potenza dissipata per effetto
Joule.
bSoluzione:
a) Mettendo J = σE e ρ = 0 nella (11.1) si ottiene
(∇2 −
1 ∂2
∂E
)E = µ0 σ
c2 ∂t2
∂t
Per onde monocromatiche E(x, t) = Ẽ(x)e−iωt l’equazione si riduce a
(∇2 +
ω2
)Ẽ = −iωµ0 σ Ẽ
c2
Con metodi simili si trova che il campo magnetico soddisfa alla stessa equazione, ottenuta rimpiazzando
E → B, che ha solo componente By .
Capitolo 12. Onde e oscillazioni
85
b) Supponiamo che il conduttore si estenda per x > 0 e che E sia polarizzato lungo z. Quindi soddisfa
l’equazione (per x > 0)
∂2
ω2
( 2 + 2 )Ẽz = −iωµ0 σ Ẽz
∂x
c
La soluzione è quindi Ez = ei(qx−ωt) Ez (0) dove q soddifsa la relazione di dispersione
2
ω2
q1 − q22 = ω 2 /c2
i.e.
q 2 = 2 + iωµ0 σ
2q1 q2 = ωµ0 σ
c
dove q = q1 + iq2 . q2 descrive lo smorzamento dell’onda, provocato dalle correnti parassite. La soluzione
esplicita per q non è illuminante, anzi fa piuttosto schifo. Numericamente il rame ha ω0 ≡ µ0 σc2 = σ/0 =
6.6 1018 Hz, nel range di frequenza dei raggi X, circa tre ordini di grandezza maggiore delle frequenza della
luce visibile. Studiamo due casi limite.
– Ad alte frequenze ω ω0 q1 ha circa il valore di vuoto, q1 ' ω/c e l’effetto della conducibilità è
descritto da un piccolo q2 ' µ0 σc. Quindi
Ez (x, t) = Ez (0)eikx−iωt e−µ0 σcx
dove
k = ω/c = 2π/λ.
Cioè l’onda si propaga in maniera simile a quanto fa nel vuoto, ma smorzandosi su una distanza
tipica 1/q2 λ. Per il rame 1/µ0 σc ∼ 0.4 10−10 m.
p
– A basse frequenze ω ω0 si ha q1 ' q2 ' ωµ0 σ/2 ≡ 1/δ e ritroviamo il caso dell’esercizio di
pagina 70: l’onda si smorza in qualche lunghezza d’onda. Questo è tipicamente il caso di metalli alle
frequenza della luce visibile, che non penetra nel metallo e viene quindi riflessa (a meno di un piccolo
assorbimento).
c) Per x < 0 i campi soddisfano l’equazione d’onda nel vuoto, con soluzione generale
Ez (x < 0, t) = EI eikx−iωt + ER e−ikx−iωt .
EI ha il significato fisico di onda incidente, mentre EI quello di onda riflessa. Bisogna calcolare ER /EI .
– Ad alte frequenze tutta l’onda entra (ER ' 0) e dissipa la sua energia per effetto Joule.
– A basse frequenze il campo dentro il metallo è
Ez (x > 0, t) = ET e−iωt−ix/δ e−x/δ
Siccome E risolve un’equazione di secondo grado in x, per raccordare le soluzioni occorre che Ez e
∂Ez /∂x siano continue a x = 0, cioè
EI + ER = ET ,
ik(EI − ER ) = iqET '
i−1
ET
δ
da cui
ER
ET
kδ − 1 − i
2kδ
,
=
=
EI
kδ + 1 + i
EI
1 + kδ + i
Poichè l’intensità delle onde sono proporzionali ai moduli quadri dei campi, la riflettività R è il
modulo quadro dell’ultima espressione:
√
1 + (kδ − 1)2
ω + ω0 − 2ωω0
√
R=
=
<1
1 + (kδ + 1)2
ω + ω0 + 2ωω0
R
Nella figura questa espressione approssimativamente valida per ω ω0 (linea tratteggiata) è confrontata con il risultato completo (linea continua):
1
0.8
0.6
0.4
0.2
0
10-4 10-3 10-2 10-1
1
10
ω/ω0
102 103 104
Per kδ 1 si ha R ' 1, cioè tutta l’energia viene riflessa. Infatti se la lunghezza di penetrazione δ è
trascurabile l’onda non può dissipare energia per effetto Joule, e deve quindi tornare indietro.
86
Capitolo 12. Onde e oscillazioni
d) A basse frequenze abbiamo ottenuto WR /WI = R2 . Quindi la potenza trasmessa dovrebbe valere WT =
T WI con
4kδ
T =1−R=
1 + (kδ + 1)2
e questa potenza dovrebbe venir dissipata dalle correnti parassite nell’interno del metallo. Verifichiamolo.
Dentro il metallo
eix/δ−iωt
2kδEI
−x/δ
e
Ez (x > 0) = EI · 2kδe−x/δ Re
=
(1
+
kδ)
cos(x/δ
−
ωt)
+
sin(x/δ
−
ωt)
1 + kδ + i
1 + (kδ + 1)2
La potenza media Wdiss dissipata per unità di superficie è
Z ∞
Z ∞
σδ(kδ)2 Ei2
2σ(kδ)2 EI2
dx e−2x/δ =
Wdiss =
dx σhEz2 t i =
2
1 + (kδ + 1) 0
1 + (kδ + 1)2
0
avendo usato h(A cos +B sin)2 it = (A2 + B 2 )/2. Usando σkδ 2 = σ(ω/c)(2/µ0 σω) = 2/(µ0 c) = 20 c
otteniamo il risultato atteso
4kδ
0 cEI2
Wdiss =
= T WI .
1 + (kδ + 1)2 2
Esercizio 145: Onde adiabatiche
Una corda uniforme di massa m e lunghezza ` è appesa nel campo di gravità g. Come si propagano le onde?
p
√
bSoluzione: La tensione ad un punto z è τ = mgz/`, quindi la velocità delle onde è v(z) = τ /λ = gz. Se
λ ` le onde si propagano adiabaticamente, cioè con v = v(z), e quindi
Z
`
t(` → z) =
z
Un corpo che cade impiega un tempo t0 (` → z) =
p
p
dz
√ = 2[ `/g − z/g]
gz
p
2(` − z)/g.
Esercizio 146: Guida d’onda
Guida d’onda rettangolare disposta lungo z e di lati a lungo y e b lungo x
bSoluzione: Come un cavo coassiale senza filo centrale, in modo da evitare elettroni che irraggiano. Usare
un tubo grosso vuoto conviene quando si devono trasportare grosse potenze. Le condizioni al bordo su di un
conduttore perfetto sono E= = 0 e B⊥ = 0. Provo la soluzione
Ey = E0 sin kx x ei(kz z−ωt)
kx =
nπ
a
Soddisfa a ∇ · E = ∂y Ey = 0. Metto n = 1. Per ottenere un’onda trasversa rispettando B⊥ = 0 aggiungo
Bx = B0 sin kx x ei(kz z−ωt)
Tuttavia ha divergenza diversa da zero: è possibile mostrare in generale che le onde in una cavità singola non
sono trasverse (le componenti trasverse soddisfano a ∇2 V = 0: in un cavo coassiale V può essere diverso sui
due bordi, con un bordo solo l’unica soluzione è V = cte). Occorre quindi aggiungere un campo longitudinale.
Mettiamo un Bz
kx
Bz = i B0 cos kx x ei(kz z−ωt)
kz
I fattori relativi fra Bz e Bx sono tali che ∇ · B = ∂z Bz + ∂x Bx = (ikz − ikz )Bz = 0, in particolare B= è
costante sul bordo, dove E = 0. Le equazioni ∇ × E = −Ḃ, componenti x e z implicano kz E0 = −ωB0 . Quindi
le linee di B circolano attorno al massimo di Ėy .
Capitolo 12. Onde e oscillazioni
87
Per finire, per via della relazione fra E0 e B0 , l’equazione (∇ × B)y = Ėy /c2 implica
p
p
kx2 + kz2 = ω 2 /c2
i.e.
kz = ± (ω/c)2 − (π/a)2 = ±(ω/c) 1 − (ωc /ω)2
che può essere immediatamente derivato dall’equazione d’onda per Ey .
Sotto la frequenza critica ωc = πc/a kz diventa immaginario, il che significa che l’onda si attenua come e−|kz |z .
Questo viene chiamato modo TE in quanto Ez = 0. Esistono altre onde ‘TM’ con Ez = E0 sin(nπx/a) sin(mπy/b)
con n, m ≥ 1 che quindi ha una frequenze di cut-off maggiore: di solito si lavora in modo che solo TE10 possa
propagarsi. Quindi necessariamente λ ∼ cm i.e. microonde.
Le velocità di fase e di gruppo sono
vf =
ω
c
>c
=p
kz
1 − (ωc /ω)2
vg =
p
dω
= c 1 − (ωc /ω)2 < c
dkz
La densità di energia si muove con velocità vg . Infatti
0
1 0 E02
1
k2 B 2
0
k2 + k2
huix,y,t =
+
(1 + x2 ) 0 =
1 + z 2 2x E02 = E02
2 2 2
2µ0
kz 2
8
ω /c
4
R
Ra
La componente x del vettore di Poynting, Sx , è diversa da zero, ma Sx ∝ 0 dx sin kx x cos kx x = 0. Lungo z
hSz ix,y,t =
11 2
0
kz
0 c E0 B0 = E02 × c2
= u · vg
22
4
ω
La lunghezza d’onda nella guida vale
λg =
2π
λ0
=p
kz
1 − (λ0 /2a)2
λ0 =
2πc
ω
Accoppiatore unidirezionale: due buchi separati di λ/4, che diventa λ/2 (fuori fase) o 0 (in fase) a seconda
della direzione di propagazione.
Un modo per capire fisicamente l’esistenza della frequenza di taglio è mettere un filo nel centro della guida;
per avere E = 0 sui bordi si aggiungono infinite immagini. Avevamo visto che se nel filo c’e’una carica costante
il campo muore esponenzialmente. Se invece la carica oscilla i campi si possono sommare costruttivamente per
via del ritardo di fase; in direzioni θ tali che la differenza di distanza fra due fili a sin θ è uguale a (n − 1/2)λ0 .
Prendo n = 1. Sommando ±θ la lunghezza d’onda nella guida vale λg = λ0 / cos θ che equivale alla formula di
prima. Si ha λg > λ0 . Questo è quello che si ottiene anche ragionando in termini di raggi di luce che rimbalzano
con angolo θ riflettendosi fra i bordi nella guida, se si tiene conto che ad ogni riflessione i campi si invertono.
La velocità di gruppo è ridotta in modo corrispondente.
Esercizio 147: Cavità risuonante
Stimare il Q
bSoluzione:
Q ≡ ω0
Energia immagazzinata
Potenza dissipata
cioè u(t) ∝ e−ω0 t/Q e E(t) ∝ eiω0 t−ω) t/2Q e quindi lo spettro di energia è
|E(ω)2 | ∝
La stima è Q ∼ V /Sδ ∼ 1000 dove δ =
1
(ω − ω0 )2 + (ω0 /2Q)2
p
20 c2 /σω è la lunghezza di pelle.
Esercizio 148: Pressione di radiazione
Una sfera di raggio R si trova a distanza r dal sole. a) Si calcoli la forza sulla particella dovuta alla radiazione
solare, assumendo che questa venga tutta assorbita. La sferetta abbia densità ρ = 1g/ cm3 e sia soggetta anche
88
Capitolo 12. Onde e oscillazioni
alla attrazione solare. b) Si determini il raggio R0 per cui tutte le sferette con raggio inferiore sono espulse dal
sistema solare.
bSoluzione: La pressione di radiazione è diretta lungo la direzione dell’onda
Z
prad = u
Frad = prad dS⊥ = πR2 u.
cioè conta solo la dimensione dell’ombra, e non la forma dell’oggetto. Se invece di essere perfettamente assorbente
fosse perfettamente riflettente la componente radiale della forza diventerebbe 1 ÷ 2 maggiore a seconda della
sua forma. Avevamo visto che hui = (d2 /r2 )4.5 10−6 N/ m2 , dove d è la distanza della terra dal sole.
Alla stessa distanza la forza di gravità produce un’accelerazione a = GM/d2 = 0.006m/s2 e quindi una forza
Fgrav = ma = 43 ρR3 a. Si ha Fgrav < Frad per ρR < 3hui/4a = 0.57 10−3 kg/ m2 . Quindi, per ρ = 1000kg/ m3 ,
si ha R0 = 5.7 10−7 m.
Esercizio 149: Velocità di gruppo
Illustrare in un caso semplice la velocità di gruppo.
bSoluzione: Consideriamo la sovrapposizione di due onde con eguali moduli di E
ω1 − ω2
k1 + k2
ω1 + ω2
k1 − k2
z−
t sin
z−
t
E = A[sin(k1 z − ω1 t) + sin(k2 z − ω2 t)] = 2A cos
2
2
2
2
L’inviluppo delle due onde produce un’onda lunga che si muove con velocità vg = ∆ω/∆k ' dω/dk. Definendo
vf = ω/k ≡ c/n(k) si ha vg = c/(n + ωdn/dω).
Esercizio 150: Pulsar
Una pulsar emette brevi impulsi a radio frequenze. Sapendo che ν1 = 400 MHz arriva ∆t = 1 s dopo ν2 = 1000
MHz, e che n2 = 1 − Ne e2 /0 me ω 2 con Ne ≈ 3 104 / m3 calcolare la distanza della pulsar.
bSoluzione: Fra la pulsar e la terra la presenza di elettroni liberi rende il mezzo dispersivo. Riassumo la
derivazione della frequenza di plasma. Un’elettrone libero si muove secondo me ẍ = qe E generando un dipolo
p = ex = αE con α = −e2 /me ω 2 . La densità di polarizzazione del mezzo è quindi P = Ne p, per cui
s
Ne e2
ωp 2
P
Ne e2
2
n ≡
=1−
=1−( )
dove
ωp =
=1+
= 104 Hz
2
0
0 E
0 mω
ω
0 me
è detta ‘frequenza di plasma’ in quanto è anche la frequenza delle oscillazioni meccaniche del plasma.
Mostriamo adesso che frequenze basse viaggiano più lente, arrivando con un ritardo ∆t = D/vg1 − D/vg2
(dove vg è la velocità di gruppo — la velocità di fase ha il comportamento opposto). Ricordando che n ≡ c/vf =
ck/ω i.e. k = nω/c
ωp2
1
dk
1 d(nω)
1
dn
1
=
=
= (n + ω
) ' (1 +
)
vg
dω
c dω
c
dω
c
2ω 2
avendo approssimato n ' 1 − ωp2 /2ω 2 in quanto n − 1 ∼ 10−11 . Quindi, ricordando ω = 2πν
D=
2c ∆t
≈ 5000 ly
− ωp2 /ω22
ωp2 /ω12
Esercizio 151: Interferenza
(dal compito del 20/6/2003) Una nave percorre una rotta parallela alla costa alla distanza di circa 100 km da
questa e alla velocità di 18 nodi. Un marinaio a bordo della nave sta ascoltando un programma musicale, sulla
frequenza di 1200 kHz, trasmesso da una stazione situata sulla costa, in direzione perpendicolare alla rotta.
Capitolo 12. Onde e oscillazioni
89
L’altezza del segnale varia regolarmente col tempo apparendo e scomparendo e l’intervallo tra il massimo ed
il minimo è 2 minuti. Nei momenti di massima intensità, il segnale ricevuto dall’antenna è stimato a circa 12
mV/m, pari a circa 8 volte il livello di rumore. Si fa l’ipotesi che una seconda stazione costiera vicina alla prima
abbia iniziato a trasmettere per errore in fase e sulla stessa frequenza. Inquadrando i fenomeni nell’ambito
dell’esperimento di Young stimare, fornendo i risultati numerici: a) La distanza d tra le due stazioni; b) La
potenza emessa da ciascuna delle due stazioni.
Note: 0. Si trascuri ogni effetto dovuto alla sfericità della Terra. 1. Un nodo è pari ad un miglio nautico
(circa 1.8 km) all’ora. 2. Si consideri solo la cosiddetta “portante” come un’onda monocromatica. 3. Si intende
che il segnale “scompare” quando è inferiore al livello di rumore. 4. Si ipotizza, salvo verifica, che la distanza
tra le stazioni sia molto minore della distanza tra queste e la nave.
bSoluzione: a) La lunghezza d’onda del segnale ricevuto è λ = c/ν = 250 m mentre la distanza tra due massimi
(o minimi) del segnale è a = 2v∆t = 2160 m.
Le onde emesse da due sorgenti a distanza d ricevute ad angolo θ sono in fase se nλ = d sin θ = da/D.
Quindi la distanza tra due massimi consecutivi vale a = λD/d (‘formula di Young’) da cui d = λD/a = 11.6 km.
b) Sia E0 il valore del campo elettrico corrispondente alla soglia del rumore; se E1 e E2 sono le ampiezze dei
campi elettrici emessi dalle due stazioni, le condizioni sui massimi e minimi di intensità sono
E1 + E2 = 8E0 ,
E1 − E2 = kE0
con k ∈ [0, 1]. Risolvendo il sistema si ottiene:
E1 = (8 + k)E0 /2,
E2 = (8 − k)E0 /2
L’intensità media dell’onda ricevuta può essere messa in relazione con la potenza della stazione emittente e con
il campo elettrico all’antenna.
I=
1
W
= c0 E 2 ,
2
4πD
2
W = (2πc0 ) D2 E 2 = (1/60)D2 E 2
Ne segue che la potenza delle due stazioni vale
W1 = (8 + k)2 D2 E02 /240,
W2 = (8 − k)2 D2 E02 /240
Sostituendo i valori numerici, al variare di k, si ha:
6 kW < W1 < 7.6 kW,
Il valore esatto dipende da k, che non è noto.
6 kW > W2 > 4.6 kW
Capitolo 13
Irraggiamento
Una carica q in moto non relativistico irraggia
E=
qe 1
n × (n × ärit ),
4π0 c2 r
cB = n × E
dW
e2 2
a sin2 θ
=
dΩ
4πc3
W =
2 e2 2
a
3 c3
dove θ è l’angolo fra a e la direzione di osservazione n, e e2 ≡ qe2 /4π0 . In media temporale corrisponde ad una
3
˙
. Un dipolo elettrico p(t) irraggia una potenza W = 2p̈2 /3c3 4π0 .
forza −2e2 ẍ/3c
Esercizio 152: Atomo di idrogeno
Calcolare quanto dovrebbe irraggiare.
bSoluzione: Un elettrone in un atomo ruota con accelerazione a = ω 2 rA , quindi in un giro irraggia
∆E = W ·
2πrA
4π e2 v 3
=
( ) ∼ Eatomo α3
v
3 rA c
dove α = v/c ∼ 1/137. e fa 1017 giri al secondo. Questa formula stima bene la vita media dei livelli eccitati, ma
lo stato base ha vita media infinita.
Modello classico per la stabilità dell’atomo di idrogeno: se invece di immaginare l’elettrone come una carica
puntiforme lo si pensasse come 2 cariche messe ai punti opposti della traiettoria, la potenza irraggiata sarebbe
zero, in approssimazione di dipolo. Con n carice e/n verrebbe ridotta di un fattore n. Nel limite n → ∞
l’elettrone viene spalmato lungo la sua traiettoria e non irraggia più, anche se i singoli pezzi lo farebbero, per
via di interferenza distruttiva.
Siccome ∆E Eatomo se cadesse, spiraleggerebbe lentamente su orbite circolari: a ciascun istante
e2
= me ω 2 r
r2
:
U=
me v 2
e2
e2
−
=−
2
r
2r
Quindi
e2
2e2 e2 2
ṙ
=
U̇
=
W
=
(
)
2r2
3c3 me r2
cioè
d(r3 )
e2
= 4re2 c
:
r3 (t) = r03 − 4re2 ct
re ≡
= 2.8 10−13 cm.
dt
me c2
Questo effetto e’stato osservato nel caso analogo della gravità, discusso in seguito: due masse in rapida
rotazione una attorno all’altra irraggiano onde gravitazionali.
Polarizzazione Una carica in moto circolare corrisponde ad un dipolo elettrico rotante p ∝ (1, i, 0)eiω . I
campi di dipolo elettrico in zona di radiazione (r cω) sono dati da
B=
1 p̈rit × n
;
4π0 rc3
E = cB × n
Quindi B ∝ (x + iy) × n: se si osserva lungo l’asse z, essendo (x + iy) × z = i(x + iy) la polarizzazione è
circolare. Se invece n = x la polarizzazione è lineare. Infatti guardando dall’altro si vede una carica che gira,
guardando di taglio una carica che oscilla.
90
Capitolo 13. Irraggiamento
91
Esercizio 153: Scattering elettrone/nucleo
Un elettrone con velocità iniziale v c urta frontalmente un nucleo di carica Ze. Calcolare la polarizzazione
della radiazione emessa, e l’energia totale irraggiata, assumendo e verificando che sia una piccola frazione
dell’energia cinetica.
bSoluzione: Secondo la formula generale E è polarizzato linearmente nel piano (n, a) ed ortogonale ad n, e
B ‘gira’ attorno ad a. L’accelerazione è a = F/me = Zqe2 /4π0 r2 . Quindi, applicando la formula generale
dW
qe2 a2
sin2 θ,
=
dΩ
16π 2 0 c2
W =
2 e2
A
a2 = 4
3 4π0 c3
r
A=
qe6 Z 2
96m2e (cπ0 )3
L’energia irraggiata vale
Z
Eirr =
∞
Z
W dt = 2
rmin
W
8 me v05
dr =
v
45 c3 Z
Eirr
v0
∼ ( )3
T0
c
Alla fine viene Z al denominatore in quanto se Z 1 l’elettrone rimane lontano dal nucleo. Per fare il conto
si usa la conservazione approssimata dell’energia
s
v(r) =
v02
2Zq 2
−
=
4π0 me r
r
v02 −
β
.
r
L’integrale vale
Z
∞
β/v02
v05
dr
p
=
β3
r4 v02 − β/r
Z
1
∞
dx
16 v05
p
=
15 β 3
x4 1 − 1/x
avendo usato la variabile di integrazione adimensionale x = r/rmin . A parte il fattore numerico 16/15 il risultato
2
2
segue in modo semplice dal fatto che l’integrale è dominato da r >
∼ β/v0 , cosicchè w ∼ v0 . Il fattore numerico è
calcolabile usando y = 1 − 1/x come variabile di integrazione.
Per fare il conto con scattering frontale abbiamo messo una forza repulsiva. In realtà la forza è attrattiva e
lo scattering non è frontale. Ma il risultato qualitativo rimane lo stesso.
Esercizio 154: Scattering ee
Si può stimare l’energia irraggiata mettendo Z = 1 nella risposta dell’esercizio precedente?
bSoluzione: Nell’esercizio precedente il nucleo era molto più pesante dell’elettrone e quindi rimaneva circa
fermo. A prima vista avere due particelle di massa uguale cambia solo qualche fattore di O(1) in quanto
irraggiano entrambe. Sbagliato. C’è una differenza qualitativa importante. Il ‘dipolo elettrico totale’ dei due
elettroni è proporzionale al loro impulso
p = er 1 + er 2 =
e
P
me
e quindi è costante. In approssimazione ‘di dipolo’ non c’è irraggiamento. Entrambi gli elettroni accelerano, ma
c’è un’interferenza distruttiva fra i loro contributi. Siccome sono messi in posizioni diverse
hanno diversi tempi
P
ritardati e la cancellazione non è totale: c’è un irraggiamento da quadrupolo Qij = qe [3xi xj − r2 δij ]
˙2
1 2 p̈2
Q̈
W =
+
+ ···
4π0 3 c3
60c5
per cui alla fine Eirr /T0 ∼ (v0 /c)5 invece che (v0 /c)3 .
92
Capitolo 13. Irraggiamento
Esercizio 155: Onde gravitazionali
Stimare la potenza irraggiata in onde gravitazionali
bSoluzione: Gli esercizi precedenti invitano ad una digressione sull’irraggiamento di onde gravitazionali, dove
si ha un fenomeno analogo all’irraggiamento con 1/0 → G e q → m:
˙2
P̈ 2
Q̈
W ∼ G 3 + 5 + ···
c
c
Nel caso gravitazionale l’irraggiamento da dipolo è sempre zero in quanto il momento è costante. (In linguaggio
profondo ma per ora incomprensibile il gravitone ha spin 2, mentre il fotone ha spin 1). L’irraggiamento da
quadrupolo può essere stimato come
Wirr ∼
2
Wint
W0
dove
W0 =
c5
= 3.62 1052 Watt
G
˙ ha il significato fisico di potenza interna (non sferica) del sistema. Ad esempio due corpi di masse M
e Wint ∼ Q̈
in orbita a distanza R l’uno dall’altro sentono una forza F ∼ GM 2 /R2 ed hanno velocità data da M ∼ v 2 R/G:
quindi Wint ∼ F v ∼ v 5 /G = W0 (v/c)5 < W0 in quanto v < c (quando si raggiunge v ∼ c il sistema collassa in
un buco nero). La costante universale W0 ha quindi il significato fisico di massima potenza possibile.
Nel caso del sistema Terra/Sole, bisogna tenere conto che hanno masse diverse: = MT /MS ∼ 10−6 . Questo
produce Wint = W0 (v/c)5 . Essendo v/c ≈ 10−4 si ha Wirr ∼ 2 (v/c)10 W0 ∼ Watt, meno di una lampadina. A
differenza dell’analogo elettromagnetico nell’atomo di idrogeno non è un fenomeno preoccupante.
Esercizio 156: Scattering elettrone/fotone
bSoluzione: Se a = qe E/m si ottiene
σ=
W
4π0 c(2/3)(e2 /mc2 )2 hE 2 i
8π 2
=
=
r
S
0 c2 hEBi
3 e
re = e2 /mc2 = 2.82 10−13 cm è il raggio classico dell’elettrone. Vale solo se si può trascurare il rinculo
dell’elettrone (cioè raggi γ).
Se illumino un atomo di idrogeno il protone non conta nulla (a energie maggiori di 13.6 eV, mentre a energie
minori conta solo lo stato legato).
Se illumino un atomo di elio ho 2 elettroni che possono irraggiare in fase. In generale Z 2 or Z.
Esercizio 157: Polarizzazione della CMB
bSoluzione: da fare
Esercizio 158: Un’antenna
bSoluzione: Un’antenna che contiene una corrente oscillante I = I0 eiωt (1−2|z|/`) contiene una carica ρ̇ = −∇·
R `/2
j = −∂I/∂z = (z/|z|)2I0 eiωt /L e quindi produce un dipolo elettrico oscillante p = −`/2 ρz dz = I0 `eiωt /2iω.
Secondo la formula di Larmor un singolo dipolo elettrico p irraggia in direzione θ rispetto al dipolo
dW
hp̈2 i sin2 θ
=
dΩ
4πc3 · 4π0
W =
I02
Rrad
2
Rrad =
(k`)2
≈ 5(k`)2 ohm
6c · 4π0
Segnali TV hanno λ ∼ (10÷100) m, quindi se un albero si interpone fra l’antenna ricevente e quella trasmittente
non è un problema serio.
Capitolo 13. Irraggiamento
93
Esercizio 159: Due antenne
Due antenne piccole rispetto a λ situate a distanza λ/4 irraggiano con dipoli uguali eccetto una differenza di
fase di 90◦ . Calcolare dW/dΩ ed il momento irraggiato
bSoluzione:
Due dipoli irraggiano un campo elettrico
E = E 1 + E2 = E 1 (1 − eiδ )
δ=
π 2π λ
π
+
cos θ = (1 + cos θ)
2
λ 4
2
Quindi
π
dW
dW1
dW1
ω 2 I02 L2
2
iδ 2
sin θ 1 + sin( cos θ)
=
|1 − e | =
2(1 − cos δ) =
dΩ
dΩ
dΩ
16πc3
2
Cioè non è simmetrica. Viene quindi anche emesso un momento
dpz
1 dU
π2
=
(1 −
)
dt
c dt
12
che per W ∼ 105 W vale Fz ∼ 10−4 N.
Esercizio 160: Dipolo magnetico
Pulsar ruotante
bSoluzione: Una pulsar ruota facendo girare anche il suo dipolo magnetico µ. Quindi irraggia W = 2µ̈2 /3c3 =
2ω 4 µ2 /3c3 riducendo l’energia cinetica rotazionale U = Iω 2 /2 (I ≈ 25 M R2 ). Se ω̇ è misurato si ricava
µ2 =
3 M R2 ω̇c3
5
ω3
per M ∼ M , R ∼ 10 km, T ∼ 10 s, Ṫ ∼ 10−10 .
B∼
2µ
∼ 1015 Gauss
R3
Capitolo 14
Relatività
Introducendo Aµ = (ϕ, A) le equazioni E = −Ȧ/c − ∇ϕ e B = ∇ × A diventano


0
0
E

F µν = ∂ µ Aν − ∂ ν Aµ =  x

Ey B z
0
Ez −By Bx 0
Le equazioni di Maxwell sono
∂µ F µν =
Le trasformazioni di Lorentz dei campi sono

γ
−βγ
0 0
0
0
−βγ
γ

F µ ν = Λµµ Λνν F µν
Λ(βx ) = 
0
0
0
0
4π ν
J
c

0 0
0 0

1 0
0 1
J ν = (cρ, J )
i.e.
Bx0 = Bx
Ex0 = Ex
0
Ey = γ(Ey − βBz ) By0 = γ(By + βEz )
Ez0 = γ(Ez + βBy ) Bz0 = γ(Bz − βEy )
L = − 41 F µν Fµν = (E 2 − B 2 )/2 e µναβ F µν F αβ ∝ E · B sono invarianti di Lorentz. Per un’onda valgono zero.
(Sporco trucco: F ≡ E + iB fa rotazioni con angolo complesso, quindi F 2 è invariante).
Non useremo le seguenti formule. Quadri-corrente di una carica puntiforme q in moto Xµ (τ ) arbitrario:
Z
Jµ = q dτ Vµ δ(xµ − Xµ (τ )).
1
Per moto rettilineo uniforme a velocità v si riduce all’ovvio Jµ = q
δ(x − vt)δ(y)δ(z), come può verificare
v
1
trasformando Jµ0 = q
δ(x0 )δ(y 0 )δ(z 0 ).
0
Il vettore di Poynting fa parte del tensore simmetrico ‘energia impulso’ Tµν che trasforma come
0
T00
= γ 2 (T00 − 2T0x β + Txx β 2 ),
0
Txx
= γ 2 (Txx − 2T0x β + T00 β 2 ),
0
T0y
= γ(T0y − Txy β),
0
T0x
= γ 2 [T0x (1 + β 2 ) − β(T00 + Txx )]
0
Txy
= γ(Txy − T0y β),
0
Tyy
= Tyy
e lo stesso per y → z.
Esercizio 161: Contrazione di Lorentz
Verso il 1900 si discuteva il seguente problema: assumendo che la materia sia tenuta assieme da forze elettromagnetiche, e sapendo come queste si trasfromano in diversi sistemi di riferimento determinare in che modo la
materia si ingrossa o rimpicciolisce se vista da un sistema in moto.
bSoluzione: Una volta capito, il problema diventa banale. Siccome l’elettromagnetismo ed il resto della fisica
trasformano in modo ben definito sotto trasformazioni di Lorentz, la distanza fra due punti di un oggetto forma
un quadrivettore Xµ , che si trasforma come un quadrivettore indipendentemente dalle forze complicate Fµν
che lo tengono assieme. E.g. è ovvio che sotto rotazioni la distanza è un invariante: il resto non è molto più
profondo.
94
Capitolo 14. Relatività
95
Esercizio 162: Che cosa è l’elettromagnetismo
Mostrare che l’elettromagnetismo è l’unica teoria relativistica di un campo vettore.
bSoluzione: In meccanica classica (ma specialmente in meccanica quantistica) un modo conveniente di descrivere una teoria consiste nello scriverne la Lagrangiana L (per quanto riguarda la presente discussione una
Lagrangiana è sostanzialmente l’energia cinetica). Se ad esempio la teoria ha una qualche simmetria può non
essere ovvio vederla dalle equazioni del moto. che la Lagrangiana. Ci interessa il caso in cui la simmetria è
l’invarianza di Lorentz: quindi la Lagrangiana deve essere uno scalare.
Nel caso una teoria relativistica di un campo scalare φ la naturale Lagrangiana è L = ±(∂µ φ)2 , con il segno
fissato ad essere + in modo che l’energia cinetica sia positiva, L = +φ̇2 + · · ·.
Per un vettore Aµ apparentemente si ha L = ±(∂µ Aν )2 , ma questa teoria non ha senso: infatti per
qualunque segno o la componente A0 o le componenti A1,2,3 hanno energia cinetica con segno sbagliato. L’unica
teoria sensata è data da L = −(∂µ Aν − ∂ν Aµ )2 . Questa forma speciale corrisponde ad una nuova simmetria ‘di
gauge’ Aµ → Aµ + ∂µ φ, necessaria (specialmente a livello quantistico) per giustificare la speciale forma sensata.
La simmetria di gauge vincola altri termini addizionali:
• Il fotone deve avere massa zero. Infatti il termine Lorentz-invariante m2 A2µ /2 (m ha dimensioni lunghezza−1 )
viene proibito. Questo sarebbe un termine di massa per il fotone. Infatti in sua presenza il quadri-vettore
d’onda che compare in eiK·X soddisferebbe a K 2 = m2 , e quindi ω/c > m. Quantisticamente il quadriimpulso è Pµ = h̄Kµ , per cui h̄m è una massa. Nel limite statico (ω = 0) si avrebbe k = im, cioè la forza
di Coulomb sarebbe ∝ e−rm /r2 .
• L’accoppiamento alla materia Jµ Aµ rispetta questa simmetria se ∂µ Jµ = 0: la carica elettrica deve essere
conservata.
Quando viene sviluppata anche una teoria della materia Jµ diventa Ψ̄γµ Ψ e l’invarianza di gauge diventa una
simmetria locale di Ψ: U(1) nel caso dell’elettromagnetismo, SU(2) per le interazioni elettro-deboli, SU(3) per
quelle forti, e Poincarè per la gravità.
Esercizio 163: Forza fra 2 cariche bis
Due elettroni si muovono parallelamente lungo traiettorie rettilinee a distanza a con velocità costante v c.
Calcolare la forza elettromagnetica
bSoluzione: Modo 1: Nel sistema in cui le cariche sono in quiete F0 = e2 /4π0 e quindi la relatività dice che
Fv = F0 /γ
Modo 2: Nel sistema di quiete esiste solo E. Trasformando i campi trovo che nel sistema in cui le cariche si
muovono Ey0 = γEy e Bz0 = −γβEy : la forza di Lorentz è
Fv = γ(
e2
µ0
e2
v2
F0
− ev ·
ev) =
γ(1 − 2 ) =
4π0
4π
4π0
c
γ
Per calcolare il segno basta ricordare che fili con correnti uguali si attirano.
Negli acceleratori di particelle si riesce ad accelerare fasci di particelle cariche, perchè a v ∼ c la forza
repulsiva di Coulomb è compensata da quella magnetica.
Esercizio 164: Scattering debole bis
Una carica q viaggia lungo l’asse x con energia E e ‘grande’ parametro d’impatto b verso una carica Q, ferma
nell’origine e di massa cosı́ grande che rimane a riposo. Calcolare il piccolo angolo di deflessione e verificare che
l’impulso acquistato da q è uguale ed opposto a quello acquistato da Q.
bSoluzione: La formula dp/dt = F = q(E + v × B) è vera anche relativisticamente, dove p = mγv è l’impulso
relativistico.
96
Capitolo 14. Relatività
Per ‘grande’ b la carica q viene deflessa di poco e si ha θ(b) ' ∆pq⊥ /p 1 con
Z +∞
Z +∞
qQ
b dt
qQ
Q
dt qE⊥
(x = vt, y = b, 0, t) =
∆pq⊥ =
=
4π0 −∞ [(vt)2 + b2 ]3/2
2π0 vb
−∞
Il calcolo è stato già effettuato a pagina 13, mostrando che uno può fare l’integrale col il teorema di Gauss.
L’impulso acquistato da Q si calcola in modo analogo, in termini del campo elettrico generato dalla carica
q in moto relativistico: partendo dal sistema S 0 dove q è ferma (x0 = γ(x − vt)) le trasformazioni di Lorentz di
E dicono che E⊥ è innalzato da un fattore γ:
Z +∞
Z +∞
γb dt
qQ
q
= ∆pq⊥
∆pQ
=
dt
Q
E
(0,
0,
0,
t)
=
⊥
⊥
4π0 −∞ [(γvt)2 + b2 ]3/2
−∞
Esercizio 165: Carica in E e B ortogonali bis
Estendere l’esercizio di pagina 52 al caso di moto relativistico.
bSoluzione: Avevamo visto che una carica in E e B ortogonali spiraleggia driftando a velocità costante,
indipendente dalla carica e dalla massa. Questo diventa ovvio√riassorbendo il drift tramite una trasformazione di
Lorentz con velocità E/B. Nel nuovo sistema E 0 = 0 e B 0 = B 2 − E 2 (se B > E) come segue immediatamente
dal fatto che E ·√B = 0 ed E 2 − B 2 sono invarianti di Lorentz. Se E > B si può andare in un sistema dove
B 0 = 0 ed E 0 = E 2 − B 2 tramite un boost di velocità c2 B/E. Fisicamente questo è dovuto al fatto che se il
campo elettrico è troppo grosso, E > B, il campo magnetico non riesce ad incurvare la traiettoria.
Esercizio 166: Filo in moto
Un filo rettilineo infinito disposto lungo l’asse x ha sezione A e contiene n elettroni per unità di lunghezza in
moto con velocità v, e n protoni fermi. Il filo viene messo in modo con velocità β lungo l’asse x. Calcolare i
campi E e B.
bSoluzione: Nel sistema S dove il filo è fermo il vettore quadricorrente J = (ρ, j) vale J = (0, i/A) dove
i = nev. Questo produce E = 0 e B = θ̂µ0 i/2πr.
1. Un primo modo di calcolare i campi nel sistema S 0 rispetto al quale il filo si muove con velocità β. Uno
può pasticciare nel tradurre le trasformazioni di Lorentz dei campi in coordinate cilindriche, per evitare
rogne è bene tenere in conto che il sistema ha simmetria cilindrica e che E 2 − B 2 e E · B sono invarianti.
Il risultato è
Er0 = −γβBθ ,
Bθ0 = γBθ
(14.1)
2. Un secondo modo consiste nel trasformare il quadrivettore J, ottenendo
ρ0 = −γβjx ,
jx0 = γjx
(14.2)
da cui è immediato riottenere la (14.1): jx0 e quindi Bθ0 diventa γ volte più grosso, e la densità di carica
lineare λ0 = Aρ0 = −γβnev genera il campo elettrico Er0 = λ0 /2π0 r calcolato a pagina 7.
3. Un terzo modo piuttosto rognoso, ma che consente di capire da dove salta fuori la carica, consiste nel
trasformare le singole particelle. Nel sistema S 0 i protoni hanno carica +e, velocità β e densità n0+ = γn,
perchè la lunghezza si contrae. Nel sistema S 0 gli elettroni hanno carica −e, velocità v 0 = (β + v)/(1 + βv)
(formula di addizione delle velocità) e densità n0− = nγ(v 0 )/γ(v). Per calcolare n0− conviene considerare
il sistema S 00 rispetto al quale gli elettroni sono fermi e n00− = n/γ(v), con γ(v) = (1 − v 2 )−1/2 . Quindi la
densità di carica del filo nel sistema S 0 vale
λ0 = λ+ γ(β) + λ−
γ(v 0 )
= −neγ(β)vβγ(β)
γ(v)
(utilizzando λ+ = −λ− = ne e γ(v 0 ) = γ(v)γ(β)(1 + vβ)) in accordo con la (14.2) e quindi con la (14.1).
Questo esercizio illustra che tutto è consistente, e che utilizzare le leggi di trasformazione dei campi è molto più
rapido.
Capitolo 14. Relatività
97
Esercizio 167: Forza prodotta da filo in moto
Una particella di carica q è in quiete a distanza r dal filo dell’esercizio precedente. Calcolare la forza che agisce
sulla carica.
bSoluzione:
1. Fr0 = qEr0 dove Er0 è stato calcolato in vari modi all’esercizio precedente.
2. Alternativamente si può calcolare la forza nel sistema S dove il filo è fermo e la carica in modo con velocità
−β lungo l’asse x. La forza di Lorentz è diretta lungo r e vale Fr = −qβBθ . Trasformando la forza al
sistema S 0 si ottiene Fr0 = γFr in accordo con il risultato precedente.
Esercizio 168: Onda vista da sistema in moto
Un’onda si muove lungo K = ω(1, nx , ny , 0) con B = (0, 0, Bz ) e E = B(ny , −nx , 0). Come diventa se vista da
un sistema in moto lungo l’asse x con velocità β?
bSoluzione: Siccome K è un quadri-vettore
K 0 = ω(γ(1 − nx β), γ(nx − β), ny , 0)
i.e.
ω 0 = ω − nx β
ny 0
1
1
=
nx0
γ (nx /ny ) − β
Il campo magnetico diventa
B 0 = (0, 0, Bz0 ) = γBz (1 − βny )(0, 0, 1)
0
Per finire il campo elettrico deve diventare E 0 = B 0 (n0y , −n0x , 0), dovendo essere E 0 = k̂ × B 0 .
Esercizio 169: Riflessione da specchio in moto
Un’onda elettromagnetica di frequenza ω si muove in direzione n = (nx , ny , 0) con il campo elettrico polarizzato
lungo l’asse z. Uno specchio è situato a x = 0 nel piano yz.
1) Calcolare direzione e campi elettromagnetici dell’onda riflessa.
Lo specchio viene ora messo in moto con velocità costante v lungo l’asse x.
2) Calcolare direzione, frequenza e campo elettrico dell’onda elettromagnetica incidente rispetto al sistema
di riferimento in cui lo specchio è fermo.
3) Calcolare direzione, frequenza e campo elettrico dell’onda riflessa rispetto al sistema in cui lo specchio è
in moto.
bSoluzione: Al bordo con una superficie riflettente E= = B⊥ = 0: quindi se mando un’onda, l’onda riflessa
ha E= e B⊥ invertiti. Ricordo la riflessione da uno specchio fermo:
kxr = kxi ,
kzr = −kzr ,
E r = −E i
Poi E · B e E 2 − B 2 sono invarianti...
Esercizio 170: Aberrazione relativistica
Siccome i fotoni emessi da dietro fanno un viaggio più lungo un cubo che viaggia a velocità β viene visto
ruotato di un angolo α → 90 per β → 1 (e con velocità appartente > c). L’effetto geometrico banale e quello di
contrazione si sommano a dare questo. NON viene visto contratto (Einstein trascurava l’effetto banale). Una
sfera rimane una sfera.
bSoluzione: Si tiene conto di questo effetto quando si osservano i getti emessi da un nucleo galattico attivo.
98
Capitolo 14. Relatività
Esercizio 171: π 0 → 2γ
Dedurre la partità del π 0 dalla misura della polarizzazione della luce emessa nel suo decadimento.
bSoluzione: Il π 0 decade in due onde elettromagnetiche (‘fotoni’ γ); per la conservazione dell’impulso le due
onde hanno eguali intensità se il π 0 è a riposo. Possono però avere diversa polarizzazione: chiamiamo θ l’angolo
fra i campi elettrici E 1 e E 2 delle due onde. Ha interesse calcolare in funzione di θ quanto valgono gli invarianti
di Lorentz: scalare e pseudo-scalare:
E 2 − B 2 ∝ Fµν F µν ,
E · B ∝ µνρσ F µν F ρσ
dove E = E 1 + E 2 e B = B 1 + B 2 .
1 Se θ = 0 (fig. a sinistra)si ha E 1 = E 2 ; tenendo conto che i due γ hanno k opposti i campi magnetici sono
anti-paralleli: B 1 = −B 2 . Quindi E 2 − B 2 6= 0 e E · B = 0.
E2
B2
π0
E1
E2
π0
B1
E1
B2
B1
2 Se θ = π/2 (fig. a destra) i campi sono ortogonali: E 1 ⊥ E 2 e B 1 ⊥ B 2 . Questo produce E 2 − B 2 = 0 e
E · B 6= 0.
3 Per θ generico si ha Ê 1 = (1, 0), B̂ 1 = (0, 1), Ê 2 = (cos θ, sin θ), B̂ 2 = (sin θ, − cos θ). Questo produce sia
E · B ∝ sin θ che E 2 − B 2 ∝ cos θ.
Sperimentalmente misurando le polarizzazioni dei due fotoni emessi in decadimenti π 0 → γγ si trova che
esse sono ortogonali (cioè θ = π/2 come nel caso 2) e che quindi il decadimento del π 0 genera un campo
elettromagnetico che ha diverso da zero l’invariante pseudo-scalare E · B. Andando avanti si concluderebbe che
il π 0 è uno pseudo-scalare accoppiato all’elettromagnetismo tramite interazione Lagrangiana π 0 µναβ F µν F αβ
e che alcune interazioni fondamentali violano la parità (cioè un esperimento dà risultati diversi dello stesso
esperimento costruito in modo speculare).
Esercizio 172: GZK
Un protone urta frontalmente un fotone di energia Eγ = 10−12 mp c2 , dove mp è la massa del protone.
1. Quale energia Ep deve avere il protone affinchè il processo pγ → ∆ sia possibile, se m∆ = 1.3mp ?
2. La ∆ viene prodotta e successivamente decade in un protone p0 ed in un fotone γ 0 . Quali energie hanno
le particelle di decadimento, nel sistema di quiete della ∆?
3. Nel sistema di quiete della ∆, p0 e γ 0 vengono emessi in direzione ortogonale alle velocità delle particelle
p e γ originarie. Quali energie Ep0 ed Eγ 0 hanno le particelle di decadimento nel sistema iniziale?
bSoluzione: È un problema di soglia più difficile del solito perchè il sistema rilevante non è nè il CM nè il lab
(ma è il sistema in cui le stelle sono ferme).
1. Predendo il modulo quadro di Pp + Pγ = P∆ trovo
2(Ep − pp cos θ) =
m2∆ − m2p
Eγ
Il protone è ultra-relativistico: pp ≈ Ep . La minima Ep è richiesta quando cos θ = −1 (scontro frontale).
Quindi protoni con energia maggiore di Ep > (m2∆ − m2p )/4Eγ = 5.5 1011 GeV vengono assorbiti.
Capitolo 14. Relatività
99
2. Da P∆ = Pp0 + Pγ0 si ha Pγ02 = (P∆ − Pp0 )2 cioè, nel sistema del CM in cui la ∆ è ferma 0 = m2∆ + m2P −
0
0
0
0
2m∆ EpCM
/c2 , da cui EpCM
/c2 = (m2∆ +m2p )/2m∆ . Procedendo in modo simile, o usando EγCM
+EpCM
=
2
0
2
2 2
m∆ c si trova EγCM = (m∆ − mp )c /2m∆ .
0
0
0
3. Facendo una trasformazione di Lorentz E 0 = γ(ECM
+ vCM p0xCM ) = γECM
dove ECM
sono le energie
2
2
11
calcolate nel CM al punto 2. Il fattore γ vale E∆ /m∆ c ≈ Ep /m∆ c ≈ 10 . Qualitativamente Ep0 ∼
Eγ0 ∼ γm∆ . Quindi nel processo (ogni volta che l’angolo di decadimento nel CM è diverso da 0) il fotone
ha acquistato energia a spese del protone.
Questo problema è attualmente anche uno dei problemi della cosmologia. L’universo è riempito da ‘radiazione
cosmica di fondo’ (fotoni con energia Eγ ≈ 2.73 K) e da raggi cosmici (protoni). I protoni possono venir
distrutti tramite il processo pγ → ∆. Quindi in questo problema abbiamo mostrato che non dovrebbero esistere
raggi cosmici di energia superiore a 5 1010 GeV (numero ufficiale ottenuto da calcolo esatto nel 1966: GreisenZatsepin-Kuzmin (GZK) cutoff), o —più precisamente, tenendo conto della densità di fotoni — che possono
arrivare sulla terra solo se prodotti entro d = 1/nγ σ ∼ 10 Mpc dove nγ ∼ 400/ cm3 e σpγ ∼ 10−28 cm2 . (In
realtà il decadimento dominante è ∆ → N π invece di ∆ → pγ, ma questo rafforza il bound) Il problema è
che raggi cosmici (protoni?) sopra il GZK cutoff sono stati osservati, mentre non si osserva nessuna possibile
sorgente abbastanza vicina. Una possibile spiegazione è che intensi campi magnetici extra galattici facciano fare
giri bislacchi ai protoni, per cui non arrivano nè dalla stessa direzione nè allo stesso tempo dei fotoni emessi
dalla sorgente che non si riesce a vedere e che quindi può essere morta nel frattempo. Altra possibile soluzione
è che la relatività sia sbagliata: nessuno la ha mai testata ad E/mc2 tanto alti. Altra possibilità è che siano
prodotti vicino alla Terra dal decadimento di particelle relitte di enorme massa.
Esercizio 173: Effetto Compton
Mandando raggi X su elettroni fermi nel 1923 Compton trovò λ0 (θ) = λ+0.024Å(1−cos θ). Si mostri che questa
è la relazione cinematica per γe → γe considerando il fotone come una particella di energia E = hν = hc/λ.
bSoluzione: Scrivo Pγ + Pe = Pγ0 + Pe0 con
Pγ = (E/c, E/c, 0, 0),
Pe = (me c, 0, 0, 0),
Pγ0 = (E 0 , E 0 cos θE 0 sin θ)/c
(mostrare che il numero incognite = equazioni +1, come dovrebbe essere). Siccome Pe0 non mi interessa ricavo
direttamente il risultato riscrivendo la conservazione del quandri-impulso come Pe0 = (Pγ − Pγ0 + Pe ) e prendendo
il modulo quadro:
m2e = m2e + 0 + 0 + 2me (E − E 0 ) + 2EE 0 (cos θ − 1)
cioè
1
1
1 − cos θ
=
+
,
0
E
E
me c2
λ0 = λ − λCompton (1 − cos θ)
dove l’ultima equazione è stata ricavata usando l’equazione quantistica (compatibile con la relatività!) λCompton =
h/me c2 = 2.4 10−12 m. E = hν significa che non posso affievolire la luce sotto un certo limite.
Insieme all’effetto fotoelettrico questo processo mostrò in modo diretto che la luce ha anche natura particellare, cosa per la prima volta proposta in modo troppo conservatore da Planck in un tentativo di spiegare lo
spettro di corpo nero.
Esercizio 174: Esperienza d Fizeau
bSoluzione: Usando la formula di addizione delle velocità
v 0 = (v + c/n)/(1 + v/nc)
t = (Ln/c)(1 + v/nc)/(1 + vn/c) ≈ T0 [1 − (v/c)(n − 1/n)]
Quindi ∆ϕ = 2πν ∆t = (2πL/λ)(v/c)(n2 − 1).
100
Capitolo 14. Relatività
Esercizio 175: Iraggiamento da elettroni relativistici
Quale è la massima energia raggiungibile da un acceleratore di elettroni?
bSoluzione: Siccome W = dE/dt = dE 0 /dt0 è uno scalare di Lorentz, la generalizzazione relativistica è
2 e2 dPµ dPµ
2 e2
W =−
=
3 m2 c3 dτ dτ
3 m2 c3
dp
dτ
2
1
− 2
c
dE
dτ
2 dove dτ = dt/γ e P = (E/c, p) = mγ(c, v). Specializziamo questa formula generale alle due tecnologie possibili
di acceleratori di particelle: lineare e circolare.
In un acceleratore lineare, usando dE = v dp (che segue da 0 = d(E 2 /c2 − p2 ) = 2(E dE/c2 − p dp)) la
formula può essere riscritta in modo uguale a quella non relativistica
W =
2 e2
3 m2 c3
dp
dt
2
=
2 e2
3 m2 c3
dE
dx
2
W
2 e2 1 dE
dE/dx
=
≈
1
2
3
dE/dt
3 m c v dx
mc2 /re
In un acceleratore si riescono a produrre campi elettrici tanto intensi da accelerare un elettrone da fermo a
relatvistico in 10 cm: cioè si raggiunge dE/dx = me c2 /` con ` ∼ 10cm. Quindi la frazione di energia persa per
irraggiamento è re /` ∼ 10−13 : completamente trascurabile. Il problema è che per accelerare elettroni fino ad
energie mai raggiunte prima (E ∼ 106 me c2 ) serve una lunghezza 106 ` ∼ 100km, il che costa 1010 e.
In un acceleratore circolare domina il termine |dp/dτ | = γ · ω· = γ · (v/R) · mγv = mγ 2 a, con a = v 2 /R.
Quindi W è dato dalla formula non relativistica maoltiplicata per γ 4 e
dE
W
2e2 4 3
=
=
γ β
dx
v
3R2
Quindi il massimo γ raggiungibile accelerando elettroni in un acceleratore circolare di raggio R e con gradiente
dE/dx vale
s
s
r
R
R
4 dE/dx|max
5
√
γmax ≈
=
∼ 6 10
e2 /R2
5 km
re `
avendo usato e2 /R2 = me c2 re /R2 . LEP con R di qualche km ha raggiunto 2 105 con una corrente di qualche mA.
La parte dell’acceleratore che costa più energia è il raffreddamento dei magneti superconduttori che producono
campi magnetici di ∼ 1 Tesla, necessari per far girare gli elettroni. Notare che il γmax non dipende dalla massa
della particella: per raggiungere energie elevate conviene usare particelle pesanti: Emax ∼ mc2 γmax . Nel 2007
nell’ex anello di LEP, LHC accelererà protoni fino a γmax ∼ 104 .
Esercizio 176: Miraggi
bSoluzione: Ricordo che in unità c = 1 k = nω con n grosso in zone più dense.
Quando la luce entra in zone poco dense rimbalza.
La fase dell’onda K · X è uno scalare e quindi K = (ω, k) = ω(1, n) è un quadrivettore. Se kz → 0 l’onda
torna indietro. Se l’onda va storta cioè kx 6= 0 kz2 = (nω)2 − kx2 questo succede non per n = 0 ma per n = kx /ω
che è tipicamente di ordine uno per angoli di ordine uno.
Altro modo: vado nel sistema dove kx0 = 0. Sfruttando l’invarianza di K 2 e di kz in tale sistema ω 02 = ω 2 −kx2
i.e.
k2
n2 − β 2
kx
k0
→0
β≡
n02 = ( z0 )2 = 2 z 2 =
ω
ω − kx
1 − β2
ω